4 - MRCP - Part One - Past-Papers - Jan 2016

You might also like

Download as pdf or txt
Download as pdf or txt
You are on page 1of 279



Question bank
PAST-PAPERS
‫‪‬إ‪‬اء‪‬‬
‫‪ ‬آﯾﺔ ﻣﻦ ﻛﺘﺎب ﷲ ﺣﻔﻈﻨﺎھﺎ وﻟﻜﻞ ﻣﻌﻠﻮﻣﺔ ﻗﺮأﻧﺎھﺎ وﻧﺴﯿﻨﺎھﺎ ﻟﻌﻞ ﷲ ﯾﺤﻤﯿﻨﺎ ﻣﻦ اﻟﻨﺴﯿﺎن وﯾﺮدھﺎ إﻟﯿﻨﺎ‬
‫‪ ‬ﺷﺨﺺ دﻋﻮﻧﺎ ﻟﮫ ﻓﻲ ظﮭﺮاﻟﻐﯿﺐ ﻟﻌﻞ ﷲ أن ﯾﺴﺘﺠﯿﺐ دﻋﺎﺋﻨﺎ‬
‫‪ ‬ﻣﺮﯾﺾ ﻗﺪ آﻟﻤﮫ ﻣﺎ أﻟﻤﮫ ﻟﻌﻞ ﷲ ﯾﺸﻔﯿﮫ وﯾﻌﯿﺪ إﻟﯿﮫ ﺻﺤﺘﮫ وﻋﺎﻓﯿﺘﮫ‬
‫‪ ‬ﻟﺤﻈﺔ ﺗﺂﻟﻢ ﺑﮭﺎ أﺧﻲ وﺟﺪي وأﺣﺒﺘﻲ أﺛﻨﺎء ﻋﻼﺟﮭﻢ ﻟﻌﻞ ﷲ ﻻ ﯾﺮﯾﮭﻢ اﻵﻟﻢ وﻻ ﯾﻀﯿﻤﮭﻢ ﻣﺮة أﺧﺮى‬
‫‪ ‬ﻣﻮﻗﻒ اﺣﺘﺠﺖ ﺑﮫ ﻋﯿﻨﺎي ﻓﻠﻢ ﺗﻠﺒﯿﺎﻧﻲ ﻟﻌﻞ ﷲ ﻻ ﯾﻀﺮﻧﻲ ﺑﮭﻤﺎ ﻓﻲ ﺣﯿﺎﺗﻲ‬
‫‪ ‬ﺣﺎﻟﻢ ﺻﺎﺣﺐ أھﺪاف وﻣﺒﺎدئ ﻟﻌﻞ ﷲ ﯾﺤﻘﻖ ﻟﮫ ﻣﻘﺎﺻﺪه ﺣﺘﻰ ﯾﺮﺿﯿﮫ وﯾﺮﺿﻰ ﻋﻨﮫ‬
‫‪ ‬ﻣﺠﺘﮭﺪ ﻟﻌﻞ ﷲ ﯾﺠﻌﻞ ﻟﮫ ﻣﻦ اﻟﺘﻮﻓﯿﻖ أوﻓﺮ ﺣﻆ و ﻧﺼﯿﺐ وﯾﮭﻮن ﻟﮫ اﻷﺳﺒﺎب ﻛﻠﮭﺎ ﻣﻦ ﺣﯿﺚ ﻻ‬
‫ﯾﺪري ﻣﻦ ﻏﯿﺮ ﺿﺮاء ﻣﻀﺮة وﻻ ﻓﺘﻨﺔ ﻣﻀﻠﮫ‬
‫‪ ‬ﻣﻦ ﻓﻘﺪ ﺷﯿﺌﺎ ﻓﻲ اﻟﺤﯿﺎة ﻟﻌﻞ ﷲ ﯾﻌﯿﺪه إﻟﯿﮫ أو ﯾﻌﻮﺿﮫ ﺑﻤﺎ ﯾﺴﻌﺪه‬
‫‪ ‬ﺷﺨﺺ اﺑﺘﻌﺪﻧﺎ ﻋﻨﮫ ﻟﻌﻞ ﷲ ﯾﺠﻤﻌﮫ ﺑﻤﻦ ھﻢ أﻓﻀﻞ ﻣﻨﺎ‬
‫وأ‪‬ا وﻟﯿﺲ آﺧﺮا ﻷھﻠﻨﺎ ﻟﻌﻞ ﷲ ﯾﺠﻌﻠﮭﺎ ﻟﮭﻢ ﺻﺪﻗﺔ ﺟﺎرﯾﮫ ﺗﺮد إﻟﯿﮭﻢ ﻗﻠﯿﻼ ﻣﻦ ﻓﻀﻠﮭﻢ ﻋﻠﯿﻨﺎ وﺗﻜﻮن ﺳﺒﺒﺎ‬
‫ﯾﻠﻢ ﺷﻤﻠﻨﺎ ﺗﺤﺖ ﺳﻘﻒ واﺣﺪ ﺑﻌﺪ أن ھ ﱠﺠ َﺮﺗﻨﺎ اﻟﺤﺮوب واﻟﻤﺼﺎﺋﺐ‬

‫‪PT Part I 2019 index‬‬


‫‪Branch‬‬ ‫‪No. of Q‬‬
‫‪Cardiology‬‬ ‫)‪(386‬‬
‫‪Clinical haematology/oncology‬‬ ‫)‪(392‬‬
‫‪Clinical‬‬
‫)‪(513‬‬
‫‪pharmacology/therapeutics/toxicology‬‬
‫‪Dermatology‬‬ ‫)‪(138‬‬
‫‪Endocrinology‬‬ ‫)‪(414‬‬
‫‪Gastroenterology‬‬ ‫)‪(403‬‬
‫‪General Revision‬‬ ‫)‪(122‬‬
‫‪Infectious diseases/sexually‬‬
‫)‪(412‬‬
‫‪transmitted diseases‬‬
‫‪Nephrology‬‬ ‫)‪(335‬‬
‫‪Neurology‬‬ ‫)‪(413‬‬
‫‪Ophthalmology‬‬ ‫)‪(81‬‬
‫‪Psychiatry‬‬ ‫)‪(268‬‬
‫‪Respiratory Medicine‬‬ ‫)‪(457‬‬
‫‪Rheumatology‬‬ ‫)‪(380‬‬
Clinical Sciences
Cell/molecular and membrane
(72)
biology
Clinical anatomy (67)
Clinical biochemistry and metabolism (118)
Clinical physiology (92)
Genetics (107)
Immunology (118)
Statistics (158)

Past-Papers index
Passing mark range between 63-66%
Year No. of Q
MRCP 1 May 19 100
MRCP 1 Jan 19 100
MRCP 1 Sept 18 100
MRCP 1 May 18 100
MRCP 1 Jan 18 100
MRCP 1 Sept 17 100
MRCP 1 May 17 100
MRCP 1 Jan 17 100
MRCP 1 Sept 16 100
MRCP 1 May 16 100
MRCP 1 Jan 2016 100
MRCP 1 Sept 2015 100
MRCP 1 May 2015 100
MRCP 1 Jan 2015 100

A. H. Murad
‫ﻻ ﺗﻨﺴﻮﻧﺎ ﻣﻦ ﺻﺎﻟﺢ دﻋﺎﺋﻜﻢ‬
‫ﺗﻢ ﺑﺤﻤﺪ ﷲ وﺗﻮﻓﯿﻘﮫ وﻣﻨﮫ‬
‫ﺟﻌﻞ ﷲ ﻋﻤﻠﻨﺎ ﻣﺘﻘﺒﻼ ﺧﺎﻟﺼﺎ ﻟﻮﺟﮭﮫ اﻟﻜﺮﯾﻢ‬
0:00:36/03:00:00

You are examining the possible impact of immunomodulators in the treatment of


atherosclerosis and are interested in how white cells play a role in the process.
Which white cells predominate in the tunica intima?

A Basophils

B B lymphocytes

C Eosinophils

D Macrophages

E T lymphocytes

Explanation 

D Macrophages

Foam cells are fat laden macrophages found in atherosclerotic plaques. They engulf oxidised
LDL by a process of endocytosis via scavenger receptors and are associated with necrotic
foci, driving an increase in local inflammatory cytokines, and potentially plaque rupture.

A Basophils

Basophils is incorrect. Basophils are not the predominant subclass of white cell involved in
atherosclerosis, but may be recruited into atherosclerotic plaques in response to CCL2.

B B lymphocytes

B lymphocytes is incorrect. B lymphocytes do not predominate, although they do have a role


(by subset), in either accelerating or retarding progression of atherosclerosis.

C Eosinophils

Eosinophils is incorrect. Although eosinophils do not predominate in plaques, they are


thought to have a role in permitting cell migration through the endothelium.

E T lymphocytes
T lymphocytes is incorrect. Similar to B cell subsets, Th1 cells are recognised to be pro-
atherogenic, whereas T regulatory cells are thought to be atheroprotective.
46277

Rate this question:

Next Question

Previous Question Tag Question

Feedback End Review

Difficulty: Average

Peer Responses %

Q. Answered Flagged

Q1

Q2

Q3

Q4

Q5

Q6

Q7

Q8

Q9

0:00:36/03:00:00

A 62-year-old woman with terminal breast cancer is admitted to the Emergency Department
with uncontrollable pain affecting her ribs, lower back and right hip. She is also tired all the
time and has reduced appetite with nausea. She is known to have extensive bony metastases
and currently takes regular paracetamol, naproxen and oral morphine every 4hrs. Her BP is
135/72 mmHg, pulse is 85/min and regular, she is unable to rest in the bed to allow you to
examine her because of pain.
Investigations:

Hb 101 g/l

WCC 8.9 x 10 9/l

PLT 122 x 10 9/l

Na 138 mool/l

K 4.5 mmol/l

Creatinine 132 micromol/l

Albumin 23 g/l

Ca 2.71 mmol/l

ALP 213 U/l

Glucose 6.1 mmol/l

Which of the following is the most appropriate intervention of pain control?

A Amitriptyline

B Cannabis resin

C Clonidine

D Dexamethasone

E Sertraline

Explanation 

D Dexamethasone
Glucocorticoids are prominent in end of life care not only for control of pain, but also to
relieve nausea and fatigue and improve general feelings of wellbeing. Randomised controlled
trials have suggested only approximately a 15% improvement in pain, although other benefits
including reduced nausea and improved energy are seen. Dexamethasone is the usual agent
of choice.

A Amitriptyline

Amitriptyline is incorrect. In this situation a tricyclic may significantly add to drowsiness, and
will not benefit appetite, hence dexamethasone is the preferred option.

B Cannabis resin

Cannabis resin is incorrect. Cannabinoids do stimulate appetite and may relieve nausea in
patients with metastatic carcinoma, although would normally be considered an add-on to
steroid therapy in the event that symptoms were not improved.

C Clonidine

Clonidine is incorrect. Clonidine is most effective in the treatment of neuropathic pain, and
can be given orally, trans-dermally or intra-spinally.

E Sertraline

Sertraline is incorrect. SSRIs have minimal effectiveness with respect to pain relief; other
options such as tricyclics are preferred for pain control.
46288

Rate this question:

Next Question

Previous Question Tag Question

Feedback End Review

Difficulty: Average

Peer Responses %
Q. Answered Flagged

Q1

Q2

Q3

Q4

Q5

Q6

Q7

Q8

Q9

0:00:36/03:00:00

A 29-year-old pregnant woman comes to the Emergency Department with a swollen, painful
left calf, which has enlarged significantly in diameter over the past 36hrs. She has no past
medical history of note, and is 18 weeks pregnant with her first child. On examination her BP
is 122/72 mmHg, pulse is 75/min and regular. Her BMI is 27. Routine blood screening apart
from an elevated d-dimer is unremarkable. Venous USS reveals a significant left DVT
extending to the proximal femoral vein.
Which of the following is the most appropriate intervention?

A Argatroban

B Enoxaparin

C Fondaparinux

D Rivaroxaban

E Warfarin

Explanation 

B Enoxaparin

Low molecular weight heparins are the default intervention of choice for patients with deep
vein thrombosis in pregnancy. Dosing of LMWH is based on early pregnancy weight, and is
given as a BD regimen.

A Argatroban

Argatroban is incorrect. Argatroban is a direct thrombin inhibitor, experience does exist with
respect to its use in pregnancy, although it is usually considered second line in patients who
are unable to take LMWHs.

C Fondaparinux

Fondaparinux is incorrect. Fondaparinux has extremely limited data in pregnancy. For this
reason its use is usually reserved for patients with HIT who are unable to take other options
such as danaparoid.

D Rivaroxaban

Rivaroxaban is incorrect. Oral factor Xa inhibitors are not recommended for use in pregnancy.
E Warfarin

Warfarin is incorrect. Although coumarins can be considered for use at the very end of
pregnancy, and outside weeks 6-12 when risk of carriage across the placenta is thought to be
highest, LMWH is preferred. Risk of foetal bleeding may be higher with warfarin vs LMWH at
any stage of pregnancy, and the risk of teratogenicity outside weeks 6-12 is unquantified.
46221

Rate this question:

Next Question

Previous Question Tag Question

Feedback End Review

Difficulty: Average

Peer Responses %

Q. Answered Flagged

Q1

Q2

Q3

Q4

Q5

Q6

Q7

Q8

Q9

0:00:36/03:00:00

A 75-year-old woman is admitted with decreased exercise tolerance, shortness of breath and
palpitations over the past week. She has a history of ischemic heart disease and COPD and
takes a number of medications including seretide, furosemide, aspirin, ramipril and
atorvastatin. Her BP is 115/80 mmHg, pulse is 95 bpm (atrial fibrillation). There is bilateral
pitting oedema of both ankles, and bilateral inspiratory crackles at the lung bases.
Investigations:

Hb 125 g/l

WCC 7.1 × 10 9/l

PLT 201 × 10 9/l

Na 138 mmol/l

K 4.9 mmol/l

Creatinine 110 µmol/l

ECG AF, previous inferior infarct

CXR Cardiomegaly, bilateral upper lobe diversion

Which of the following is the most appropriate intervention for rate control?

A Amiodarone

B Amlodipine

C Bisoprolol

D Digoxin

E Diltiazem

Explanation 

C Bisoprolol

In this situation rate control rather than rhythm control is the objective, bisoprolol and
digoxin are potential options. Bisoprolol is the preferred choice because it is associated with
reduced mortality when prescribed for patients with pre-existing cardiac failure. Low dose
bisoprolol will positively impact both on her ventricular rate and symptoms of heart failure.
A Amiodarone

Amiodarone is incorrect. Amiodarone is not appropriate because chemical cardioversion to


sinus rhythm has an extremely low probability of success.

B Amlodipine

Amlodipine is incorrect. Dihydropyridine calcium channel blockers are better agents for
hypertension, and won’t impact significantly on ventricular rate.

D Digoxin

Digoxin is incorrect. Although digoxin is efficient in slowing the ventricular rate, it is not the
preferred option here, as a meta-analysis of digoxin studies has raised the possibility of
increased mortality associated with its use.

E Diltiazem

Diltiazem is incorrect. Diltiazem is significantly negatively inotropic, although it does slow


ventricular rate. It is therefore typically a third line option in patients who do not tolerate beta
blockade or digoxin.
46247

Rate this question:

Next Question

Previous Question Tag Question

Feedback End Review

Difficulty: Average

Peer Responses %

Q. Answered Flagged

Q1
Q. Answered Flagged

Q2

Q3

Q4

Q5

Q6

Q7

Q8 
0:00:36/03:00:00

A 45-year-old woman who works in a supermarket presents to the Rheumatology Clinic for
review. She complains of pain over her right forearm, and loss of sensation over her thumb
and index finger affecting the hand. She is also finding it difficult to grip the scanner at work
that is used to price large items. There is no past medical history of note. General
examination is unremarkable. On examination of the right hand you can demonstrate
numbness over the palmar aspect affecting the thumb and index finger and over the lateral
aspect of the palm. There is weakness affecting the thenar eminence.
Which of the following is the most likely diagnosis?

A Carpal tunnel syndrome

B Golfer’s elbow

C Median nerve inflammation

D Pronator syndrome

E Tennis elbow

Explanation 

D Pronator syndrome

This patient’s symptoms are definitely in the distribution of the median nerve, although the
sensory symptoms suggest the lesion is proximal to the formation of the sensory branch. This
makes pronator syndrome the most likely diagnosis. Three causes of compression are
identified, either compression from the ligament of Struthers, the pronator teres muscle, or
by the bicipital aponeurosis. If this patient is, as suspected, lifting heavy items as the possible
cause of her condition, she should be offered alternative duties. In the event that this
patient’s symptoms fail to improve with an alternative work pattern, a surgical
decompression can be considered.

A Carpal tunnel syndrome

Carpal tunnel syndrome is incorrect. In carpal tunnel syndrome sensation over the thenar
eminence is spared.

B Golfer’s elbow

Golfer’s elbow is incorrect. Golfer’s elbow produces pain over the medial epicondyle.
C Median nerve inflammation

Median nerve inflammation is incorrect. We are given no past medical history to suggest
causes of mononeuritis, and a compression syndrome is more likely given her job in the
supermarket, related to lifting.

E Tennis elbow

Tennis elbow is incorrect. Tennis elbow results in pain over the lateral epicondyle.
49708

Rate this question:

Next Question

Previous Question Tag Question

Feedback End Review

Difficulty: Difficult

Peer Responses %

Q. Answered Flagged

Q1

Q2

Q3

Q4

Q5

Q6

Q7

Q8

Q9

0:00:36/03:00:00

A three-year-old boy is referred to the tertiary paediatric service with features of cardiac
failure. Investigations reveal a progressive cardiomyopathy. He does not respond to diuretics
or inotropes. Further work up exposes a deficit in intracellular fatty acid transport.
Which enzyme is essential for transporting free fatty acids within the cell for providing
energy?

A Arginine hydroxylase

B Carnitine acyltransferase I

C Cyclooxygenase

D Phosphoribosyl pyrophosphate synthetase

E Xanthine oxidase

Explanation 

B Carnitine acyltransferase I

Transport of fatty acyl-CoA into mitochondria is via an acyl-carnitine intermediate, itself


generated by the action of carnitine acyltransferase I, an enzyme that resides in the outer
mitochondrial membrane. Acyl-carnitine is then transported into the mitochondria where
carnitine acyltransferase II catalyses the regeneration of the fatty acyl-CoA molecule.
Carnitine deficiency leads to an inability to transport fatty acids into the mitochondria; the
most common manifestation of primary carnitine deficiency in children is dilated
cardiomyopathy; carnitine must be administered. Secondary carnitine deficiency, e.g. in the
context of haemodialysis, has a broad range of severity from occasional muscle cramping to
sudden death.

A Arginine hydroxylase

Arginine hydroxylase is incorrect. Arginine hydroxylase in an enzyme responsible for


modification of arginine - a basic amino acid. Deficiencies may play a role in remodelling
secondary to hypoxia in tissues.

C Cyclooxygenase

Cyclooxygenase is incorrect. Cyclooxygenase is responsible for prostaglandin synthesis.

D Phosphoribosyl pyrophosphate synthetase


Phosphoribosyl pyrophosphate synthetase is incorrect. Phosphoribosyl pyrophosphate
synthetase is concerned with uric acid production.

E Xanthine oxidase

Xanthine oxidase is incorrect. Xanthine oxidase generates reactive oxygen species.


49614

Rate this question:

Next Question

Previous Question Tag Question

Feedback End Review

Difficulty: Average

Peer Responses %

Q. Answered Flagged

Q1

Q2

Q3

Q4

Q5

Q6

Q7

Q8

Q9

0:00:36/03:00:00

An 18-year-old man from a traveller family presents to the Cardiology Clinic with increasing
shortness of breath, headaches and syncopal episodes. He tells you he has never been able to
exercise, and that his mother removed him from hospital follow up in the Cardiology
Department as a baby. Examination reveals a BP of 162/91 mmHg in the right arm, systolic BP
is 30mmHg lower in the left arm. There is a systolic murmur.
You suspect coarctation of the aorta. Which of the following other abnormalities is most
likely to be found?

A Bicuspid aortic valve

B Intracranial aneurysms

C Mitral stenosis

D Pulmonary stenosis

E Ventricular septal defect

Explanation 

A Bicuspid aortic valve

Case series suggest that in adults diagnosed with coarctation of the aorta, bicuspid aortic
valve is found to co-exist in up to 60%.

B Intracranial aneurysms

Intracranial aneurysms is incorrect. Intracranial haemorrhage is recognised as a more


common cause of death in patients with coarctation, although intracranial aneurysms are
thought to occur in only 10% of patients with the disorder.

C Mitral stenosis

Mitral stenosis is incorrect. Mitral valve abnormalities are rarer in conjunction with
coarctation, they are more likely to be seen in children presenting with the condition who
also have a VSD.

D Pulmonary stenosis

Pulmonary stenosis is incorrect. Pulmonary valve abnormalities are uncommon in patients


with coarctation.
E Ventricular septal defect

Ventricular septal defect is incorrect. Ventricular septal defects are seen in conjunction with
coarctation, but in one adult case series they were only seen 13% of the time.
46281

Rate this question:

Next Question

Previous Question Tag Question

Feedback End Review

Difficulty: Average

Peer Responses %

Q. Answered Flagged

Q1

Q2

Q3

Q4

Q5

Q6

Q7

Q8

Q9

0:00:36/03:00:00

A 37-year-old woman states she has special powers that allow her to know what other
people are thinking. She also believes she is related to the Royal family. She has been
overactive, elated in mood and not sleeping.
How would you describe her thoughts?

A Delusions of guilt

B Delusions of reference

C Persecutory delusions

D Nihilistic delusions

E Grandiose delusions

Explanation 

E Grandiose delusions

These are beliefs of exaggerated importance and are associated with mania. This patient has
the core symptoms of mania: elevation of mood, overactivity and grandiosity.

A Delusions of guilt

Delusions of guilt is incorrect. These are also often found in individuals with a depressive
illness, generally related to a minor error in their past that they believe will, or has caused,
disaster.

B Delusions of reference

Delusions of reference is incorrect. These are ideas that objects, events or people have a
personal significance for the patient.

C Persecutory delusions

Persecutory delusions is incorrect. These are a set of delusional conditions in which the
affected person believes they are being targeted. They have been defined as containing two
central elements: (1) the individual thinks that harm is occurring, or is going to occur; and (2)
the individual thinks that the perceived persecutor has the intention to cause harm.
Persecutory delusions are the most common form of delusions in paranoid schizophrenia.
D Nihilistic delusions

Nihilistic delusions is incorrect. These are beliefs about the non-existence of some person or
thing, and are associated with extremes of low mood.
1463

Rate this question:

Next Question

Previous Question Tag Question

Feedback End Review

Difficulty: Average

Peer Responses %

Q. Answered Flagged

Q1

Q2

Q3

Q4

Q5

Q6

Q7

Q8

Q9

0:00:36/03:00:00

A 67-year-old woman with a history of ischaemic heart disease is admitted to the Emergency
Department with gram negative sepsis and is extremely unwell with hypotension and
tachycardia. Given her previous cardiovascular history, the ST3 decides to insert a right
subclavian line to enable CVP measurement. You are fast bleeped to review her 45 m later
because nursing staff are concerned that her BP has deteriorated and she is markedly more
short of breath. On examination her BP is 85/50 mmHg, pulse is 105 bpm and regular. The
right side of the chest is dull to percussion and there is reduced movement on the right side
on inspiration. There are reduced breath sounds on the right side affecting the mid and lower
zones.
Which of the following is the most likely diagnosis?

A Haemothorax

B Myocardial infarction

C Pericardial effusion

D Pneumothorax

E Pulmonary embolism

Explanation 

A Haemothorax

The dullness to percussion, coupled with marked hypotension and tachycardia is suggestive
of venous leak leading to a significant haemothorax. Although venous puncture leading to
massive haemorrhage is rare, it is more likely to occur with subclavian vein cannulation versus
jugular venous cannulation. For this reason jugular vein lines are preferred.

B Myocardial infarction

Myocardial infarction is incorrect. This would not lead to the lateralising signs consistent with
blood accumulation.

C Pericardial effusion

Pericardial effusion is incorrect. Proximal perforation does not lead to pericardial effusion and
subsequent tamponade.

D Pneumothorax
Pneumothorax is incorrect. Pneumothorax is associated with hyper-resonance on percussion.

E Pulmonary embolism

Pulmonary embolism is incorrect. Pulmonary embolism is associated with hypotension and


tachycardia but not with the right sided chest signs seen here.
46218

Rate this question:

Next Question

Previous Question Tag Question

Feedback End Review

Difficulty: Average

Peer Responses %

Q. Answered Flagged

Q1

Q2

Q3

Q4

Q5

Q6

Q7

Q8

Q9

0:00:36/03:00:00

A 34-year-old man who is known to be HIV positive, but is poorly compliant with therapy, is
brought to the Emergency Department having suffered 3 seizures over the past 12hrs. His
partner was reluctant to call the ambulance but did so after the third seizure lasted for some
3-4 minutes. Apparently he has become increasingly confused over the past few days and
has complained of weakness and poor co-ordination affecting his right arm. On examination
he is pyrexial 37.9°C, BP is 110/70 mmHg, pulse is 85/min and regular. He is poorly compliant
with power testing, although it is obvious he has decreased movement affecting the right
hand side.
Investigations:

Hb 109 g/l

WCC 4.3 × 10 9/l

PLT 121 × 10 9/l

Na 132 mmol/l

K 4.5 mmol/l

Creatinine 90 µmol/l

CRP 58 mg/l

CT head Ring enhancing lesion in the left parietal lobe

Which of the following is most useful to differentiate toxoplasma from other diagnosis?

A Brain biopsy

B CSF PCR

C IgG toxoplasma antibody titre

D MRI

E PET scanning

Explanation 

B CSF PCR

The best way to rapidly confirm the suspected diagnosis of cerebral toxoplasmosis is with
CSF PCR. Detection rates are lower for ocular toxoplasmosis, and where there is isolated eye
disease, this is not the detection method of choice. Once diagnosis is confirmed,
pyrimethamine is the initial intervention of choice.

A Brain biopsy

Brain biopsy is incorrect. Although brain biopsy does achieve diagnosis of toxoplasmosis, it is
not an easily accessible investigation.

C IgG toxoplasma antibody titre

IgG toxoplasma antibody titre is incorrect. Given a high chance of previous environmental
exposure, antibody titres are poorly predictive of acute toxoplasma infection.

D MRI

MRI is incorrect. Although MRI is efficient at identifying the typical structural lesions
associated with toxoplasma infection, differentiation from other infections such as
Cryptococcus, or from lymphoma, is poor.

E PET scanning

PET scanning is incorrect. Although this is thought to differentiate between toxoplasma and
CNS lymphoma, it is not widely available.
46246

Rate this question:

Next Question

Previous Question Tag Question

Feedback End Review

Difficulty: Average

Peer Responses %

Q. Answered Flagged
Q. Answered Flagged

Q1

Q2

Q3

Q4

Q5

Q6

Q7

Q8
0:00:36/03:00:00

You are asked to see a 74-year-old woman who is trying to get out of bed and is very
agitated, calling out and seeing people at the end of her bed trying to take her away. She
lives independently in her own house and was admitted some 48hrs earlier for a routine left
total knee replacement, and is now some 16hrs post op. Past history of hypertension and
three urinary tract infections over the past year are noted. She is a non-smoker, she tells you
that she drinks a glass of brandy each evening. On examination she is apyrexial, her BP is
132/82 mmHg, pulse is 80 bpm and regular. She refuses to let you listen to her chest or
palpate her abdomen.
Investigations:

Hb 104 g/l

WCC 8.1 × 10 9/l

PLT 179 × 10 9/l

CRP 14 mg/l

Na 138 mmol/l

K 4.3 mmol/l

Creatinine 121 µmol/l

Urine blood +

Protein and nitrites Negative

Which of the following is the most likely diagnosis?

A Charles Bonnet syndrome

B Delirium

C Early Alzheimer’s disease

D Hypomania

E Multi-infarct dementia

Explanation 

B Delirium
Postoperative delirium is common in the elderly, especially after a major orthopaedic
procedure such as a total knee replacement. Confusion may have been exacerbated by
alcohol withdrawal, given that estimation of alcohol consumption is notoriously difficult. Its
optimal to manage this patient in a well lit room, keep to one nurse for personal care per shift
if possible, and give reassurance, rather than reaching for pharmacological intervention. In
the event that medication is required, both haloperidol and lorazepam are commonly used
agents.

A Charles Bonnet syndrome

Charles Bonnet syndrome is incorrect. This is a syndrome of visual hallucinations with


preserved consciousness, associated with severe visual impairment.

C Early Alzheimer’s disease

Early Alzheimer’s disease is incorrect. This woman lives independently, and there is no
indication of dementia pre-dating her admission to the hospital.

D Hypomania

Hypomania is incorrect. Hypomania is can present with agitation, delusions and


hallucinations, although there is no evidence of prior mania or depression here.

E Multi-infarct dementia

Multi-infarct dementia is incorrect. There appear to be no risk factors for vascular dementia,
and she has lived independently before her admission to the hospital.
46260

Rate this question:

Next Question

Previous Question Tag Question

Feedback End Review

Difficulty: Average

Peer Responses %
Q. Answered Flagged

Q1

Q2

Q3

Q4

Q5

Q6

Q7

Q8

Q9

0:00:36/03:00:00

A 19-year-old woman comes to the Allergy Clinic for review. She has suffered from allergic
rhinitis which is worst in April and May for many years, but has become very concerned as
over the past few months her mouth becomes sore and tingles when she eats raw apples.
She is however able to eat apples cooked in a pie. She has no past medical history of note
and her only regular medication is the progesterone only pill. Examination in the clinic is
entirely normal.
Which of the following is the most likely diagnosis?

A Birch pollen allergy

B C1 esterase deficiency

C Grass pollen allergy

D Latex allergy

E Ragweed allergy

Explanation 

A Birch pollen allergy

There is cross reactivity in patients who are allergic to birch pollens with proteins found in
raw apples. These are denatured by cooking and so the patient is able to eat apples that are
in a pie. In patients with birch apple allergy syndrome, tingling and soreness in the mouth
develops a short time after eating the raw fruit. Patients may also experience a reaction when
they eat peach, cherries and plums.

B C1 esterase deficiency

C1 esterase deficiency is incorrect. In this situation episodes of angioedema are not


precipitated by exposure to an allergen.

C Grass pollen allergy

Grass pollen allergy is incorrect. Patients with grass pollen allergy are more likely to develop
symptoms when they eat melons, oranges or tomatoes.

D Latex allergy
Latex allergy is incorrect. The latex fruit syndrome is most commonly seen in association with
avocados, bananas, and kiwi fruit.

E Ragweed allergy

Ragweed allergy is incorrect. Ragweed allergy patients develop food allergy symptoms after
exposure to banana and melon.
46243

Rate this question:

Next Question

Previous Question Tag Question

Feedback End Review

Difficulty: Average

Peer Responses %

Q. Answered Flagged

Q1

Q2

Q3

Q4

Q5

Q6

Q7

Q8

Q9

0:00:36/03:00:00

A 65-year-old woman, who lives alone, complains of increasing pain in her left knee. She finds
it difficult to climb stairs and is on maximal co-dydramol (paracetamol and dihydrocodeine)
and has been for the last six months. She stopped taking NSAIDs because of reflux
symptoms and did not receive any benefit from physiotherapy. Valgus deformity with
obvious instability is also noted. Her knee radiograph shows severe osteoarthritis with very
minimal joint space and no evidence of a loose body. Her body mass index (BMI) is 19.9
kg/m 2.
What would be the treatment of choice for this patient?

A Arthrodesis of the knee joint

B Arthroscopic washout

C Joint replacement

D Oral NSAIDs with gastric protection and further physiotherapy

E Weight loss and physiotherapy

Explanation 

C Joint replacement

Joint replacement would be an effective treatment for this woman. The primary indication for
joint replacement is relief of disabling pain caused by severe arthritis, usually present for six
months when medical and conservative measures have failed. Initial management of knee
osteoarthritis may include nonpharmaceutical treatment, including weight loss, aerobic
exercise, orthotics, and osteopathic manipulative treatment, or pharmaceutical treatment,
including nonsteroidal anti-inflammatory drugs (NSAIDs), corticosteroid injections. The
evidence for hyaluronic acid (HA) injections and glucosamine is limited.

A Arthrodesis of the knee joint

Arthrodesis of the knee joint is incorrect. Arthrodesis (fusing the joint) would limit mobility
further,

B Arthroscopic washout

Arthroscopic washout is incorrect. An arthroscopic washout is unlikely to confer much


benefit, given the absence of loose bodies on the radiograph.
D Oral NSAIDs with gastric protection and further physiotherapy

Oral NSAIDs with gastric protection and further physiotherapy is incorrect. NSAIDs, albeit
with gastroprotection, and further physiotherapy are unlikely to help significantly, given that
she has been on maximal co-dydramol therapy for 6 months, has considerable instability and
disabling pain.

E Weight loss and physiotherapy

Weight loss and physiotherapy is incorrect. This lady’s weight is appropriate for her height, as
her BMI is in the normal range (at the lower limit of normal). Body mass index = weight in
kg/(height in metres) 2. A normal body mass index (BMI) is between 20 kg/m 2 and 25 kg/m 2.
This woman’s weight is quite appropriate for her height, hence weight loss with
physiotherapy would be an inappropriate management of her condition.
48410

Rate this question:

Next Question

Previous Question Tag Question

Feedback End Review

Difficulty: Easy

Peer Responses %

Q. Answered Flagged

Q1

Q2

Q3

Q4

Q5

Q6
Q. Answered Flagged

Q7

Q8

0:00:36/03:00:00

A 29-year-old woman who has been suffering episodes of drowsiness, sweating and
tachycardia, that have increased over the past 3-4 months comes to the Emergency
Department having suffered a collapse at work. According to her workmate she responded
well to a few sips of coca-cola and a Mars Bar. She admits to increasing hunger over the past
6-9 months and a gradual increase in her weight. She takes no medication and has no
significant previous medical history. On examination her BP is 122/82 mmHg, pulse is 80 bpm
and regular. Her BMI is 29. Abdominal palpation is unremarkable.
Investigations:

Hb 12.1 g/dl

WCC 7.4 × 10 9/l

PLT 159 × 10 9/l

Na + 137 mmol/l

K+ 3.9 mmol/l

Cr 95 µmol/l

Glucose 6.1 mmol/l

Which of the following is the most appropriate next step?

A Exercise provocation test

B Insulin assay

C Supervised overnight fast

D Referral to the psychology service

E Urine screen for sulphonylureas

Explanation 

C Supervised overnight fast

The patients’ symptoms are suggestive but not diagnostic of an insulinoma. The episodes of
drowsiness, sweating and tachycardia, relieved by eating raise the possibility of intermittent
hypoglycaemia. Overnight fasting is often sufficient to precipitate an episode, and insulin,
glucose and c-peptide are used to confirm inappropriate hyperinsulinaemia. MRI is the usual
radiological investigation to localise any tumour.
A Exercise provocation test

Exercise provocation test is incorrect. This is considered where symptoms are not
precipitated by fasting and there is high suspicion of an underlying insulinoma.

B Insulin assay

Insulin assay is incorrect. In the absence of glucose and c-peptide measurement, insulin assay
is much less useful in confirming the diagnosis of insulinoma.

D Referral to the psychology service

Referral to the psychology service is incorrect. We are given no indication as yet that this
patient’s symptoms are due to self administration of insulin, although if insulin abuse is
suspected, the psychology service may be helpful.

E Urine screen for sulphonylureas

Urine screen for sulphonylureas is incorrect. Although sulphonylurea abuse is a possibility, in


the absence of previous psychiatric history, or access to SUs, insulinoma would be considered
first.
46220

Rate this question:

Next Question

Previous Question Tag Question

Feedback End Review

Difficulty: Average

Peer Responses %

Q. Answered Flagged

Q1
Q. Answered Flagged

Q2

Q3

Q4

Q5

Q6

Q7

Q8 
0:00:36/03:00:00

A 19-year-old man is admitted to the Emergency Department with a stab wound to the chest.
He is hypotensive and tachycardic with a BP of 80/60 mmHg and a pulse of 95 bpm. There is
a wound 3 cm lateral to the left lower sternal border.
Which of the following structures is most likely to have been injured?

A Coronary sinus

B Left atrium

C Left ventricle

D Right atrium

E Right ventricle

Explanation 

C Left ventricle

The ventricles are anterior to the atria, and given the position of the stab wound, in the left
lateral position, involvement of the left ventricle is most likely.

A Coronary sinus

Coronary sinus is incorrect. The coronary sinus is located in the posterior portion of the
coronary sulcus on the posterior surface of the heart.

B Left atrium

Left atrium is incorrect. The left atrium is the most posterior cardiac chamber.

D Right atrium

Right atrium is incorrect. The right atrium is the right most cardiac structure.

E Right ventricle

Right ventricle is incorrect. The right ventricle is anterior, and had the stab wound been just
lateral to the sternum, rather than 3cm away, right ventricle would have been the correct
answer.
46251
Rate this question:

Next Question

Previous Question Tag Question

Feedback End Review

Difficulty: Average

Peer Responses %

Q. Answered Flagged

Q1

Q2

Q3

Q4

Q5

Q6

Q7

Q8

Q9

0:00:36/03:00:00

A 23-year-old woman presents to the Emergency Department following a flight from


Australia. She complains of left calf tenderness, although the leg is not swollen, and has
minor pleuritic chest pain, accompanied by a dry cough. She takes no regular medication and
runs up to three miles per day. Her mother had a DVT during pregnancy. Examination reveals
a BP of 110/70 mmHg, pulse is 65 bpm and regular. General physical is unremarkable apart
from some minor calf tenderness. The diameters of both calves are the same. O 2 saturation
on air is 96%.
Which of the following is the most appropriate next step?

A CTPA

B D dimer

C Echocardiography

D Reassurance

E Venous Doppler

Explanation 

B D dimer

In this situation the Wells score is low, (i.e. in the less than or equal to 1 category). Guidelines
suggest in this scenario that if there is no obvious alternative diagnosis, D-dimer is indicated.
If this is then negative, (i.e. less than 450ng/ml), venous Doppler is not indicated and no
further investigations for DVT are required.

A CTPA

CTPA is incorrect. This involves a significant dose of radiation and should be reserved until
after further investigations, including D-dimer, CXR and venous Doppler have been
completed.

C Echocardiography

Echocardiography is incorrect. Although this may demonstrate right ventricular strain in the
presence of a large, acute PE, other investigations should be concluded before this is
considered.

D Reassurance
Reassurance is incorrect. This inappropriate until the results of a D-dimer blood test is known.

E Venous Doppler

Venous Doppler is incorrect. Venous Doppler is only indicated in this case where the D-dimer
is elevated.
46233

Rate this question:

Next Question

Previous Question Tag Question

Feedback End Review

Difficulty: Average

Peer Responses %

Q. Answered Flagged

Q1

Q2

Q3

Q4

Q5

Q6

Q7

Q8

Q9

0:00:36/03:00:00

A young man being evaluated for social withdrawal says that he has no friends, is not
interested in other people and does not feel lonely. A mental status examination reveals no
abnormality.
What is the most probable personality diagnosis?

A Avoidant personality disorder

B Narcissistic personality disorder

C Paranoid personality disorder

D Schizoid personality disorder

E Schizotypal personality disorder

Explanation 

D Schizoid personality disorder

The symptoms exhibited by this patient are characteristic of schizoid personality disorder.
Emotional aloofness, indifference to praise or criticism and the absence of bizarre or
idiosyncratic thinking are other features seen in this personality.

A Avoidant personality disorder

Avoidant personality disorder is incorrect. Avoidant personality disorder implies that the
individual has a need for human contact and feels lonely, with social anxiety limiting their
exposure to others, and not the case here.

B Narcissistic personality disorder

Narcissistic personality disorder is incorrect. Narcissistic individuals have an inflated level of


self-esteem, as well as having a high level of disdain for others.

C Paranoid personality disorder

Paranoid personality disorder is incorrect. Continuous, unfounded, pervasive and irrelevant


suspiciousness are features of paranoid personality disorder.

E Schizotypal personality disorder


Schizotypal personality disorder is incorrect. As well as being withdrawn and aloof, patients
with schizotypal personality disorder also display abnormalities of thought.
50350

Rate this question:

Next Question

Previous Question Tag Question

Feedback End Review

Difficulty: Difficult

Peer Responses %

Q. Answered Flagged

Q1

Q2

Q3

Q4

Q5

Q6

Q7

Q8

Q9

0:00:36/03:00:00

A 23-year-old woman who admits to habitual cocaine abuse is admitted to the Emergency
Department with dull lower abdominal pain, worse on the left hand side, and bloody
diarrhoea. She admits to using increasing amounts of cocaine to obtain the same high. Over
the past 24 hours her pain and diarrhoea has increased and she is unable to eat. Medication
of note includes the combined oral contraceptive pill, and she takes a salbutamol inhaler for
mild asthma. On examination her BP is 105/70 mmHg, pulse is 78 bpm and regular. Her
abdomen is tender, particularly on the left-hand side, with active bowel sounds. Rectal
examination reveals bloody diarrhoea.
Investigations:

Hb 121 g/l

WCC 10.9 × 10 9/l

PLT 201 × 10 9/l

Na + 138 mmol/l

K+ 3.6 mmol/l

Creatinine 122 µmol/l

Urea 10.2 mmol/l

CT abdomen Mucosal thickening of sigmoid colon and caecum

Flexible sigmoidoscopy Patchy colitis with areas of normal mucosa

Which of the following is the most appropriate course of action?

A IV antibiotics

B IV corticosteroids

C IV fluids

D Oral mesalazine

E Rectal corticosteroids

Explanation 

C IV fluids
This patient has ischaemic colitis as a result of chronic cocaine abuse. The pathological
process is progressive mesenteric artery disease as a result of arterial vasoconstriction and
fibrosis. Conservative management with IV fluids and analgesia is appropriate because
symptoms are usually related to non-occlusive disease.

A IV antibiotics

IV antibiotics is incorrect. This patient’s symptoms are not infective in origin, as such
antibiotics are not required.

B IV corticosteroids

IV corticosteroids is incorrect. Ischaemic colitis is not a primarily inflammatory process;


corticosteroids do not therefore have a role in management of the condition.

D Oral mesalazine

Oral mesalazine is incorrect. This is primarily a treatment for inflammatory bowel disease.

E Rectal corticosteroids

Rectal corticosteroids is incorrect. Rectal steroids are the treatment of choice for proctitis
related to ulcerative colitis.
46232

Rate this question:

Next Question

Previous Question Tag Question

Feedback End Review

Difficulty: Average

Peer Responses %

Q. Answered Flagged
Q. Answered Flagged

Q1

Q2

Q3

Q4

Q5

Q6

Q7

Q8 
0:00:36/03:00:00

A 62-year-old woman who has chronic back pain presents to the clinic with frequent tripping
due to weakness in her right lower limb. She also has a history of type 1 diabetes for which
she takes a basal bolus insulin regime. A most recent HbA1c is measured as 75 mmol/mol
(9%), and she has suffered episodes of mononeuritis previously as a result of the diabetes.
Which of the following would favour common peroneal nerve palsy rather than an L5 lesion?

A Loss of hip abduction

B Loss of plantar flexion

C Preserved ankle dorsiflexion

D Preserved foot inversion

E Preserved knee extension

Explanation 

D Preserved foot inversion

Preserved foot inversion and plantar flexion are both features of common peroneal nerve
palsy and can be used to differentiate the condition from L5 radiculopathy.

A Loss of hip abduction

Loss of hip abduction is incorrect. Loss of hip abduction is seen in conjunction with L3
radiculopathy, as is weakness of adductor longus.

B Loss of plantar flexion

Loss of plantar flexion is incorrect. Plantar flexion is spared in patients suffering from
common peroneal nerve palsy.

C Preserved ankle dorsiflexion

Preserved ankle dorsiflexion is incorrect. Ankle dorsiflexion is lost in patients suffering from a
common peroneal nerve palsy.

E Preserved knee extension


Preserved knee extension is incorrect. Knee extension is preserved in both L5 lesions and in
common peroneal nerve palsy, it is not therefore a differentiator.
46255

Rate this question:

Next Question

Previous Question Tag Question

Feedback End Review

Difficulty: Average

Peer Responses %

Q. Answered Flagged

Q1

Q2

Q3

Q4

Q5

Q6

Q7

Q8

Q9

0:00:36/03:00:00

You are asked to review a 32-year-old man who has returned to the ward after a spell on the
Intensive Care Unit following emergency surgery for a bleeding duodenal ulcer and a
subsequent line infection for which he received 10 days of vancomycin. He is known to have
non-Hodgkin’s lymphoma and has been undergoing chemotherapy over the past few weeks.
His main complaint now is of blurred vision. Fundoscopy reveals bilateral yellowish / white
retinal exudates and multiple haemorrhages.
Which of the following organisms is most likely to be responsible for the retinal changes
seen?

A Candida albicans

B Cytomegalovirus

C Pseudomonas aeruginosa

D Staphylococcus aureus

E Toxoplasma gondii

Explanation 

B Cytomegalovirus

Clinical CMV infection is more common in patients who are immunosuppressed, and the
mixed picture of exudates and retinal haemorrhages is typical of CMV associated retinitis.
CMV PCR is now widely available in the NHS for confirming the presence of virus, and
ganciclovir is the usual first line intervention.

A Candida albicans

Candida albicans is incorrect. Candida retinitis presents as several small creamy white retinal
lesions with overlying vitreous inflammation.

C Pseudomonas aeruginosa

Pseudomonas aeruginosa is incorrect. Pseudomonas is not associated with retinitis. It is


however recognised as a cause of bacterial conjunctivitis.

D Staphylococcus aureus
Staphylococcus aureus is incorrect. Staphylococcus, like pseudomonas is more likely to be
associated with bacterial conjunctivitis.

E Toxoplasma gondii

Toxoplasma gondii is incorrect. Toxoplasma choroidoretinitis typically affects only one eye
and is associated with areas of pale/white change affecting the retina, rather than areas of
frank haemorrhage.
46214

Rate this question:

Next Question

Previous Question Tag Question

Feedback End Review

Difficulty: Average

Peer Responses %

Q. Answered Flagged

Q1

Q2

Q3

Q4

Q5

Q6

Q7

Q8

Q9

0:00:36/03:00:00

A 64-year-old man with a history of hypertension and ischaemic heart disease is admitted to
the Emergency Department with a second episode of acute pancreatitis over the past 6
months. You ssupect that one of his cardiovascular medications may be responsible.
Which of the following is most likely to be associated with acute pancreatitis?

A Atorvastatin

B Bisoprolol

C Furosemide

D Lisinopril

E Spironolactone

Explanation 

C Furosemide

Although furosemide is not called out by case-control studies as being associated with acute
pancreatitis, at least 1 case report with positive re-challenge, excluding all other causes, such
as alcohol, hypertriglyceridemia, gallstones, and other drugs exist in the literature. This is
considered the strongest evidence for an agent to be implicated in pancreatitis.

A Atorvastatin

Atorvastatin is incorrect. Atorvastatin has class III evidence as a cause of pancreatitis. This
means on literature review, although case reports exist, there is no consistent exposure
response relationship described in the literature, and no information on re-challenge.

B Bisoprolol

Bisoprolol is incorrect. Cardioselective beta blockers are not implicated in increased risk of
pancreatitis.

D Lisinopril

Lisinopril is incorrect. Lisinopril, similar to atorvastatin is associated with case reports of


pancreatitis, although an exposure response relationship has not been established. This is in
contrast to enalapril, where the evidence is much stronger.
E Spironolactone

Spironolactone is incorrect. There is no evidence that potassium sparing diuretics drive an


increased risk of pancreatitis.
Badalov N, Baradarian R, Kadirawel I, et al. Drug-Induced Acute Pancreatitis: An Evidence-
Based Review. Clin Gastroenterol Hepatol 2007; 5:648.
46278

Rate this question:

Next Question

Previous Question Tag Question

Feedback End Review

Difficulty: Average

Peer Responses %

Q. Answered Flagged

Q1

Q2

Q3

Q4

Q5

Q6

Q7

Q8

Q9

0:00:36/03:00:00

A 49-year-old man comes to the clinic for review. He is taking carbamazepine slow release
for complex partial epilepsy.
Which of the following is the likely mechanism of action for carbamazepine?

A Voltage dependent calcium channel inhibition

B Voltage dependent chloride channel inhibition

C Voltage dependent magnesium channel inhibition

D Voltage dependent potassium channel inhibition

E Voltage dependent sodium channel inhibition

Explanation 

E Voltage dependent sodium channel inhibition

The exact MoA of carbamazepine has not been elucidated, although it is thought most likely
that prevention of repetitive firing of sodium-dependent action potentials in depolarised
neurons, via use- and voltage-dependent blockade of sodium channels may be its main
mechanism of action.

A Voltage dependent calcium channel inhibition

Voltage dependent calcium channel inhibition is incorrect. Ethosuximide is thought to act via
calcium channel inhibition.

B Voltage dependent chloride channel inhibition

Voltage dependent chloride channel inhibition is incorrect. GABA agonists used in the
treatment of epilepsy actually cause influx of chloride ions and are treatments for epilepsy.

C Voltage dependent magnesium channel inhibition

Voltage dependent magnesium channel inhibition is incorrect. Hypomagnesaemia is


recognised as a rare cause of epilepsy.

D Voltage dependent potassium channel inhibition


Voltage dependent potassium channel inhibition is incorrect. The potassium channel activator
drug (leading to potassium efflux) retigabine was proposed for the treatment of epilepsy
after success in preclinical trials, but has since been withdrawn due to toxicity profile.
46228

Rate this question:

Next Question

Previous Question Tag Question

Feedback End Review

Difficulty: Average

Peer Responses %

Q. Answered Flagged

Q1

Q2

Q3

Q4

Q5

Q6

Q7

Q8

Q9

 External Links

Pharmacodynamic properties
medicines.org.uk/emc/medicine/24201#PHARMACODYNAMIC_PROPS
(http://www.medicines.org.uk/emc/medicine/24201#PHARMACODYNAMIC_PROPS)
0:00:36/03:00:00

A 54-year-old woman presents to the diabetes clinic for review. She has had diabetes
mellitus for 8 years and has progressed from maximum oral treatment to insulin therapy. A
past history of gallstones is noted, and she failed to tolerate metformin due to chronic
diarrhoea. She complains of chronic diarrhoea, fatigue and breathlessness. Her routine blood
tests reveal mildly elevated transaminases and mild anaemia. An ultrasound scan of her liver
reveals what are thought to be a number of small metastases within the liver, and gallstones;
further imaging reveals one cystic lesion in the pancreatic head.
Which of the following statements best fits the condition that this patient has?

A Somatostatin, adrenocorticotropic hormone (ACTH) and calcitonin levels can all be


raised

B Contrast spiral computed tomography scanning is effective for demonstrating the


primary tumour in 90% of cases

C Curative surgery is possible in the majority of cases

D There is almost always an association with multiple endocrine neoplasia type 1


(MEN1)

E Diabetes mellitus is not related to this condition

Explanation 

A Somatostatin, adrenocorticotropic hormone (ACTH) and calcitonin levels can all be


raised

These tumours are often multi-secretory.

B Contrast spiral computed tomography scanning is effective for demonstrating the


primary tumour in 90% of cases

Contrast spiral computed tomography scanning is effective for demonstrating the primary
tumour in 90% of cases is incorrect as even with thin cuts (3–5 mm) through the pancreas,
this inexpensive and non-invasive modality has failed to identify up to 70% of pancreatic
endocrine tumours. MRI and endoscopic ultrasound (EUS) may be helpful, however.

C Curative surgery is possible in the majority of cases


Curative surgery is possible in the majority of cases is incorrect because the tumours are
slow-growing and symptoms are often present for several years before diagnosis – the
disease is often quite advanced by the time patients obtain appropriate medical attention.

D There is almost always an association with multiple endocrine neoplasia type 1


(MEN1)

There is almost always an association with multiple endocrine neoplasia type 1 (MEN1) is
incorrect as approximately 45% of people with this condition will be found to have MEN1.

E Diabetes mellitus is not related to this condition

Diabetes mellitus is not related to this condition is incorrect as nearly all such patients will
have impaired glucose tolerance or diabetes mellitus (95%).

Somatostatinoma

The history is suggestive of a somatostatinoma. The annual incidence of this rare tumour is
estimated to be 1 in 40 million. Somatostatinomas arise from the pancreas or the
gastrointestinal tract and are characterised by excessive secretion of somatostatin hormone
by tumour cells of D-cell origin. They are frequently associated with a classic clinical pentad
of diabetes mellitus, cholelithiasis, weight loss, steatorrhoea and diarrhoea, and
hypochlorhydria. Somatostatinomas very rarely arise in other locations (eg lungs, liver,
kidneys). Larger tumours are usually found in the pancreas (5 cm vs 2.5 cm in the
duodenum), although malignancy is of equal incidence for primary tumours of the pancreas
and duodenum (65%). Metastases are present in most patients at the time of clinical
presentation – most patients with somatostatinomas are symptomatic – because tumours are
slow-growing and symptoms are often present for several years before diagnosis, the disease
is often quite advanced by the time patients obtain appropriate medical attention.

Up to 10% of patients with neurofibromatosis type 1 (NF1; von Recklinghausen’s disease)


develop somatostatinomas. NF1-associated somatostatinomas are characteristically
duodenal, are rarely associated with somatostatinoma syndrome and are less likely to
metastasise as compared with sporadic somatostatinomas.
50258

Rate this question:

Next Question

Previous Question Tag Question

Feedback End Review

Difficulty: Difficult

Peer Responses %
Q. Answered Flagged

Q1

Q2

Q3

Q4

Q5

Q6

Q7

Q8

Q9

0:00:36/03:00:00

A 45-year-old woman with a history of rheumatoid arthritis presents to the Emergency


Department with an acutely red left eye. Current medication includes methotrexate and folic
acid, and her synovitis is currently under control. On examination the right eye is red, and she
tells you it feels gritty although is not painful. There is watering and mild photophobia with
engorged episcleral vessels extending radially. Visual acuity is normal.
Which of the following is the most likely diagnosis?

A Anterior uveitis

B Conjunctivitis

C Episcleritis

D Optic neuritis

E Posterior uveitis

Explanation 

C Episcleritis

The engorged episcleral vessels with a gritty feeling to the eye rather than frank pain,
coupled with watering, fit well with a diagnosis of episcleritis. Episcleritis is associated with a
number of systemic inflammatory conditions including rheumatoid arthritis. Episcleritis is
usually managed with NSAIDS +/- topical steroids.

A Anterior uveitis

Anterior uveitis is incorrect. Anterior uveitis is associated with eye pain, blood vision, and
inflammatory cells in the anterior chamber.

B Conjunctivitis

Conjunctivitis is incorrect. Conjunctivitis is usually associated with a mucoid / purulent


discharge, and is more likely to be bilateral.

D Optic neuritis

Optic neuritis is incorrect. Optic neuritis is associated with eye pain and visual impairment,
particularly with respect to colour vision.
E Posterior uveitis

Posterior uveitis is incorrect. Posterior uveitis is associated with inflammatory lesions


affecting the retina or choroid.
46219

Rate this question:

Next Question

Previous Question Tag Question

Feedback End Review

Difficulty: Average

Peer Responses %

Q. Answered Flagged

Q1

Q2

Q3

Q4

Q5

Q6

Q7

Q8

Q9

0:00:36/03:00:00

A 39-year-old man presents to the Dermatology Clinic for review of his psoriasis. His
symptoms have significantly worsened since he was started on medication for hypertension.
He has gone from using topical therapies only for scalp, elbow and knee psoriasis to more
generalised plaque disease. He tells you his GP tried 2-3 agents before finding one that
suited. You confirm extensive psoriasis on inspection.
Which of the following agents is most likely to be responsible?

A Amlodipine

B Bisoprolol

C Doxazosin

D Indapamide

E Valsartan

Explanation 

B Bisoprolol

Case series have suggested an increased risk of psoriasis in patients prescribed beta blockers
for the treatment of hypertension, and this was confirmed in an analysis of the nurses’ health
study. As such bisoprolol is the most appropriate answer here. The study demonstrated a
hazard ratio of 1.11 for worsening psoriasis in patients taking beta blockers versus other anti-
hypertensive medication.

A Amlodipine

Amlodipine is incorrect. No significant association between calcium channel blocker use and
psoriasis has been demonstrated.

C Doxazosin

Doxazosin is incorrect. Although small studies have raised the possibility of an increased risk
of psoriasis in patients taking alpha blockers, this has not been convincingly demonstrated.

D Indapamide

Indapamide is incorrect. There is no evidence that thiazide like diuretics increase the risk of
psoriasis.
E Valsartan

Valsartan is incorrect. Although ACE inhibitors may promote a small increase in psoriasis in
those who take them, the magnitude is less than that for beta blockers, and ARBs are not
associated with psoriasis risk.
46212

Rate this question:

Next Question

Previous Question Tag Question

Feedback End Review

Difficulty: Average

Peer Responses %

Q. Answered Flagged

Q1

Q2

Q3

Q4

Q5

Q6

Q7

Q8

Q9

 External Links
Hypertension, Antihypertensive Medication Use, and Risk of Psoriasis
archderm.jamanetwork.com/article.aspx?articleid=1884296
(http://archderm.jamanetwork.com/article.aspx?articleid=1884296)
0:00:36/03:00:00

A 42-year-old woman who is undergoing chemotherapy for breast cancer is reviewed on the
ward five days after admission from home with neutropaenic sepsis characterised by fevers,
cough and a sore throat. Despite five days of antibiotic therapy, including tazocin,
vancomycin and gentamicin. On examination she is pyrexial 38.5 °C, pulse is 88 bpm and
regular, there are scattered crackles and wheeze on auscultation of the chest.
Which of the following is the most appropriate next add in therapy?

A Amphotericin B

B Ciprofloxacin

C Meropenem

D Metronidazole

E Teicoplanin

Explanation 

A Amphotericin B

This patient’s chest symptoms, coupled with poor response to triple antibiotic therapy after
five days raises the possibility of fungal infection, (invasive aspergillosis). For this reason,
addition of amphotericin B is the most appropriate next step.

B Ciprofloxacin

Ciprofloxacin is incorrect. Ciprofloxacin is a first line option for neutropaenic sepsis in


patients with penicillin allergy.

C Meropenem

Meropenem is incorrect. Meropenem is a potential third line option / antibiotic change when
there is non-response after five days and risk of fungal infection is considered low.

D Metronidazole

Metronidazole is incorrect. Metronidazole is added where the suspicion of anaerobic infection


is high, (e.g. where there is mucositis, sinusitis, perineal or abdominal infection).
E Teicoplanin

Teicoplanin is incorrect. Teicoplanin is a potential first line option for patients already given
quinolone prophylaxis or where a line infection is suspected.
Typical guidance: (University hospital Bath)
46268

Rate this question:

Next Question

Previous Question Tag Question

Feedback End Review

Difficulty: Average

Peer Responses %

Q. Answered Flagged

Q1

Q2

Q3

Q4

Q5

Q6

Q7

Q8

Q9

 External Links
Neutropenic sepsis
ruh.nhs.uk/for_clinicians/departments_ruh/pathology/documents/haematology/gl_neutropenic_sepsis_14.6.pdf
(http://www.ruh.nhs.uk/for_clinicians/departments_ruh/pathology/documents/haematology
0:00:36/03:00:00

A 67-year-old man comes to the Cardiology Clinic for review, He is 8 months post prosthetic
aortic valve replacement and has noticed a significant deterioration in his exercise tolerance
over the past 2 months, with shortness of breath and bilateral ankle swelling. On examination
his BP is 145/80 mmHg, pulse is 85 bpm and regular. She has a systolic murmur loudest in the
aortic area. There are bilateral basal crackles on auscultation of the chest and you note
splinter haemorrhages on examination of the hands.
You suspect endocarditis.
Which of the following would be considered a good prognostic indicator for this patient?

A Abnormal creatinine

B Presence of cardiac failure

C PR prolongation on ECG

D Staphylococcus epidermidis on culture

E Streptococcus viridans on culture

Explanation 

E Streptococcus viridans on culture

Streptococcal valve infection is associated with improved outcomes compared to other


causative organisms in endocarditis such as staphylococci. This is potentially because of
lower levels of virulence and higher rates of bacterial clearance due to less antimicrobial
resistance compared to other organisms, and that S aureus is associated with greater
comorbidity at the time of initial infection.

A Abnormal creatinine

Abnormal creatinine is incorrect. Abnormal creatinine may indicate pre-existing renal disease
or vasculitis as a consequence of endocarditis, both of which have a negative impact on
morbidity and mortality.

B Presence of cardiac failure

Presence of cardiac failure is incorrect. Cardiac failure is associated with more extensive
infection and reduced survival in patients who progress to surgery.
C PR prolongation on ECG

PR prolongation on ECG is incorrect. This is potentially indicative of abscess formation,


necessitating progression to surgery.

D Staphylococcus epidermidis on culture

Staphylococcus epidermidis on culture is incorrect. S epidermidis is associated with more


extensive endocarditis compared to streptococcal infection.
46258

Rate this question:

Next Question

Previous Question Tag Question

Feedback End Review

Difficulty: Average

Peer Responses %

Q. Answered Flagged

Q1

Q2

Q3

Q4

Q5

Q6

Q7

Q8

Q9

0:00:36/03:00:00

You are asked to review a 25-year-old man in the Respiratory Clinic who presents with early
onset emphysema. He is continuing to experience worsening shortness of breath despite
treatment with high dose inhaled seretide and regular salbutamol nebulisers. You are
concerned that he may have alpha-1-antitrypsin deficiency.
Which of the following alpha-1-antitripsin genotypes is most associated with severe lung
disease?

A MM

B MS

C MZ

D SS

E ZZ

Explanation 

E ZZ

Alpha 1-antitrypsin deficiency is associated with autosomal co-dominant transmission with


patients carrying the ZZ genotype having only 10-15% of the normal level of alpha-1
antitrypsin. They are prone to both early onset emphysema and extra-hepatic manifestations
of alpha-1 antitrypsin deficiency such as liver disease.

A MM

MM is incorrect. Patients with the MM genotype carry two copies of the normal gene for
alpha-1 antitrypsin and are therefore not subject to accelerated emphysema.

B MS

MS is incorrect. Patients with the MS genotype have 80% of the normal level of alpha-1
antitrypsin. They do not have the same risk of emphysema as those with the ZZ genotype,
although in patients with the MS genotype there may be more rapid decline in lung function
associated with occupational dust exposure or smoking.

C MZ
MZ is incorrect. Patients with the MZ genotype have approximately 60% of the normal level
of alpha-1 antitrypsin. Similar to the MS genotype, some case series have suggested more
rapid progression of lung disease in those with exposure to other risk factors.

D SS

SS is incorrect. Patients with the SS genotype have approximately 60% of the normal level of
alpha-1 antitrypsin, they are subject to more rapid progression of lung disease where other
factors exist such as cigarette smoking.
46274

Rate this question:

Next Question

Previous Question Tag Question

Feedback End Review

Difficulty: Average

Peer Responses %

Q. Answered Flagged

Q1

Q2

Q3

Q4

Q5

Q6

Q7

Q8

Q9

0:00:36/03:00:00

A 29-year-old worker in a warehouse comes to the Rheumatology Clinic for review. He


complains of sensory loss affecting part of his right hand, and some weakness when it comes
to grasping heavy articles in the workplace. He notes that he is tall (190cm in height), and has
to lean on a low counter to sort /pick items for buyers. There is no past medical history of
note and he takes no regular medication. There is decreased sensation over the palmar
aspect of the 4th and 5th fingers and the ulnar aspect of the hand. You note minor weakness
of the intrinsic muscles of the hand.
What is the most likely underlying diagnosis?

A C8 lesion

B Early motor neurone disease

C Median nerve lesion

D Radial nerve lesion

E Ulnar nerve lesion

Explanation 

E Ulnar nerve lesion

An ulnar nerve lesion at the elbow is a definite possibility here, with sensory symptoms the
most prominent feature. Motor symptoms are less common, and can range from minor
weakness of the intrinsic muscles of the hand to severe wasting / clawing of the fingers. In
this case a simple change to the work routine may be all that is required to relieve symptoms.

A C8 lesion

C8 lesion is incorrect. This is unlikely as in C8 / T1 lesions sensory loss usually extends


proximally to the wrist because of involvement of the medial cutaneous nerve of the forearm.

B Early motor neurone disease

Early motor neurone disease is incorrect. The sensory loss effectively rules this out as the
underlying diagnosis.

C Median nerve lesion


Median nerve lesion is incorrect. Median nerve lesions at the elbow lead to weakness of
flexion of the index finger, defective thumb opposition, and sensory loss over the palm of the
hand, but over the thumb, index and middle fingers, rather than the 4th and 5th fingers.

D Radial nerve lesion

Radial nerve lesion is incorrect. These lead to sensory loss over the lateral aspect of the hand,
including the thumb
46239

Rate this question:

Next Question

Previous Question Tag Question

Feedback End Review

Difficulty: Average

Peer Responses %

Q. Answered Flagged

Q1

Q2

Q3

Q4

Q5

Q6

Q7

Q8

Q9

0:00:36/03:00:00

A 67-year-old man is admitted to the Emergency Department by paramedic ambulance after


suffering from increasing shortness of breath over the past 12 hours. He has been suffering
from increasing nausea and anorexia over the past week, and has a history of ischaemic heart
disease, chronic renal impairment and COPD. On examination his BP is 100/70 mmHg, pulse
is 92 bpm and regular. There are no murmurs and his chest appears clear. Abdomen is soft
and non-tender and there are no masses.
Investigations (arterial blood gas):

pH 7.36

pO2 12.3

pCO2 2.9

Na 135 mmol/l

K 5.5 mmol/l

Chloride 119 mmol/l

Bicarbonate 12 mmol/l

Urea 17.1 mmol/l

Glucose 13.1 mmol/l

Which of the following is the most likely diagnosis?

A Acute renal failure

B Cardiac failure

C Diabetic ketoacidosis

D Exacerbation of COPD

E Hyperosmolar non-ketotic state

Explanation 

A Acute renal failure

This patient has a hyperchloraemic metabolic acidosis, the most likely cause of which is acute
renal failure, which fits with the progressive nausea and anorexia which pre-dated the
shortness of breath. Although most blood gas analysers do not report creatinine, they do
report urea, which is elevated here.

B Cardiac failure

Cardiac failure is incorrect. Although cardiac failure can result in respiratory acidosis, it also
results in impaired gas exchange and CO2 retention, not seen here. Chest signs would also be
expected on auscultation.

C Diabetic ketoacidosis

Diabetic ketoacidosis is incorrect. This results in raised anion gap metabolic acidosis, not
seen here.

D Exacerbation of COPD

Exacerbation of chronic obstructive pulmonary disease (COPD) is incorrect. Chest signs


would be expected, as would impaired gas exchange, not seen here.

E Hyperosmolar non-ketotic state

Hyperosmolar non-ketotic state is incorrect. Hyperosmolar non-ketotic state is not associated


with acidosis, but is associated with dehydration and a significant rise in blood glucose not
seen here.
46284

Rate this question:

Next Question

Previous Question Tag Question

Feedback End Review

Difficulty: Average

Peer Responses %

Q. Answered Flagged
Q. Answered Flagged

Q1

Q2

Q3

Q4

Q5

Q6

Q7

Q8 
0:00:36/03:00:00

A 35-year-old man is attending the Gastroenterology Clinic for irritable bowel syndrome.
While in the clinic he mentions he is having problems attaining an adequate erection with his
new partner. Morning erections are still spontaneously occurring.
What would be the most appropriate first-line treatment?

A Sensate focus therapy

B Intraurethral prostaglandin inserted before intercourse

C Intracavernosal prostaglandin injections before intercourse

D Sildenafil

E Selective serotonin-reuptake inhibitors (SSRIs)

Explanation 

A Sensate focus therapy

Erectile dysfunction is the persistent or recurrent inability to attain or maintain an erection. It


is frequently associated with sexual anxiety, fear of failure and concerns about sexual
performance. Therefore, erectile dysfunction is often a temporary response to stress or loss
of confidence and it responds well to psychosexual treatment, especially if morning erections
occur.

Alternatively, it may also be caused by physical factors or by medications (including the


selective serotonin-reuptake inhibitors). For tobacco smokers, smoking cessation results in a
significant improvement.

The presence of morning erections make a physical cause much less likely than a
psychological cause. Given the above and the patient’s young age, sensate focus therapy is a
treatment that is worth pursuing.

B Intraurethral prostaglandin inserted before intercourse

Intraurethral prostaglandin inserted before intercourse is incorrect. Should the above therapy
fail to work, several other treatment options are available. Oral pharmacotherapy and vacuum
erection devices are first-line treatments followed by injections of drugs into the penis, and
penile implants. Intraurethral administration is a non-invasive way to deliver prostaglandin (a
vasodilating agent) to the affected area whilst minimising systemic absorption and related
side effects.

C Intracavernosal prostaglandin injections before intercourse


Intracavernosal prostaglandin injections before intercourse in incorrect. This intervention
offers higher efficacy compared with intraurethral administration but is also more invasive
and should be reserved for patients who have failed to respond to other treatments.

D Sildenafil

Sildenafil is incorrect. Sildenafil is a PDE5 inhibitor that is used in patients with erectile
dysfunction but also against pulmonary arterial hypertension. It has well-documented
efficacy and is recommended as a first-line pharmacological agent against erectile
dysfunction. However, given the systemic absorption of the drug it is associated with certain
side effects such as headaches, indigestion and skin flushing and is contraindicated in those
that take concomitant nitrates.

E Selective serotonin-reuptake inhibitors (SSRIs)

Selective serotonin-reuptake inhibitors (SSRIs) is incorrect. SSRIs are not used to treat
erectile dysfunction. On the contrary, they are known to lead to sexual dysfunction, which is
one of the main reasons for the drug to be discontinued.
51125

Rate this question:

Next Question

Previous Question Tag Question

Feedback End Review

Difficulty: Average

Peer Responses %

Q. Answered Flagged

Q1

Q2

Q3
Q. Answered Flagged

Q4

Q5

Q6

Q7

Q8 
0:00:36/03:00:00

The medical FY2 telephones the consultant to say she has diagnosed a patient as having
Behçet’s disease. The patient presented to the Emergency Department with reduction in
vision in one eye.
Which one of the following features supports her diagnosis?

A The patient is of Celtic extraction

B There is a strong family history of blindness

C The patient has a small-joint polyarthropathy

D The patient has active oral ulceration

E The patient’s visual loss is due to retinal artery occlusion

Explanation 

D The patient has active oral ulceration

Behçet’s disease is more common in people of Turkish origin, and is the combination of oral
and genital ulceration with uveitis. It is a vasculitis associated with large-joint arthropathy and
peripheral retinal venous occlusions. Other features include erythema nodosum and a
positive pathergy skin test (a sterile pustule appears 24–48 hours after skin puncture with a
needle) – and a predilection for medical exams! Either colchicine or thalidomide is used in
treatment.

A The patient is of Celtic extraction

The patient is of Celtic extraction is incorrect. Behçet’s disease is more common in people of
Turkish origin, not Celtic extraction.

B There is a strong family history of blindness

There is a strong family history of blindness is incorrect. The family history is only positive in
6%.

C The patient has a small-joint polyarthropathy

The patient has a small-joint polyarthropathy is incorrect. This does not support the diagnosis
of Behçet’s disease.
E The patient’s visual loss is due to retinal artery occlusion

The patient’s visual loss is due to retinal artery occlusion is incorrect. Although Behçet’s
disease can result in retinal artery occlusion, it is the presence of active oral ulceration that
most supports the diagnosis.
41826

Rate this question:

Next Question

Previous Question Tag Question

Feedback End Review

Difficulty: Easy

Peer Responses %

Q. Answered Flagged

Q1

Q2

Q3

Q4

Q5

Q6

Q7

Q8

Q9

0:00:36/03:00:00

A 52-year-old black woman comes to you for another opinion regarding a history of anaemia
that has been unresponsive to oral iron supplementation. She sought your opinion because
her other physician was recommending iron supplementation IV. She has been on nearly
continuous iron supplementation therapy ever since her second child was born 23 years ago.
Over the years she says her doctors have prescribed her to take anywhere from one to three
pills daily, sometimes with vitamin C concomitantly. Although she has never needed a
transfusion, she says she has been told that her red blood cell (RBC) count has never
completely normalised. She is otherwise healthy and has no unusual dietary habits. Her
menstrual history reveals relatively normal menstrual periods until about 3 years ago, when
she attained menopause. The patient believes that her mother was also iron deficient. Your
physical exam is normal. Laboratory values show a haemoglobin of 10.6 g/dl; haematocrit,
33%; MCV, 70 fl; normal white blood cell (WBC) with differential; normal platelet count; serum
iron, 70 µg/l (12–30); iron-binding capacity, 255 µg/dl (45–75); and ferritin, 158 µg/l (15–300).

Which is the most likely diagnosis?

A α-thalassaemia trait

B Acquired α-thalassaemia myelodysplastic syndrome

C ß-thalassaemia major

D Haemoglobin C disease

E Sickle-cell disease

Explanation 

A α-thalassaemia trait

This lady has a microcytic anaemia with a normal serum ferritin. It is interesting to note that
her blood count has never completely normalised despite many years of iron deficiency. We,
therefore, certainly know that this lady is not iron deficient. Alpha-thalassaemia trait has a
higher frequency in the Afro-Caribbean population.

B Acquired α-thalassaemia myelodysplastic syndrome

Acquired α-thalassaemia myelodysplastic syndrome is incorrect. This sounds like a


longstanding problem going back very many years. In addition there is nothing else to
suggest myelodysplasia, because there is a normal WBC count with a normal differential and
a normal platelet count.
C ß-thalassaemia major

ß-thalassaemia major is incorrect. Beta-thalassaemia major would cause anaemia present


from birth, requiring regular blood transfusions.

D Haemoglobin C disease

Haemoglobin C disease is incorrect. The haemoglobin levels range from about 8 g/dl up to a
normal concentration, but there is usually a marked microcytosis.

E Sickle-cell disease

Sickle-cell disease is incorrect. Sickle-cell disease would have come to light before now.
Although this certainly can cause anaemia, the haemoglobin level is usually much lower than
10.6 g/dl.
46299

Rate this question:

Next Question

Previous Question Tag Question

Feedback End Review

Difficulty: Average

Peer Responses %

Q. Answered Flagged

Q1

Q2

Q3

Q4

Q5
Q. Answered Flagged

Q6

Q7

Q8

0:00:36/03:00:00

A 20-year-old woman who is visiting the UK from Nepal presents to the Emergency
Department with a dry cough, night sweats and gradual weight loss over the past 4 months.
She has no past medical history of note and takes no regular medication. On examination her
BP is 105/70 mmHg, pulse is 75 bpm and regular. Her BMI is 21. There is an enlarged lymph
node in the right anterior triangle of the neck. Auscultation of the chest reveals scattered
wheeze, abdomen is soft and non tender.
Investigations:

Hb 104 g/l

WCC 9.9 × 10 9/l

PLT 159 × 10 9/l

Na 136 mmol/l

K 4.5 mmol/l

Creatinine 100 µmol/l

CRP 105 mg/l

Bilateral hilar lymphadenopathy, evidence of right upper lobe


CXR
consolidation/collapse

Interferon gamma release


Positive
assay (IGRA)

Which of the following is the most likely diagnosis?

A HIV

B Hodgkin’s lymphoma

C Kikuchi disease

D Sarcoidosis

E Tuberculosis

Explanation 

E Tuberculosis
This patient comes from an area with a high prevalence of tuberculosis, and the clinical
scenario, (gradual weight loss, night sweats, lymphadenopathy and chest signs), supports the
diagnosis. Although interferon gamma release assay is not recommended as part of
guidelines with respect to confirming the diagnosis, positivity certainly increases the
likelihood of underlying TB. Given the CXR findings, sputum sampling to look for AAFBs is
the initial investigation to confirm the diagnosis.

A HIV

HIV is incorrect. Although underlying HIV is a possibility, it would not account alone for the
chest signs and symptoms seen here.

B Hodgkin’s lymphoma

Hodgkin’s lymphoma is incorrect. The right sided upper lobe consolidation and positive IGRA
is more consistent with underlying TB.

C Kikuchi disease

Kikuchi disease is incorrect. Kikuchi disease is associated with necrotising lymphadenitis


rather than the consolidation seen here.

D Sarcoidosis

Sarcoidosis is incorrect. Sarcoidosis is more likely to be associated with hilar


lymphadenopathy alone versus the unilateral upper lone changes seen here, and positive
IGRA raises the possibility of TB rather than sarcoid as the alternative diagnosis.
46213

Rate this question:

Next Question

Previous Question Tag Question

Feedback End Review

Difficulty: Average

Peer Responses %
Q. Answered Flagged

Q1

Q2

Q3

Q4

Q5

Q6

Q7

Q8

Q9

0:00:36/03:00:00

A 72-year-old man suffers a cardiac arrest whilst waiting for an ambulance at the General
Practitioner’s office, having presented there with central crushing chest pain radiating down
both arms. An ECG conducted by the GP shortly before his death showed anterior ST
elevation of 3mm. His last recorded BP was 115/80 mmHg, with a pulse of 75/min.
Unfortunately, the GP did not have access to a defibrillator.
Which of the following is the most likely cause of death?

A Acute aortic regurgitation

B Complete heart block

C Septal rupture

D Ventricular fibrillation

E Ventricular wall rupture

Explanation 

D Ventricular fibrillation

Given the time course of this patient’s cardiac arrest in relation to the episode of chest pain,
arrhythmia is the most likely cause of death. This of course is the reason that access to
automated cardioverter / defibrillators. In the FAST-MI registry, 80% of VF events occurred
during the early period after infarct, and approximately 40% of them were associated with an
anterior infarct.

A Acute aortic regurgitation

Acute aortic regurgitation is incorrect. Acute AR is most commonly seen in association with
endocarditis, it is mitral regurgitation which is seen in conjunction with MI, usually due to
papillary muscle rupture.

B Complete heart block

Complete heart block is incorrect. Complete heart block is more likely to present with
collapse and severe hypotension rather than cardiac arrest.

C Septal rupture
Septal rupture is incorrect. This presents with hypotension and tachycardia, and occurs most
often some 48-72hrs after infarct.

E Ventricular wall rupture

Ventricular wall rupture is incorrect. This also presents with hypotension and tachycardia, and
is seen most often a few days after infarct.
46256

Rate this question:

Next Question

Previous Question Tag Question

Feedback End Review

Difficulty: Average

Peer Responses %

Q. Answered Flagged

Q1

Q2

Q3

Q4

Q5

Q6

Q7

Q8

Q9

0:00:36/03:00:00

A 45-year-old man presents to the Emergency Department with central chest pain radiating
to his back. He says that over the past 3-4 months he has noticed intermittent chest and back
pain that has slowly been getting worse. He has recently moved to the UK from Jamaica for
work, but according to his partner he seems depressed and intermittently confused, and has
struggled to even get out of the house on most days. There is also a history of arthritis for
which he takes regular ibuprofen. On examination his BP is 145/82 mmHg, pulse is 82/min
and regular. Chest is clear and abdominal palpation appears normal. You notice a nodular
skin rash, and neurological exam reveals loss of fine touch and proprioception affecting both
lower limbs.
Investigations:

Hb 119 g/l

WCC 7.2 × 10 9 /l

PLT 156 × 10 9 /l

Na + 138 mmol/l

K+ 4.9 mmol/l

Creatinine 100 µmol/l

CRP 18 mg/l

CT angiogram Evidence of thoracic aortic aneurysm

Which of the following is the most likely cause of his thoracic aneurysm?

A Lyme disease

B Salmonellosis

C Syphilis

D Weil’s disease

E Yaws

Explanation 

C Syphilis
There are clues to the diagnosis of syphilis here, including the West Indian origin, nodular
rash, chronic confusion, and dorsal column sensory loss, consistent with tertiary disease.
Serologic testing is the obvious next step, with penicillin the treatment of choice.

A Lyme disease

Lyme disease is incorrect. Myocarditis and pericarditis are both reported in late Lyme disease
and are more common than aneurysms related to Lyme disease. Where these occur, they are
more likely to be seen affecting smaller arteries.

B Salmonellosis

Salmonellosis is incorrect. Salmonella can result in aortitis, although the time course is
considerably less than this man’s symptoms.

D Weil’s disease

Weil’s disease is incorrect. Moya Moya disease, thought to related to leptospirosis is a cause
of cerebral arterial inflammation.

E Yaws

Yaws is incorrect. Tertiary yaws is a rarer condition than tertiary syphilis and is more usually
associated with osteoperiostitis, subcutaneous nodules, and ulceration of the palate and
nasopharynx.
46224

Rate this question:

Next Question

Previous Question Tag Question

Feedback End Review

Difficulty: Average

Peer Responses %
Q. Answered Flagged

Q1

Q2

Q3

Q4

Q5

Q6

Q7

Q8

Q9 
0:00:36/03:00:00

A 21-year-old Ghanaian student comes to the clinic for review. He has a number of lesions on
his trunk which he feels are unsightly, and he wants to know the underlying diagnosis. Apart
from the skin lesions he is otherwise well and takes no regular medication. You identify 3-4
hypopigmented macular lesions over the trunk, with a slightly raised erythematous edge, and
decreased sensation within the central portion of the lesions. Routine bloods are
unremarkable apart from a small elevation in CRP to 35 mg/l.
Which of the following is the most likely diagnosis?

A Lichen planus

B Pityriasis versicolor

C Psoriasis

D Ringworm

E Tuberculoid leprosy

Explanation 

E Tuberculoid leprosy

The isolated macular lesions with hypopigmentation and loss of sensation are most typical of
tuberculoid leprosy. Routine laboratory blood testing is only rarely abnormal, and skin biopsy
from the edge of a lesion is the optimal way to confirm the diagnosis.

A Lichen planus

Lichen planus is incorrect. Lichen planus is an itchy rash which is most common on
extremities, and tends to be more common in spring and summer where sunlight may
precipitate appearance of the rash.

B Pityriasis versicolor

Pityriasis versicolor is incorrect. This is associated with multiple macular depigmented lesions,
although a loss of sensation is not seen.

C Psoriasis

Psoriasis is incorrect. This is a scaling rash most likely to be seen on extensor surfaces, and is
itchy rather than associated with decreased sensation.
D Ringworm

Ringworm is incorrect. Ringworm is an itchy rash associated with scaling and redness, rather
than the depigmentation and loss of sensation seen here.
46234

Rate this question:

Next Question

Previous Question Tag Question

Feedback End Review

Difficulty: Average

Peer Responses %

Q. Answered Flagged

Q1

Q2

Q3

Q4

Q5

Q6

Q7

Q8

Q9

0:00:36/03:00:00

A 31-year-old woman who is treated with ciclosporin for a flare of Crohn’s disease comes to
the Gastroenterology Clinic for review. She is concerned about a recent news article linking
immune modulating drugs to risk of cancer and wants to know how ciclosporin works.
Production of which cytokine is primarily reduced by ciclosporin therapy?

A IL-1

B IL-2

C IL-6

D IL-10

E IL-13

Explanation 

B IL-2

Ciclosporin inhibits the development of cell-mediated reactions, including allograft immunity,


delayed cutaneous hypersensitivity, experimental allergic encephalomyelitis, Freund's
adjuvant arthritis, graft-versus-host disease (GVHD), and also T-cell dependent antibody
production. At the cellular level it inhibits production and release of lymphokines including
predominantly interleukin 2, also known as T-cell growth factor, TCGF.

A IL-1

IL1 is incorrect. IL-1 antagonists include canakinumab, used for a number of inflammatory
conditions including severe gouty arthritis.

C IL-6

IL6, is incorrect. IL-6 antagonists such as tocilizumab are used in the treatment of rheumatoid
arthritis.

D IL-10

IL10 is incorrect. IL-10 is produced by a range of immune cells including T regulatory cells,
and it is thought to be associated with downregulation of immune response.
E IL-13

IL13 is incorrect. IL-13 is a pro-inflammatory cytokine involved in the pathogenesis of severe


asthma.
46215

Rate this question:

Next Question

Previous Question Tag Question

Feedback End Review

Difficulty: Average

Peer Responses %

Q. Answered Flagged

Q1

Q2

Q3

Q4

Q5

Q6

Q7

Q8

Q9

 External Links

Pharmacodynamic properties
medicines.org.uk/EMC/medicine/1307/SPC/Neoral+Soft+Gelatin+Capsules,+Neoral+Oral+Solution/#PHARMACODYN…
(https://www.medicines.org.uk/EMC/medicine/1307/SPC/Neoral+Soft+Gelatin+Capsules,+Ne
0:00:36/03:00:00

A 22-year-old nurse is referred to the Gastroenterology Clinic because of failure to mount a


surface antibody response to hepatitis B vaccination. She has a past history of asthma for
which she takes a seretide inhaler, and coeliac disease for which she follows a gluten free
diet. Clinical examination is unremarkable, and her BMI is normal at 23. Routine bloods apart
from the low Hep B surface antigen antibody titre are normal.
Which of the following is the most likely cause of Hep B vaccination failure?

A HLA type

B Inhaled corticosteroids

C Injection failure

D Non-compliance with coeliac diet

E Previous exposure to hepatitis B

Explanation 

A HLA type

Increased rates of vaccine failure are seen in patients with coeliac disease, and the reason for
this is thought to be that HLA types that predispose to vaccine failure also predispose to
coeliac. Some HLA haplotypes have been identified as marking out increased risk of non
response including B8, SC01, DR3, and DQB1*0604; DQA1*0102DRB1*1302. Response can be
improved by administration of a second course of therapy, and changing the injection site to
an intradermal one for the second course.

B Inhaled corticosteroids

Inhaled corticosteroids is incorrect. Systemic immunosuppression related to a standard dose


of fluticasone would be considered minimal in the context of vaccination.

C Injection failure

Injection failure is incorrect. Injection failure is seen when an in-experienced vaccinator injects
into an inappropriate space, for example an intra-gluteal injection.

D Non-compliance with coeliac diet


Non-compliance with coeliac diet is incorrect. Given her weight is normal and she complains
of no symptoms of coeliac, non-compliance with diet is unlikely.

E Previous exposure to hepatitis B

Previous exposure to hepatitis B is incorrect. Previous exposure to hepatitis B is associated


with vaccine failure, although we are given no evidence for that here.
46227

Rate this question:

Next Question

Previous Question Tag Question

Feedback End Review

Difficulty: Average

Peer Responses %

Q. Answered Flagged

Q1

Q2

Q3

Q4

Q5

Q6

Q7

Q8

Q9

0:00:36/03:00:00

A 17-year-old attends the Epilepsy Clinic for review. He suffered a generalised tonic clonic
seizure which lasted in total for approximately two minutes, whilst waiting to go to college
one morning. There was no incontinence but he did have tongue biting. He is studying for
four A-levels and hopes to go to University to become a doctor. He admits to clumsiness in
the mornings for the past two years or so when he tells you he suffers from uncontrollable
jerks affecting his arms and legs. He admits to anxiety about his exams, but has coped with
this by utilising the school counsellor. Neurological examination in the clinic is unremarkable.
Which of the following is the most likely diagnosis?

A Idiopathic generalised tonic clonic seizure

B Juvenile absence epilepsy

C Juvenile myoclonic epilepsy

D Psychogenic non-epileptic seizure

E Progressive myoclonic epilepsy

Explanation 

C Juvenile myoclonic epilepsy

This is a typical presentation of JME, a form of genetic (formerly called “idiopathic”)


generalised epilepsy. Most patients with JME suffer from myoclonic jerks which may be
described as morning clumsiness, for some time before they present to the clinic, usually
after a generalised tonic clonic seizure. The classical EEG pattern between fits in JME is 4 to
6 Hz bilateral polyspike and slow wave discharges with frontal predominance over a normal
background activity. Usual options for therapy include valproate, lamotrigine, topiramate and
levetiracetam.

A Idiopathic generalised tonic clonic seizure

Idiopathic generalised tonic clonic seizure (GTCS) is incorrect. The pattern of myoclonic jerks
is more consistent with JME than with idiopathic tonic clonic seizures.

B Juvenile absence epilepsy

Juvenile absence epilepsy (JAE) is incorrect. JAE has an earlier peak age of onset, (10-12
years of age), and is associated with more pronounced absence seizures than those seen in
JME.
D Psychogenic non-epileptic seizure

Psychogenic non-epileptic seizure (PNES) is incorrect. The myoclonic jerks followed by a


generalised tonic clonic seizure with tongue biting are classical for JME, rather than a non-
epileptic attack.

E Progressive myoclonic epilepsy

Progressive myoclonic epilepsy (PME) is incorrect. PME is associated with progressively more
severe seizures, and significant cognitive impairment, presenting at a younger age than that
seen here.
46282

Rate this question:

Next Question

Previous Question Tag Question

Feedback End Review

Difficulty: Average

Peer Responses %

Q. Answered Flagged

Q1

Q2

Q3

Q4

Q5

Q6

Q7
Q. Answered Flagged

Q8
0:00:36/03:00:00

A 42-year-old woman is referred to the Endocrinology Clinic by the Emergency Department


having been involved in a road traffic accident where a car side swiped her from the right
hand side. On examination in the Emergency Department she was found to have a bitemporal
hemianopia. She has no past medical history, although her periods have stopped some 4
months earlier with associated lack of libido, and her GP has told her this is because she is
entering the menopause. There is no medication history. On examination her BP is 132/89
mmHg, pulse is 75 bpm and regular. Heart sounds are normal and the chest is clear.
Abdomen is soft and non-tender with a BMI of 25. You confirm the bitemporal hemianopia.
There are no breast masses although the patient can express milk on minimal nipple
stimulation.
Investigations:

Hb 123 g/l

WCC 8.9 × 10 9/l

PLT 201 × 10 9/l

Na 137 mmol/l

K 4.5 mmol/l

Creatinine 85 µmol/l

Glucose 5.6 mmol/l

Prolactin 9200 U/l

MRI pituitary Macroadenoma abutting the optic chiasm

Which of the following is the most appropriate intervention?

A Bromocriptine

B L-dopa

C Octreotide

D Pegvisomant

E Somatostatin

Explanation 

A Bromocriptine
Although previously, early progression to surgery early would be recommended, modern
management recommends a trial of dopamine agonist therapy even in patients with evidence
of optic chiasm compression. As such bromocriptine a dopamine agonist is recommended.
Alternatives to bromocriptine include more modern dopamine agonists such as cabergoline
or ropinirole. Even if progression to surgery is required, dopamine agonists can drive tumour
shrinkage and improved outcomes.

B L-dopa

L-dopa is incorrect. L-dopa is not used as a treatment for prolactinoma, Historically the
prolactin response to L-dopa was used in the evaluation of prolactinoma, (up to the late 80s).

C Octreotide

Octreotide is incorrect. Octreotide is a somatostatin analogue which has a role in the


treatment of acromegaly rather than hyperprolactinaemia.

D Pegvisomant

Pegvisomant is incorrect. Pegvisomant binds to the growth hormone receptor, preventing


the binding of endogenous growth hormone, it is used in the treatment of acromegaly.

E Somatostatin

Somatostatin is incorrect. Long acting analogues of somatostatin are used in the treatment
of acromegaly, not prolactinoma.
46276

Rate this question:

Next Question

Previous Question Tag Question

Feedback End Review

Difficulty: Average

Peer Responses %
Q. Answered Flagged

Q1

Q2

Q3

Q4

Q5

Q6

Q7

Q8

Q9

0:00:36/03:00:00

A 27-year-old man presents with upper-limb pain that is exacerbated by coughing or


exercise. He has also noticed a weakness in his lower limbs and difficulty in walking. On
examination, his fingernails are dystrophic and there is loss of upper limb reflexes.
What is the most likely diagnosis?

A Adult polymyositis

B Ankylosing spondylitis

C Psoriatic arthropathy

D Syringomyelia

E Thoracic disc prolapse

Explanation 

D Syringomyelia

The clinical features in this scenario are suggestive of syringomyelia, which is the
development of a fluid-filled cavity or syrinx within the spinal cord. This produces a
combination of long tract signs, notably lower limb spasticity and extensor plantar responses,
in association with signs at the level of the syrinx, characteristically dissociated sensory loss
and lower motor neurone signs such as clawing of the hand due to extension to anterior horn
cells (‘upper motor neurone signs in the lower limbs and lower motor neurone signs in the
upper limbs’).

A Adult polymyositis

Adult polymyositis is incorrect. This patient does not have any features of adult polymyositis,
which is much more common in women (3:1) and usually presents with malaise, weight loss
and fever. Proximal muscle weakness affecting shoulder and pelvic girdle muscles is
progressive. Pain and tenderness are uncommon.

B Ankylosing spondylitis

Ankylosing spondylitis is incorrect. Although ankylosing spondylitis usually occurs in young


males, it classically causes low back pain and stiffness, worse in the morning and relieved by
exercise. Uveitis and costochondritis may accompany this condition, but upper limb
involvement with weakness and loss of reflexes is not seen.
C Psoriatic arthropathy

Psoriatic arthropathy is incorrect. Although nail dystrophy may occur in psoriasis, this patient
does not have ‘arthritis’ – swollen joints. There is a type of psoriatic arthritis that causes back
pain (axial type of psoriatic arthritis), but this resembles ankylosing spondylitis – see below.
Limb reflexes are not affected and weakness of limbs does not occur.

E Thoracic disc prolapse

Thoracic disc prolapse is incorrect. Thoracic disc prolapse is uncommon in young adult males
and would not cause weakness in the lower limbs.
48834

Rate this question:

Next Question

Previous Question Tag Question

Feedback End Review

Difficulty: Average

Peer Responses %

Q. Answered Flagged

Q1

Q2

Q3

Q4

Q5

Q6

Q7
Q. Answered Flagged

Q8
0:00:36/03:00:00

A 45-year-old man presents to the Rheumatology Clinic for review. His GP is concerned as to
whether he may have an underyling vasculitis as he has recently been diagnosed with an
isolated left third nerve palsy, and has a history of hypertension and mesenteric angina. There
was also apparently an episode of common peroneal nerve palsy 6 months ago. He has not
previously wanted a hospital referral, but the most recent event has prevented him from
driving. Medication includes ramipril 10mg, amlodipine 5mg and atorvastatin. On examination
his BP is 155/85 mmHg, pulse is 77/min and regular. You confirm the third nerve palsy. His
chest is clear and abdomen is soft and non tender. His BMI is 24. There is livedo reticularis
affecting his forearms and lower limbs.
Investigations:

Hb 120 g/l

WCC 10.9 × 10 9/l

PLT 203 × 10 9/l

Na + 139 mmol/l

K+ 5.1 mmol/l

Creatinine 142 micromol/l

ESR 70 mm/1st hour

Urine Blood+, Protein+

pANCA Negative

cANCA Negative

Anti-nuclear antibody Negative

CXR Unremarkable

Which of the following is the most likely diagnosis?

A Polyarteritis nodosa

B Polymyalgia rheumatica

C SLE

D Temporal arteritis

E Wegener’s granulomatosis
Explanation 

A Polyarteritis nodosa

The prominent symptoms of mesenteric angina, coupled with mononeuritis multiplex and
evidence of renal vasculitis and negative pANCA is suggestive of a diagnosis of polyarteritis
nodosa. Small artery biopsy demonstrates necrotising inflammation, although where possible
skin biopsy is preferred for diagnostic purposes. Corticosteroids are the mainstay of therapy.

B Polymyalgia rheumatica

Polymyalgia rheumatica is incorrect. PMR most commonly presents in the over 50s age group
with bilateral shoulder or pelvic girdle aching, morning stiffness and evidence of an acute
phase response.

C SLE

SLE is incorrect. Although lupus can present with features of mononeuritis and renal
vasculitis as here, mesenteric angina is unusual in SLE, and negative anti-nuclear antibody
counts against the diagnosis.

D Temporal arteritis

Temporal arteritis is incorrect. Temporal arteritis presents in the elderly and is associated with
headaches and temporal artery tenderness.

E Wegener’s granulomatosis

Wegener’s granulomatosis is incorrect. Wegener’s patients are usually c-ANCA positive, the
negative antibody results therefore count against Wegener’s as the underlying diagnosis.
46273

Rate this question:

Next Question

Previous Question Tag Question

Feedback End Review

Difficulty: Average

Peer Responses %
Q. Answered Flagged

Q1

Q2

Q3

Q4

Q5

Q6

Q7

Q8

Q9

0:00:36/03:00:00

A 32-year-old man comes to the clinic for review. He suffered extensive lacerations to his
forearms after falling through a window, and is concerned that he has residual weakness
affecting his right hand. On examination he is unable to touch the tip of his thumb to his
other fingers because of weakness of flexion at the first carpometacarpal joint.
Which of the following muscles is weak?

A Abductor pollicis brevis

B Abductor pollicis longus

C Adductor pollicis

D Flexor pollicis brevis

E Opponens pollicis

Explanation 

E Opponens pollicis

Opponens pollicis originates from the flexor retinaculum of the hand and the tubercle of the
trapezium. It passes downward and laterally, and is inserted into the metacarpal bone of the
thumb on its radial side. It is supplied by the recurrent branch of the median nerve, which is
liable to damage because of forearm injury.

A Abductor pollicis brevis

Abductor pollicis brevis is incorrect. Abductor pollicis brevis is responsible for thumb
abduction, movement of the thumb anteriorly, perpendicular to the palm.

B Abductor pollicis longus

Abductor pollicis longus is incorrect. It is responsible for thumb abduction at the wrist and is
supplied by the radial nerve after it passes through the supinator.

C Adductor pollicis

Adductor pollicis is incorrect. Adductor pollicis is responsible for movement of the thumb
towards the palm of the hand, and is injured during skiing falls due to incorrect handling of
ski poles.
D Flexor pollicis brevis

Flexor pollicis brevis is incorrect. Flexor pollicis brevis is responsible for thumb flexion, rather
than opposition.
46283

Rate this question:

Next Question

Previous Question Tag Question

Feedback End Review

Difficulty: Average

Peer Responses %

Q. Answered Flagged

Q1

Q2

Q3

Q4

Q5

Q6

Q7

Q8

Q9

0:00:36/03:00:00

A 34-year-old man with a history of Type 1 diabetes is found in a state of collapse in his
house, having not been seen over the past 48hrs. On examination in the Emergency
Department he is drowsy and tachypnoeic. His BP is 100/70 mmHg and he looks dehydrated.
Auscultation of the chest reveals mild wheeze. There is mild tenderness on palpation of the
abdomen.
Investigations:

Hb 141 g/l

WCC 13.2 × 10 9/l

PLT 334 × 10 9/l

Na + 144 mmol/l

K+ 5.9 mmol/l

Chloride 115 mmol/l

Urea 16.2 mmol/l

Creatinine 156 µmol/l

Glucose 34.1 mmol/l

pH 7.31

Bicarbonate 12 mmol/l

CO 2 3.3 kPa

O2 13.1 kPa

Which of the following is the most likely diagnosis?

A Carbon monoxide poisoning

B Diabetic ketoacidosis

C Hyperosmolar non-ketotic state

D Ischaemic colitis

E Pulmonary embolus

Explanation 
B Diabetic ketoacidosis

There are many pointers to diabetic ketoacidosis as the underlying diagnosis here including
the diagnosis of Type 1 diabetes, raised anion gap metabolic acidosis with hyperglycaemia,
and elevated white count suggesting an infection precipitating the event. IV access and rapid
fluid replacement are of course the most important initial interventions.

A Carbon monoxide poisoning

Carbon monoxide poisoning is incorrect. Although carbon monoxide produces a metabolic


acidosis because of tissue hypoxia, the hyperglycaemia and raised anion gap are not
consistent with carbon monoxide poisoning.

C Hyperosmolar non-ketotic state

Hyperosmolar non-ketotic state is incorrect. The metabolic acidosis here counts against
hyperosmolar non-ketotic state as the underlying diagnosis.

D Ischaemic colitis

Ischaemic colitis is incorrect. Although patients with DKA are at increased risk of ischaemic
colitis, it is unlikely to be the primary event here.

E Pulmonary embolus

Pulmonary embolus is incorrect. Although the normal range O 2 with reduced CO 2 can be
seen in PE, the marked metabolic acidosis suggests the respiratory picture is because of
compensation rather than the primary cause of his presentation.
46285

Rate this question:

Next Question

Previous Question Tag Question

Feedback End Review

Difficulty: Average

Peer Responses %
Q. Answered Flagged

Q1

Q2

Q3

Q4

Q5

Q6

Q7

Q8

Q9

0:00:36/03:00:00

A 23-year-old woman presents to the Neurology Clinic for review. She feels dizzy when she
moves, particularly when she turns her head to the side. She has a past history of numbness
affecting both hands and around her peri-oral paraesthesia, which caused her to attend the
Emergency Department one year ago, but nil else of note. On examination her BP is 110/80
mmHg, pulse is 65/min and regular. Cardiovascular and respiratory systems are normal,
abdomen is soft and non tender, her BMI is 22. There are no cranial nerve palsies. Dix Hallpike
test is positive to the left.
Which of the following is the most likely diagnosis?

A Acoustic neuroma

B Benign positional vertigo

C Labyrinthitis

D Ménière’s disease

E Multiple sclerosis

Explanation 

B Benign positional vertigo

The positive Dix Hallpike test (which is used to diagnose posterior canal benign positional
vertigo) supports the diagnosis of BPV. It is thought to occur when otoliths become detached
from the macula, continuing to move within the semi-circular canals even when the head has
stopped moving. Approximately 90% of BPV is due to posterior canal disease. Epley’s
manoeuvre can be performed, with case series suggesting 90% or more of individuals where
it is performed do not suffer a serious recurrence of BPV over a 5-year period (although
natural history data to act as a control are not widely available).

A Acoustic neuroma

Acoustic neuroma is incorrect. The lack of cranial nerve palsies or tinnitus, make this an
unlikely diagnosis.

C Labyrinthitis

Labyrinthitis is incorrect. This is usually associated with vertigo with any movement, rather
than movement to one side as is seen here.
D Ménière’s disease

Ménière’s disease is incorrect. Ménière’s disease is associated with episodes of vertigo,


tinnitus and a sensation of aural pressure. They may last from minutes to hours, and tend to
occur in clusters with up to 12 attacks in any one year.

E Multiple sclerosis

Multiple sclerosis is incorrect. The Dix Hallpike test is very consistent with BPV, and the
previous history of numbness is most likely to represent a panic attack.
46257

Rate this question:

Next Question

Previous Question Tag Question

Feedback End Review

Difficulty: Average

Peer Responses %

Q. Answered Flagged

Q1

Q2

Q3

Q4

Q5

Q6

Q7

Q8
Q. Answered Flagged 
0:00:36/03:00:00

A 45-year-old woman presents with worsening symptoms of muscle weakness over the past
week. She has myaesthenia gravis which is usually well controlled on oral azathioprine, and
attended her GP shortly before her symptoms worsened, where she was started on
medication for hypertension. On examination in the clinic you confirm 4/5 muscle weaness
affecting proximal muscles, and marked eye muscle fatigability.
Which of the following medications is most likely to be responsible?

A Atenolol

B Bendroflumethiazide

C Indapamide

D Ramipril

E Valsartan

Explanation 

A Atenolol

Four mechanisms have been postulated by which medications worsen symptoms of


myaesthenia gravis (MG):

Neuronal transmission may be inhibited at the presynaptic terminal


Lack of acetylcholine release (possibly related to inhibition of calcium influx into the
presynaptic terminal);
Blockade of postsynaptic AChRs, thereby preventing the binding of acetylcholine to
the postsynaptic AChR
Prevention of action potential transmission past the postsynaptic terminal due to
changes in postsynaptic ion permeability.

Both calcium channel blockers and beta blockers are well recognised to worsen symptoms of
MG and should therefore be avoided.

B Bendroflumethiazide

Bendroflumethiazide is incorrect. Although thiazide diuretics may be associated with


hypokalaemia and subsequent muscle weakness, they do not worsen muscle weakness
specifically in MG.
C Indapamide

Indapamide is incorrect. Indapamide is a thiazide like diuretic, is less likely versus thiazides to
lead to electrolyte imbalance, and does not impact on muscle power in patients with MG.

D Ramipril

Ramipril is incorrect. ACE inhibitors or ARBS do not impact on muscle power in patients with
MG.

E Valsartan

Valsartan is incorrect. ACE inhibitors or ARBS do not impact on muscle power in patients
with MG.
46272

Rate this question:

Next Question

Previous Question Tag Question

Feedback End Review

Difficulty: Average

Peer Responses %

Q. Answered Flagged

Q1

Q2

Q3

Q4

Q5
Q. Answered Flagged

Q6

Q7

Q8

 External Links

College of Pharmacy
pharmacy.uic.edu/departments/pharmacy-practice/centers-and-sections/drug-information-group/2014/april-2015-fa…
(https://pharmacy.uic.edu/departments/pharmacy-practice/centers-and-
sections/drug-information-group/2014/april-2015-faqs#q1)
0:00:36/03:00:00

A 22-year-old medical student is admitted in an acutely confused state to the Emergency


Department. He believes that he has been sent by God to prepare for the second coming of
Christ. You suspect that he either has a primary psychiatric disorder or has been using
cannabis.
Which of the following features, if present, would be most likely to be associated with
cannabis abuse rather than schizophrenia?

A Auditory hallucinations

B A predominantly negative symptom picture

C Short duration of symptoms

D Depressive symptomatology

E History of poor university performance

Explanation 

C Short duration of symptoms

Patients suffering acute-onset cannabis-related psychosis tend to have a short duration of


illness, while schizophrenia may present as a more insidious, longer-lasting condition.
Cannabis use may increase the relative risk of later schizophrenia two- to fourfold.

A Auditory hallucinations

Auditory hallucinations is incorrect. This is much more strongly associated with schizophrenia
than cannabis misuse, with an estimated 75% of patients with schizophrenia suffering
auditory hallucinations vs up to 15% of those with cannabis suffering psychosis.

B A predominantly negative symptom picture

A predominantly negative symptom picture is incorrect. Patients with cannabis psychosis


often present with a rather grandiose, often hypomanic, symptom picture.

D Depressive symptomatology

Depressive symptomatology is incorrect. Patients with cannabis psychosis often present with
a rather grandiose, often hypomanic, symptom picture.
E History of poor university performance

Poor university performance is incorrect. Patients with cannabis psychosis will often have had
a good premorbid intellectual performance, while low intelligence is a risk factor for
schizophrenia.

Useful external weblinks

Royal College of Psychiatrists: cannabis and mental health


www.rcpsych.ac.uk/healthadvice/problemsdisorders/cannabis.aspx?theme=mobile
(http://www.rcpsych.ac.uk/healthadvice/problemsdisorders/cannabis.aspx?theme=mobile)
8295

Rate this question:

Next Question

Previous Question Tag Question

Feedback End Review

Difficulty: Average

Peer Responses %

Q. Answered Flagged

Q1

Q2

Q3

Q4

Q5

Q6

Q7

Q8



Q. Answered Flagged
0:00:36/03:00:00

A 32-year-old arable farmer comes to the Emergency Department for review. He has bought
a new kitten a few weeks earlier and has sustained a number of scratches to his hands from
playing with the cat. He tells you that since he sustained some particularly nasty scratches to
his right hand a week or so ago, he has begun suffering from headaches and fevers, and has
some swollen lymph nodes in his arm pit. He is otherwise well and takes no regular
medication. On examination his temperature is 37.9 °C, BP is 110/70 mmHg, pulse is 80 bpm
and regular. There are some crusted papules around a healing scratch on the back of his right
hand, and you confirm axillary lymphadenopathy.
Which of the following is the most appropriate intervention?

A Aciclovir

B Ciprofloxacin

C Fluconazole

D Prednisolone

E Reassurance

Explanation 

E Reassurance

This patient is suffering from cat scratch disease, which is usually self-limiting for those who
are immunocompetent, with lymphadenopathy and fevers gradually subsiding over a few
weeks. As such intervention with antibiotics is not indicated. In those where symptoms do
not improve or who are immunocompromised, both azithromycin and ciprofloxacin are
potential interventions. Bartonella henselae is the causative organism.

A Aciclovir

Aciclovir is incorrect. Cat scratch disease is not related to herpes infection, there is therefore
no role for aciclovir.

B Ciprofloxacin

Ciprofloxacin is incorrect. Ciprofloxacin is a possible intervention for cat scratch disease but
only usually considered when there are persistent symptoms or the patient is
immunocompromised.
C Fluconazole

Fluconazole is incorrect. Cat scratch disease is not fungal in origin.

D Prednisolone

Prednisolone is incorrect. Corticosteroids are not indicated in managing cat scratch disease
and may drive more generalised infection.
46235

Rate this question:

Next Question

Previous Question Tag Question

Feedback End Review

Difficulty: Average

Peer Responses %

Q. Answered Flagged

Q1

Q2

Q3

Q4

Q5

Q6

Q7

Q8

Q9

0:00:36/03:00:00

A 50-year-old woman with a 20-year history of rheumatoid arthritis in both hands suddenly
develops finger drop of the ring finger of her right hand.
Which deformity is most likely to have caused this complication?

A Boutonnière deformity

B Fixed hyperextension (swan-neck deformity) of the proximal interphalangeal joint

C Squared hand and fixed adduction of the thumb

D Swelling and dorsal subluxation of the ulnar styloid

E Ulnar deviation at the metacarpophalangeal joints

Explanation 

D Swelling and dorsal subluxation of the ulnar styloid

Swelling and dorsal subluxation of the ulnar styloid leads to wrist pain and may cause rupture
of the finger extensor tendons. This causes finger drop of the little and ring fingers
predominantly, which needs urgent surgical repair.

A Boutonnière deformity

Boutonnière deformity is incorrect because Boutonnière deformities occur in rheumatoid


arthritis, but do not cause finger drop.

B Fixed hyperextension (swan-neck deformity) of the proximal interphalangeal joint

Fixed hyperextension (swan-neck deformity) of the proximal interphalangeal joint is incorrect


because Swan-neck deformities occur in rheumatoid arthritis, but do not cause finger drop.

C Squared hand and fixed adduction of the thumb

Squared hand and fixed adduction of the thumb is incorrect because the squared hand is
seen in severe nodal osteoarthritis and is caused by bony swelling of the carpometacarpal
joint and fixed adduction of the thumb.

E Ulnar deviation at the metacarpophalangeal joints


Ulnar deviation at the metacarpophalangeal joints is incorrect because ulnar deviation at the
metacarpophalangeal joints occurs in rheumatoid arthritis, but does not cause finger drop.
48499

Rate this question:

Next Question

Previous Question Tag Question

Feedback End Review

Difficulty: Average

Peer Responses %

Q. Answered Flagged

Q1

Q2

Q3

Q4

Q5

Q6

Q7

Q8

Q9

0:00:36/03:00:00

A 34-year-old woman who has taken multiple overdoses over the past 3 years is admitted to
the Emergency Department having taken an overdose of 40 dosulepin tablets some 90
minutes earlier. She is drowsy and agitated, having also consumed alcohol, but is co-
operative with the Emergency room staff. On examination her BP is 105/80 mmHg, pulse is
95/min and regular. Her GCS is 14. Chest is clear, abdomen is soft and non tender.
Investigations:

Hb 120 g/l

WCC 6.1 × 10 9/l

PLT 245 × 10 9/l

Na 138 mmol/l

K 3.9 mmol/l

Creatinine 102 µmol/l

ECG QRS 120ms

pH 7.30

Bicarbonate 17 mmol/l

Which of the following is the most appropriate next intervention?

A Amiodarone

B Flumazenil

C Lorazepam

D Naloxone

E Sodium bicarbonate

Explanation 

E Sodium bicarbonate

This patient has a metabolic acidosis with QRS widening, as such she is at significant risk of
ventricular arrhythmia. Guidelines recommend sodium bicarbonate when the QRS is >100ms,
that is therefore the most appropriate next step here. Where hypotension exists and there is
significant dysrhythmia, a target pH above 7.45 is recommended.
A Amiodarone

Amiodarone is incorrect. Amiodarone may promote further complex widening and increase
the risk of ventricular arrhythmias rather than decreasing it.

B Flumazenil

Flumazenil is incorrect. Flumazenil is a reversal agent for benzodiazepine overdose, and when
used in patients who are taking a tricyclic is associated with significant risk of seizures.

C Lorazepam

Lorazepam is incorrect. This could be considered as an option to manage seizures in the


context of tricyclic overdose.

D Naloxone

Naloxone is incorrect. Naloxone is a reversal agent for opiate overdose.


46244

Rate this question:

Next Question

Previous Question Tag Question

Feedback End Review

Difficulty: Average

Peer Responses %

Q. Answered Flagged

Q1

Q2

Q3
Q. Answered Flagged

Q4

Q5

Q6

Q7

Q8

0:00:36/03:00:00

A 42-year-old man who has HIV comes to the Emergency Department complaining of severe
shortness of breath, arthralgia and a rash over his arms and trunk. He took anti-retrovirals for
a brief period from 2008-2010, but did not tolerate them, but returned to the clinic a few
weeks ago and was then started on HAART. On examination he has a temperature of 37.9°C,
BP is 105/70 mmHg, pulse is 95.min and regular. You note a maculopapular rash over his arms
and trunk, and evidence of small joint polyarthritis. He looks pale and is short of breath at
rest. His BMI is 22.
Investigations:

Hb 65 g/l

WCC 5.8 × 10 9/l

PLT 178 × 10 9/l

Na + 138 mmol/l

K+ 4.5 mmol/l

Creatinine 104 µmol/l

Bone marrow Evidence of erythroid aplasia

CD4 count 189 x 10 9/l

Which of the following is the most likely cause of his severe anaemia?

A Cytomegalovirus

B Epstein Barr virus

C Parvovirus B19 infection

D Rubella

E Salmonella infection

Explanation 

C Parvovirus B19 infection

The clues here to parvovirus infection here include the rash, arthritis, and anaemia with red
cell aplasia on bone marrow examination. In patients with reduced CD4 count, even where
highly active antiretroviral therapy (HAART) has commenced, there is increased risk of
clinical Parvovirus infection. HAART should be continued, as a repaired immune system is
likely to accelerate resolution of anaemia.

A Cytomegalovirus

Cytomegalovirus is incorrect. CMV in the immunocompromised more usually results in


respiratory symptoms and signs, and potentially hepatic dysfunction, neither of which are
seen here.

B Epstein Barr virus

Epstein Barr virus is incorrect. Epstein Barr virus in the immunocompromised can result in
anaemia, but an infectious mononucleosis type syndrome is usually seen.

D Rubella

Rubella is incorrect. It is very likely this patient has been vaccinated against rubella, making
this unlikely as a cause of anaemia.

E Salmonella infection

Salmonella infection is incorrect. There are no signs of GI upset to suggest salmonella


infection as a cause of anaemia.
46242

Rate this question:

Next Question

Previous Question Tag Question

Feedback End Review

Difficulty: Average

Peer Responses %

Q. Answered Flagged
Q. Answered Flagged

Q1

Q2

Q3

Q4

Q5

Q6

Q7

Q8 
0:00:36/03:00:00

A 76-year-old woman is referred on the Orthogeriatric Ward where she is recovering after a
left hemi-arthroplasty for hip fracture. She has an indwelling catheter since the operation and
has been spiking fevers over the past 48hrs. She has a temperature of 37.9 °C, BP is 122/82
mmHg, pulse is 80 bpm and regular. Her chest is clear, there is supra-pubic tenderness on
abdominal palpation.
Investigations:

Hb 109 g/l

WCC 12.1 x 10 9/l

PLT 172 x 10 9/l

CRP 189 mg/l

Na + 137 mmol/l

K+ 4.3 mmol/l

Creatinine 110 µmol/l

Catheter specimen of urine analysis Mixed growth of E coli and pseudomonas

Which of the following is the most appropriate next intervention?

A Amoxicillin

B Ciprofloxacin

C Meropenem

D Removal of catheter

E Tazocin

Explanation 

D Removal of catheter

In this situation chronic colonisation of the catheter means infection will not be eradicated
with antibiotic therapy alone. Removal of catheter will however permit successful
intervention. Once the catheter is removed, ciprofloxacin is a potential option for both E coli
and pseudomonas infection.
A Amoxicillin

Amoxicillin is incorrect. Antibiotic therapy is not appropriate without catheter removal first.
Pseudomonas is also usually resistant to amoxicillin.

B Ciprofloxacin

Ciprofloxacin is incorrect. Although both pseudomonas and E coli are sensitive to


ciprofloxacin, the catheter should be removed before commencing therapy.

C Meropenem

Meropenem is incorrect. E coli resistance to meropenem is increasingly emergent, and the


catheter should always be removed first.

E Tazocin

Tazocin is incorrect. Resistance to tazocin is now seen in 10% or more of pseudomonas


isolates, and catheter removal is the essential next step.
46254

Rate this question:

Next Question

Previous Question Tag Question

Feedback End Review

Difficulty: Average

Peer Responses %

Q. Answered Flagged

Q1

Q2
Q. Answered Flagged

Q3

Q4

Q5

Q6

Q7

Q8 
0:00:36/03:00:00

A 24-year-old man presents complaining of passing red urine. This has also occurred on four
to six occasions over the previous few years. The current episode occurred after a game of
football and was associated with weakness and cramping of his thigh muscles. His blood
pressure is 120/76 mmHg and he has no oedema or muscle tenderness. Urinalysis shows 3+
blood, his plasma creatinine concentration is 146 µmol/l and his creatine kinase is 62,652 IU/l.
Which one of the following is the most likely cause of this presentation?

A Carnitine palmitoyltransferase deficiency

B IgA nephropathy

C Muscle trauma with myoglobin-induced tubular injury

D Postinfectious glomerulonephritis

E Renal-cell carcinoma

Explanation 

A Carnitine palmitoyltransferase deficiency

Carnitine palmitoyltransferase (CPT) deficiency is the commonest cause of inherited


metabolic myopathy, resulting in recurrent myoglobinuria. Although the metabolic
myopathies represent a small percentage of those presenting with rhabdomyolysis, they
should be suspected when myoglobinuria is recurrent, being associated with exercise or
fasting and occurring with muscle cramps or weakness. On close questioning, a family history
is often present. In a series of 77 cases of ‘idiopathic’ myoglobinuria, a specific muscle
enzyme abnormality was identified in 36 (47%). It is likely that, in exercising muscle, ATP and
creatine phosphate become depleted and the maintenance of myocyte integrity is
compromised. The enzyme defect is diagnosed using ischaemic forearm testing and muscle
biopsy, which demonstrates abnormal lipid or glycogen deposits.

B IgA nephropathy

IgA nephropathy is incorrect. IgA nephropathy would present with haematuria, but there is
no association with muscle tenderness.

C Muscle trauma with myoglobin-induced tubular injury

Muscle trauma with myoglobin-induced tubular injury is incorrect. The recurrent nature of the
haematuria would go against this as a muscle trauma rhabdomyolysis.
D Postinfectious glomerulonephritis

Renal-cell carcinoma is incorrect. Renal cell carcinoma may present with abdominal mass,
haematuria and flank pain. This scenario is not typical of renal cell carcinoma.

E Renal-cell carcinoma

Post-infectious glomerulonephritis is incorrect. Acute proliferative glomerulonephritis (post-


streptococcal glomerulonephritis) is caused by an infection with Streptococcus spp.,
generally three weeks after infection, usually of the pharynx or the skin, given the time
required to raise antibodies and complement proteins. It is not associated with muscle pain.
1564

Rate this question:

Next Question

Previous Question Tag Question

Feedback End Review

Difficulty: Average

Peer Responses %

Q. Answered Flagged

Q1

Q2

Q3

Q4

Q5

Q6

Q7
Q. Answered Flagged

Q8
0:00:36/03:00:00

A 29-year-old woman presents to the Emergency Department with sudden painless loss of
vision affecting her left eye. She says she awoke with blurred vision that morning, and when it
did not improve came to the hospital. Normally able to read with glasses using her left eye,
she is reduced to count fingers only. Past medical history of a deep vein thrombosis affecting
her right leg some 5 years earlier is noted. She keeps two cats. Apparently her mother
suffered from two DVTs and now takes life-long warfarin. Examination of the right eye is
normal. Examination of the left eye reveals multiple dot/blot and flame haemorrhages
throughout the fundus and some disc oedema. There is an afferent pupillary defect. BP is
155/82 mmHg.
Which of the following is the most likely diagnosis?

A Central retinal artery occlusion

B Central retinal vein occlusion

C CMV retinitis

D Hypertensive retinopathy

E Toxoplasmosis

Explanation 

B Central retinal vein occlusion

The presentation is typical for central retinal vein occlusion, with the onset of painless loss of
visual acuity, accompanied by significant retinal haemorrhages. There is a suspicion of
inherited thrombophilia given the patient’s own and her mothers’ history of venous
thrombosis as the underlying cause. Unfortunately, there are no effective acute interventions.

A Central retinal artery occlusion

Central retinal artery occlusion is incorrect. This is associated with a pale retina, and a cherry
red spot at the centre of the macula.

C CMV retinitis

CMV retinitis is incorrect. This appears as areas of thick white infiltrate surrounded by retinal
haemorrhage, and is slower in onset versus the sudden loss of vision seen here.

D Hypertensive retinopathy
Hypertensive retinopathy is incorrect. Severe retinopathy is more likely to be associated with
headaches and bilateral blurred vision, rather than the marked unilateral signs seen here.

E Toxoplasmosis

Toxoplasmosis is incorrect. Toxoplasmosis is associated with choroidoretinitis rather than the


scattered haemorrhages seen here.
46238

Rate this question:

Next Question

Previous Question Tag Question

Feedback End Review

Difficulty: Average

Peer Responses %

Q. Answered Flagged

Q1

Q2

Q3

Q4

Q5

Q6

Q7

Q8

Q9

0:00:36/03:00:00

A 27-year-old man presents via his GP for review. He wants to start a relationship but is
concerned about his small penis; he also has difficulty becoming aroused. On examination he
is slim and has gynaecomastia; there is a general paucity of body hair, his penis is small and
he has small testes. He has a BMI of 23kg/m 2 and he is tall. His bloods show testosterone 4
nmol/l (NR 9–29).
Which diagnosis fits best with this history and examination?

A Androgen insensitivity syndrome

B Klinefelter syndrome

C Congenital adrenal hyperplasia

D True hermaphroditism

E 5α-reductase deficiency

Explanation 

B Klinefelter syndrome

He has all of the features: hypergonadotropic hypogonadism, microorchidism, gynaecomastia


and tall stature.

A Androgen insensitivity syndrome

Androgen insensitivity syndrome is incorrect. Individuals with this are phenotypic women.

C Congenital adrenal hyperplasia

Congenital adrenal hyperplasia is incorrect. Men with this generally have ambiguous genitalia
or female genitalia, because of inadequate testosterone production in the first trimester of
fetal life.

D True hermaphroditism

True hermaphroditism is incorrect. True hermaphrodites are vanishingly rare and can be
XX/XY or mosaic in karyotype; they possess both testes and ovaries, and usually have either
male or ambiguous external genitalia.
E 5α-reductase deficiency

5α-reductase deficiency is incorrect. Individuals with this are phenotypic women; it is an


autosomal recessive condition.

Klinefelter syndrome/47,XXY

This refers to a group of chromosomal disorders in which the normal male karyotype, 46,XY,
has at least one extra X chromosome. XXY aneuploidy has a prevalence of 1 in 500 men; it is
also the most common chromosomal disorder associated with male hypogonadism and
infertility.

Klinefelter syndrome is characterised by hypogonadism (microorchidism [small testes],


oligospermia/azoospermia), gynecomastia in late puberty, hyalinisation and fibrosis of the
seminiferous tubules, elevated urinary gonadotropin levels and behavioural concerns.
Testosterone replacement therapy is an important aspect of treatment.
50368

Rate this question:

Next Question

Previous Question Tag Question

Feedback End Review

Difficulty: Average

Peer Responses %

Q. Answered Flagged

Q1

Q2

Q3

Q4

Q5

Q6
Q. Answered Flagged

Q7

Q8 
0:00:36/03:00:00

A 27-year-old woman comes to the Sexually Transmitted Diseases Clinic complaining of pain
on passing urine, a urethral discharge and discomfort on sexual intercourse. She has
previously attended the clinic with an episode of gonorrhoea, and she has just left her
partner after he admitted to intercourse outside the relationship. You confirm pelvic
tenderness on examination, and discharge.
Investigations;
Positive NAAT for gonorrhoea
HIV: negative
Chlamydia: negative
Which of the following complications is of most concern in recurrent gonorrhoea infection?

A Arthropathy

B Conjunctivitis

C Fitz High Curtis syndrome

D Infertility

E Jarisch Herxheimer reaction

Explanation 

D Infertility

The risk of infertility associated with gonorrhoea infection increases with recurrent infection,
because of pelvic inflammatory disease coupled with scarring and adhesions.

A Arthropathy

Arthropathy is incorrect. Disseminated gonococcal arthritis results in septic arthritis, and


inflammatory arthritis without bacterial infection is also seen in association with gonococcal
infection. The risk of this occurring does not increase with recurrent gonorrhoea.

B Conjunctivitis

Conjunctivitis is incorrect. Conjunctivitis is occasionally seen in association with gonorrhoea


infection, the risk does not however increase with recurrent infective episodes.

C Fitz High Curtis syndrome


Fitz High Curtis syndrome (FHC) is incorrect. FHC is rarely seen, and is associated with
severe. It is associated with right upper quadrant pain due to peri-hepatitis, due to ascending
infection, which may be due to gonorrhoea or to chlamydia. It can be managed with
antibiotic therapy.

E Jarisch Herxheimer reaction

Jarisch Herxheimer reaction is incorrect. This is most likely to be seen in association with
syphilis infection, and is due to massive death of spirochetes in response to antibiotic
therapy.
46253

Rate this question:

Next Question

Previous Question Tag Question

Feedback End Review

Difficulty: Average

Peer Responses %

Q. Answered Flagged

Q1

Q2

Q3

Q4

Q5

Q6

Q7

Q8
Q. Answered Flagged 
0:00:36/03:00:00

You are asked to examine management and outcomes for patients admitted via the
Emergency Department with community acquired pneumonia at the local district general
hospital where you are working.
Which of the following would make your work audit rather than research?

A Examination of care versus agreed standards

B Examination of co-morbidities

C Examination of mortality outcomes

D Examination of position of X-ray changes

E Examination of resistance patterns

Explanation 

A Examination of care versus agreed standards

The main difference between audit and research is that audit examines level of care versus
agreed standards. In this case the agreed standard would be the BTS/SIGN guideline on the
management of community acquired pneumonia. The other key facet of audit is the ability to
close the loop, (i.e. to recommend changes, and then re-audit their impact on clinical care).

B Examination of co-morbidities

Examination of co-morbidities is incorrect. It is clear that examination of co-morbidities may


be important in audit, (where these may drive treatment decisions), and research, where you
may be interested in how co-morbidities impact on outcomes.

C Examination of mortality outcomes

Examination of mortality outcomes is incorrect. Again, mortality outcomes may be measured


against SMRs for other hospitals / a national standard, and in the setting of an observational
clinical study.

D Examination of position of X-ray changes

Examination of position of X-ray changes is incorrect. X-ray changes in no way relate to an


audit standard and would therefore be considered research.
E Examination of resistance patterns

Examination of resistance patterns is incorrect. Resistance patterns may be audited against


standards for antibiotic prescribing, or constitute observational research against changing
patterns of pneumonia in the UK.
46229

Rate this question:

Next Question

Previous Question Tag Question

Feedback End Review

Difficulty: Average

Peer Responses %

Q. Answered Flagged

Q1

Q2

Q3

Q4

Q5

Q6

Q7

Q8

Q9

0:00:36/03:00:00

A 42-year-old woman who is pregnant with her second child comes to the clinic to get the
results of her amniocentesis. The amniocentesis chromosome analysis confirms the offspring
is 47 XY + 21.
With regards to chromosome 21, how are his cells described?

A Diploid

B Haploid

C Pentoid

D Tetroid

E Triploid

Explanation 

E Triploid

Down’s syndrome is trisomy 21, where each cell contains three rather than two copies of
chromosome 21. Mosaic Downs also exists, where only some of the cells contain three rather
than two copies of chromosome 21.

A Diploid

Diploid is incorrect. Diploid cells contain two copies of the chromosome.

B Haploid

Haploid is incorrect. Haploid cells contain one copy of the chromosome.

C Pentoid

Pentoid is incorrect. Pentoid cells contain five copies of the chromosome. Pentoid cells
containing multiple copies of the x chromosome are rarely identified.

D Tetroid

Tetroid is incorrect. Tetroid cells contain four copies of the chromosome. Tetroid cells
containing multiple copies of the x chromosome are also rarely identified.
46280
Rate this question:

Next Question

Previous Question Tag Question

Feedback End Review

Difficulty: Average

Peer Responses %

Q. Answered Flagged

Q1

Q2

Q3

Q4

Q5

Q6

Q7

Q8

Q9

0:00:36/03:00:00

A 38-year-old amateur rugby player is admitted to the Emergency Department some 2hrs
after the onset of fast, irregular palpitations. He has been at a wedding where he drank a
substantial amount of alcohol the previous evening. The palpitations began soon after he
awoke the next morning and he has not yet eaten or drunk anything. He has no past medical
history of note apart from mild asthma for which he takes a salbutamol inhaler, and is a non-
smoker. On examination his BP is 95/70 mmHg, pulse is 130/min (irregular), heart sounds are
normal and his chest is clear.
Investigations:

Hb 135 g/l

WCC 6.7 × 10 9/l

PLT 203 × 10 9/l

Na + 139 mmol/l

K+ 4.9 mmol/l

Creatinine 105 µmol/l

ECG Atrial fibrillation, no evidence of previous MI or acute ischaemia

CXR No cardiomegaly, no focal abnormality

Which of the following is the most appropriate intervention?

A Amiodarone

B Bisoprolol

C Digoxin

D Electrical cardioversion

E Flecainide

Explanation 

D Electrical cardioversion

In this case with no evidence of previous cardiovascular disease, and AF of very short
duration, electrical cardioversion is appropriate. He has a very good chance of achieving sinus
rhythm. In the event that cardioversion is not achieved, an appropriate anti-arrhythmic such
as flecainide or bisoprolol can be instituted with plans for a further attempt at electrical
cardioversion later.

A Amiodarone

Amiodarone is incorrect. Amiodarone is most suitable for chemical cardioversion where there
is existing structural or ischaemic heart disease, (where flecainide is contra-indicated).

B Bisoprolol

Bisoprolol is incorrect. Although bisoprolol is recommended by NICE for oral use in patients
with AF, it is much less effective versus other options with respect to achieving sinus rhythm
acutely.

C Digoxin

Digoxin is incorrect. Digoxin is appropriate only for rate control, and does not increase the
chances of attaining sinus rhythm.

E Flecainide

Flecainide is incorrect. Flecainide in this situation would be a first choice for chemical
cardioversion and could be considered in this patient if they refused electrical cardioversion.
46248

Rate this question:

Next Question

Previous Question Tag Question

Feedback End Review

Difficulty: Average

Peer Responses %

Q. Answered Flagged
Q. Answered Flagged

Q1

Q2

Q3

Q4

Q5

Q6

Q7

Q8 
0:00:36/03:00:00

A 42-year-old woman presents to the Gastroenterology Clinic with gradually worsening


tiredness and lethargy, itching and vague right upper quadrant tenderness over the past 4-6
months. She also has significantly reduced appetite and suffers from extended periods of
nausea. On examination her BP is 110/70 mmHg, pulse is 70/min and regular. There are
multiple spider naevi, scratch marks over the upper body, and 2 finger breadth hepatomegaly.
Investigations:

Hb 111 g/l

WCC 7.9 x 10 9 /l

PLT 112 x 10 9 /l

Na 139 mmol/l

K 4.5 mmol/l

Creatinine 90 micromol/l

ALT 790 U/l

ALP 229 U/l

Bilburin 29 micromol/l

Anti-smooth muscle antibody Positive

Anti-nuclear antibody Positive

Which of the following is the most likely diagnosis?

A Autoimmune hepatitis

B Haemochromatosis

C Primary biliary cirrhosis

D Primary sclerosing cholangitis

E Wilson’s disease

Explanation 

A Autoimmune hepatitis
This patient has a much larger rise in transaminases compared with the rise seen in alkaline
phosphatase, and positive anti-smooth muscle antibodies, taken together these are features
are consistent with a diagnosis of autoimmune hepatitis. Polyclonal
hypergammaglobulinaemia is also a common finding. Liver biopsy is the most important
diagnostic investigation, and corticosteroids are the initial therapy of choice.

B Haemochromatosis

Haemochromatosis is incorrect. Haemochromatosis rarely presents in pre-menopausal


women because of the monthly blood loss afforded by the menstrual cycle.

C Primary biliary cirrhosis

Primary biliary cirrhosis is incorrect. PBC is associated with a proportionally greater rise in
alkaline phosphatase, and is associated with anti-mitochondrial antibodies.

D Primary sclerosing cholangitis

Primary sclerosing cholangitis is incorrect. PSC is most often seen against a background of
inflammatory bowel disease.

E Wilson’s disease

Wilson’s disease is incorrect. Wilson’s usually presents in the second or early in the third
decade, often with neuropsychiatric symptoms.
46225

Rate this question:

Next Question

Previous Question Tag Question

Feedback End Review

Difficulty: Average

Peer Responses %
Q. Answered Flagged

Q1

Q2

Q3

Q4

Q5

Q6

Q7

Q8

Q9 
0:00:36/03:00:00

A 73-year-old man with a history of Parkinson’s disease is brought to the clinic by his family.
They are concerned about a change in his behaviour over the past few months associated
with starting ropinirole for the treatment of Parkinson’s disease. He has been sleeping
excessively in the day, and has also taken to gambling large amounts of money on a poker
machine at the local bookmakers.
Which of the following represents the mode of action of ropinirole?

A Anti-cholinergic

B COMT inhibitor

C Dopamine agonist

D Glutamate antagonist

E MAO-B inhibitor

Explanation 

C Dopamine agonist

Ropinirole is a dopamine agonist used in the treatment of Parkinson’s disease, specifically it is


a D2/D3 agonist, (non-ergot), hence it does not carry the liability of pulmonary or cardiac
fibrosis of earlier compounds used for the indication. It is associated with an increase in risk
taking behaviour, and changes in sleep patterns.

A Anti-cholinergic

Anti-cholinergic is incorrect. Anti-cholinergics are used primarily to improve tremor,


examples include orphenadrine and procyclidine.

B COMT inhibitor

COMT inhibitor is incorrect. COMT inhibitors block metabolism of levodopa, and tend to be
used late in the treatment of the condition. Examples include entacapone.

D Glutamate antagonist

Glutamate antagonist is incorrect. Amantadine is a glutamate antagonist, it has relatively


weak effects in terms of improving tremor and rigidity, as such it is rarely used in the
treatment of Parkinson’s.
E MAO-B inhibitor

MAO-B inhibitor is incorrect. Monoamine oxidase is responsible for dopamine metabolism, as


such inhibiting it potentiates the action of dopamine and L-dopa, improving Parkinson’s
symptoms. Selegiline is an example of the class.
46230

Rate this question:

Next Question

Previous Question Tag Question

Feedback End Review

Difficulty: Average

Peer Responses %

Q. Answered Flagged

Q1

Q2

Q3

Q4

Q5

Q6

Q7

Q8

Q9

 External Links
Pharmacodynamic properties
medicines.org.uk/emc/medicine/17844#PHARMACODYNAMIC_PROPS
(http://www.medicines.org.uk/emc/medicine/17844#PHARMACODYNAMIC_PROPS)
0:00:36/03:00:00

A 40-year-old patient has been diagnosed with Hodgkin’s disease. He is being treated with
radiotherapy and chemotherapy.
Which one of the following is the most important factor predicting his prognosis?

A Erythrocyte sedimentation rate

B Gender

C Lymph node sizes

D Response to treatment

E Spleen size

Explanation 

D Response to treatment

The response to treatment is the most important factor for predicting this man’s prognosis. In
fact, this is usually the case for all malignant processes. A good response to initial treatment
will naturally be of good prognosis, whereas refractory disease usually indicates a poor
overall prognosis.

A Erythrocyte sedimentation rate

Erythrocyte sedimentation rate (ESR) is incorrect. The ESR is considered prognostic for early
stage Hodgkin’s disease, with a raised ESR being considered a risk factor.

B Gender

Gender is incorrect. Gender is considered to be a poor risk factor in advanced Hodgkin’s


disease, with being male an adverse prognostic factor.

C Lymph node sizes

Lymph node sizes is incorrect. The lymph nodes sizes are less prognostic, but a bulky
mediastinal mass is considered a risk factor for prognosis in early stage Hodgkin’s disease.
Advanced stage disease (stage IV) is considered prognostic for advanced Hodgkin’s disease.

E Spleen size
Spleen size is incorrect. Spleen size is not considered prognostic in either early or advanced
Hodgkin’s disease.
43694

Rate this question:

Next Question

Previous Question Tag Question

Feedback End Review

Difficulty: Average

Peer Responses %

Q. Answered Flagged

Q1

Q2

Q3

Q4

Q5

Q6

Q7

Q8

Q9

0:00:36/03:00:00

An 18-year-old man presents to his GP with thirst and polyuria. Some three months previously
he had a significant head injury as the result of a road traffic accident. He is referred to the
local endocrine clinic.
Which of the following results would be the most useful in confirming a diagnosis of
diabetes insipidus after a water deprivation test (without additional desmopressin)?

A Plasma sodium of 126 mmol/l

B Plasma sodium of 150 mmol/l

C Plasma osmolality of 335 mOsmol/kg and urine osmolality of 700 mOsmol/kg

D Plasma osmolality of 280 mOsmol/kg and urine osmolality of 700 mOsmol/kg

E Plasma osmolality of 335 mOsmol/kg and urine osmolality of 200 mOsmol/kg

Explanation 

E Plasma osmolality of 335 mOsmol/kg and urine osmolality of 200 mOsmol/kg

This elevated plasma osmolarity, combined with dilute urine of <340 m)smo/kg in a patient
that has completed a period of supervised fluid deprivation, is consistent with a diagnosis of
diabetes insipidus. To differentiate between cranial and nephrogenic DI, subcutaneous
DDAVP will be administered and urine osmolality measured at intervals thereafter. In patients
with cranial DI, urine osmolality will become more concentrated after DDAVP, but in patients
with nephrogenic DI there will be no change in urine concentration post-DDAVP.

A Plasma sodium of 126 mmol/l

Plasma sodium of 126 mmol/l is incorrect. Patients with diabetes insipidus will continue to
produce large volumes of dilute urine despite fluid deprivation. Therefore hypernatremia due
to volume contraction and clinical dehydration is more common than hyponatremia.

B Plasma sodium of 150 mmol/l

Plasma sodium of 150 mmol/l is incorrect. Hypernatremia is more common in patients who
have deficiency of ADH (cranial DI) or resistance to the action of ADH (nephrogenic DI).
However, hypernatremia can be multifactorial and therefore cannot be used alone to
diagnose diabetes insipidus.

C Plasma osmolality of 335 mOsmol/kg and urine osmolality of 700 mOsmol/kg


Plasma osmolality of 335 mOsmol/kg and urine osmolality of 700 mOsmol/kg is incorrect.
The plasma osmolality in this case, which is increased, would be consistent with the plasma
osmolality expected in patients with DI, but a urine osmolality of >350 mOsmol/kg after fluid
deprivation indicates an ability of the kidney to concentrate before any DDAVP. Ttherefore
this would rule out a diagnosis of diabetes insipidus.

D Plasma osmolality of 280 mOsmol/kg and urine osmolality of 700 mOsmol/kg

Plasma osmolality of 280 mOsmol/kg and urine osmolality of 700 mOsmol/kg is incorrect.
The serum osmolality in this case is low and the urine osmolality is concentrated. This would
be more consistent with a diagnosis of SIADH rather than ADH deficiency.
50229

Rate this question:

Next Question

Previous Question Tag Question

Feedback End Review

Difficulty: Average

Peer Responses %

Q. Answered Flagged

Q1

Q2

Q3

Q4

Q5

Q6

Q7
Q. Answered Flagged

Q8
0:00:36/03:00:00

A 50-year-old man with a history of peptic ulcer disease who was H pylori positive and
received eradication therapy, comes to the Gastroenterology Clinic some three months later
for review. His symptoms have resolved, and he is now taking low dose PPI therapy only.
Which of the following is the most appropriate way to test for H pylori eradication?

A Endoscopy and biopsy

B H pylori serology

C Stool antigen

D 2 months trial off PPI and symptom assessment

E Urea breath test with 2 weeks off PPI

Explanation 

E Urea breath test with 2 weeks off PPI

NICE guidance recommends the urea breath test as the optimal way to confirm eradication
of H pylori after therapy. Other non-invasive tests are not considered reliable enough.

A Endoscopy and biopsy

Endoscopy and biopsy is incorrect. This is unnecessary when a urea breath test is adequate
to confirm H pylori eradication.

B H pylori serology

H pylori serology is incorrect. Office based serology testing is considered too unreliable with
respect to confirming H pylori status.

C Stool antigen

Stool antigen is incorrect. Although stool antigen testing can be used as an initial
investigation, it is not recommended for confirming eradication by NICE guidelines.

D 2 months trial off PPI and symptom assessment


2 months trial off PPI and symptom assessment is incorrect. With respect to long term
prognosis it is clearly important to establish whether clearance of H pylori has been
successful.
46236

Rate this question:

Next Question

Previous Question Tag Question

Feedback End Review

Difficulty: Average

Peer Responses %

Q. Answered Flagged

Q1

Q2

Q3

Q4

Q5

Q6

Q7

Q8

Q9

 External Links

Helicobacter pylori testing and eradication in adults


pathways.nice.org.uk/pathways/dyspepsia-and-gastro-oesophageal-reflux-disease/helicobacter-pylori-testing-and-e…
(http://pathways.nice.org.uk/pathways/dyspepsia-and-gastro-oesophageal-reflux-
disease/helicobacter-pylori-testing-and-eradication-in-adults#content=view-
node%3Anodes-h-pylori-testing)
0:00:36/03:00:00

A 27-year-old woman comes to the Sexually Transmitted Diseases Clinic complaining of pain
on passing urine, a urethral discharge and discomfort on sexual intercourse. She has
previously attended the clinic with an episode of gonorrhoea, and she has just left her
partner after he admitted to intercourse outside the relationship. You confirm pelvic
tenderness on examination, and discharge.
Investigations:
Positive NAAT for gonorrhoea
HIV: negative
Chlamydia: negative
Which of the following is the most appropriate intervention?

A Amoxicillin

B Ceftriaxone and azithromycin

C Ciprofloxacin

D Doxycycline

E Erythromycin and metronidazole

Explanation 

B Ceftriaxone and azithromycin

The fact this patient has been infected for the second time with gonorrhoea, does not change
the accepted intervention, a single dose of ceftriaxone IM, and oral azithromycin.

A Amoxicillin

Amoxicillin is incorrect. Penicillins are not recommended for the treatment of gonorrhoea
because of increasing resistance to penicillins.

C Ciprofloxacin

Ciprofloxacin is incorrect. There is widespread resistance of gonorrhoea to quinolones, as


such they are no longer recommended as first line intervention in the UK for gonorrhoea.

D Doxycycline
Doxycycline is incorrect. Doxycycline is recommended for the treatment of chlamydia
infection, not for gonorrhoea because of resistance.

E Erythromycin and metronidazole

Erythromycin and metronidazole is incorrect. Twice daily erythromycin and metronidazole for
14 days is a recommended intervention for pelvic inflammatory disease.
46252

Rate this question:

Next Question

Previous Question Tag Question

Feedback End Review

Difficulty: Average

Peer Responses %

Q. Answered Flagged

Q1

Q2

Q3

Q4

Q5

Q6

Q7

Q8

Q9

0:00:36/03:00:00

A 20-year-old woman is admitted to the Emergency Department with a right sided weakness
affecting her face and arm. She returned from Australia some 3 days earlier and has been
confined to bed since with a heavy cold. On examination her BP is 155/100 mmHg, pulse is 80
bpm and regular, she is anxious. You confirm right sided facial droop and 3/5 arm weakness.
She also has a swollen tender right calf. Her symptoms of weakness gradually resolve over a
period of four hours.
Investigations:

Hb 130 g/l

WCC 7.0 x 10 9/l

PLT 201 x 10 9/l

CRP 33 mg/l

Na + 138 mmol/l

K+ 4.4 mmol/l

Creatinine 90 µmol/l

Glucose 7.1 mmol/l

CT Head No acute lesion

ECG Sinus rhythym

Which is the most likely cause of her stroke?

A Cerebral amyloidosis

B Paroxysmal atrial fibrillation

C Patent foramen ovale

D Septic embolus

E Thrombophilia

Explanation 

C Patent foramen ovale


The suggestion from the swollen calf is that this patient has an underlying DVT related to her
long flight from Australia and period of inactivity. Any clot occurring in the venous system
has then passed to the arterial side of the circulation and led to a TIA. Echocardiography is
therefore the definitive next investigation with respect to confirming the diagnosis.

A Cerebral amyloidosis

Cerebral amyloidosis is incorrect. Cerebral amyloidosis is a disease of the elderly and most
often presents with multiple small strokes due to microhaemorrhages.

B Paroxysmal atrial fibrillation

Paroxysmal atrial fibrillation (PAF) is incorrect. PAF is a possibility although we are given no
history of palpitations, and this would not fit as the cause in the presence of the swollen right
calf.

D Septic embolus

Septic embolus is incorrect. CRP is mildly elevated and white count is normal, this does not
therefore fit well with a diagnosis of bacterial sepsis.

E Thrombophilia

Thrombophilia is incorrect. Although this may explain the presence of a DVT, it would not
provide linking aetiology between the DVT and stroke presentation.
46241

Rate this question:

Next Question

Previous Question Tag Question

Feedback End Review

Difficulty: Average

Peer Responses %
Q. Answered Flagged

Q1

Q2

Q3

Q4

Q5

Q6

Q7

Q8

Q9 
0:00:36/03:00:00

A 55-year-old man is found incidentally to have hypercalcaemia during a routine health


screen.
Which one of the following biochemical findings would be most suggestive of this being
caused by primary hyperparathyroidism rather than any other cause of hypercalcaemia?

A Elevated 24 h urinary calcium excretion

B Elevated serum alkaline phosphatase activity

C Low serum concentration of calcitriol (1,25-dihydroxycholecalciferol)

D Normal or elevated serum phosphate concentration

E Serum parathyroid hormone (PTH) concentration within the normal range

Explanation 

E Serum parathyroid hormone (PTH) concentration within the normal range

Although PTH concentrations are often increased in patients with primary


hyperparathyroidism, they are not always so. PTH secretion should be suppressed by
hypercalcaemia from any other cause, so that a PTH value in the normal range is consistent
with the diagnosis.

A Elevated 24 h urinary calcium excretion

Elevated 24 h urinary calcium excretion is incorrect. Urinary calcium excretion is increased in


many cases of hypercalcaemia (except in familial hypocalciuric hypercalcaemia), including
hypercalcaemia due to hyperparathyroidism. This is because the kidney attempts to filter off
excess calcium regardless of the cause of high calcium. In familial hypocalciuric
hypercalcaemia there would be high serum calcium and low urinary calcium. In this condition,
which is an autosomal dominant condition, there may be symptoms of hypercalcaemia.

B Elevated serum alkaline phosphatase activity

Elevated serum alkaline phosphatase activity is incorrect. Serum alkaline phosphatase activity
can also be elevated with hypercalcaemia, regardless of the cause (with the exception of
myeloma). It is also elevated in liver disease, especially in cholestasis.

C Low serum concentration of calcitriol (1,25-dihydroxycholecalciferol)


Low serum concentration of calcitriol (1,25-dihydroxycholecalciferol) is incorrect. Calcitriol
concentrations tend to be increased in hyperparathyroidism (PTH stimulates the formation of
this hormone). As calcitriol is elevated in any cause of hyperparathyroidism, it does not give
further information in regard to the aetiology of hypercalcaemia. It can be used to help
define the cause of low calcium.

D Normal or elevated serum phosphate concentration

Normal or elevated serum phosphate concentration is incorrect. Serum phosphate


concentrations tend to be reduced in hyperparathyroidism (PTH is phosphaturic). Therefore,
elevated phosphate would not be in keeping with primary hyperparathyroidism.
2578

Rate this question:

Next Question

Previous Question Tag Question

Feedback End Review

Difficulty: Difficult

Peer Responses %

Q. Answered Flagged

Q1

Q2

Q3

Q4

Q5

Q6

Q7

Q8
Q. Answered Flagged 
0:00:36/03:00:00

A 71-year-old man is diagnosed with early Alzheimer’s disease and started on donepezil, and
over the past few months his wife has noticed a gradual fall off in his appetite. Apart from
that there has been an improvement in his mental state score.
Which of the following represents the mode of action of donepezil?

A Acetylcholinesterase inhibitor

B Dopamine agonist

C Nicotinic receptor agonist

D NMDA antagonist

E Noradrenaline reuptake inhibitor

Explanation 

A Acetylcholinesterase inhibitor

Donepezil is a specific reversible acetylcholinesterase inhibitor used in the treatment of


Alzheimer’s. Prominent side effects include those associated with cholinergic activation,
including GI upset, (diarrhoea, nausea and vomiting), and urinary incontinence.

B Dopamine agonist

Dopamine agonist is incorrect. Dopamine agonists such as ropinirole are commonly used in
the treatment of Parkinson’s disease, and may result in increased risk of hallucinations /
agitation in patients with dementia.

C Nicotinic receptor agonist

Nicotinic receptor agonist is incorrect. Galantamine, another reversible inhibitor of


acetylcholinesterase also enhances the action of acetylcholine on nicotinic receptors,
probably through allosteric receptor binding.

D NMDA antagonist

NMDA antagonist is incorrect. Memantine is an NMDA antagonist, which is known to reduce


glutamate toxicity and activation.
E Noradrenaline reuptake inhibitor

Noradrenaline reuptake inhibitor is incorrect. Noradrenaline reuptake inhibitors such as


reboxetine are used as antidepressants and for the treatment of attention deficit
hyperactivity disorder.
46245

Rate this question:

Next Question

Previous Question Tag Question

Feedback End Review

Difficulty: Average

Peer Responses %

Q. Answered Flagged

Q1

Q2

Q3

Q4

Q5

Q6

Q7

Q8

Q9

 External Links
Undesirable effects
medicines.org.uk/EMC/medicine/577/SPC/Aricept+Tablets/#UNDESIRABLE_EFFECTS
(https://www.medicines.org.uk/EMC/medicine/577/SPC/Aricept+Tablets/#UNDESIRABLE_E
0:00:36/03:00:00

A 45-year-old woman who has undergone a renal transplant and is taking a tacrolimus based
immunosuppressive regime is reviewed in the Transplant Clinic.
Which of the following cytokines does tacrolimus inhibit the production of?

A IL-1

B IL-2

C IL-4

D IL-6

E IL-10

Explanation 

B IL-2

Tacrolimus binds to FKBP-12, inhibiting a range of T cell signal transduction pathways. In


particular it inhibits the formation of cytotoxic lymphocytes, in turn reducing production of
pro-inflammatory cytokines such as IL2 and IL3.

A IL-1

IL1 is incorrect. IL1 is a pro-inflammatory cytokine involved in the pathogenesis of a number of


conditions including acute flares of gout. Canakinumab is an example of an IL-1 inhibitor.

C IL-4

IL4 is incorrect. IL4 is binds to a macrophage surface receptor and is involved in the
pathogenesis of atopic dermatitis.

D IL-6

IL6 is incorrect. IL6 is an inflammatory cytokine thought to play a role in the development of
rheumatoid arthritis. Tocilizumab is an IL6 inhibitor.

E IL-10
IL10 is incorrect. IL10 is an anti-inflammatory cytokine and is thought to play a role in
regulatory T cell signalling.
46216

Rate this question:

Next Question

Previous Question Tag Question

Feedback End Review

Difficulty: Average

Peer Responses %

Q. Answered Flagged

Q1

Q2

Q3

Q4

Q5

Q6

Q7

Q8

Q9

0:00:36/03:00:00

A 24-year-old woman attends the neurological clinic for review of multiple sclerosis,
diagnosed 2 years before. She had presented with blurring of vision and mild pain in her left
eye, which had resolved over a period of 3 months and had not recurred. On examination
now, the following observations are made: light shone in the left eye causes constriction of
the left and right pupils; light shone into the right eye causes constriction of the right and left
pupils, but when the light is shone back into the left eye, the left pupil dilates slightly.
Which of the following is the most likely site of the lesion responsible?

A Left ciliary ganglion

B Left oculomotor nerve

C Left optic nerve

D Right ciliary ganglion

E Right optic nerve

Explanation 

C Left optic nerve

The abnormal response is on the left; the right pupil shows a normal direct and consensual
response. The normal constriction of the left pupil indicates that the efferent pathway
(involving the Edinger–Westphal nucleus, oculomotor nerve and ciliary ganglion) is intact.
The defect is in the afferent pathway on the left, which involves fibres in the optic nerve. The
sign demonstrated is a relative afferent pupillary defect (RAPD), implying partial damage
only to the afferent pathway: if all function is lost, neither the direct reflex nor the consensual
reflex (constriction of the right pupil in response to light shone into the left eye) will be
present.

A Left ciliary ganglion

Left ciliary ganglion is incorrect. A defect in the left ciliary ganglion would drive
abnormal/lack of constriction of the left pupil due to an efferent pathway defect.

B Left oculomotor nerve

Left oculomotor nerve is incorrect. This would result in features of a third nerve palsy causing
a down and out movement of the eye position.
D Right ciliary ganglion

Right ciliary ganglion is incorrect. This would result in an efferent pathway defect affecting
the right eye.

E Right optic nerve

Right optic nerve is incorrect. A right optic nerve lesion would result in an abnormal response
to light when light is shone into the right eye.
48815

Rate this question:

Next Question

Previous Question Tag Question

Feedback End Review

Difficulty: Average

Peer Responses %

Q. Answered Flagged

Q1

Q2

Q3

Q4

Q5

Q6

Q7

Q8

Q9

0:00:36/03:00:00

A 49-year-old man with a history of psoriasis presents to the Rheumatology Clinic with a
deforming small joint polyarthritis particularly affecting the distal interphalangeal (DIP) joints
of the fingers, and to a lesser extent his wrists and elbows. He has active skin disease, for
which his GP has prescribed topical corticosteroids. He is unable to continue his job as an
electrician due to pain in his hands. On examination, there is nail pitting and obvious psoriasis
affecting the backs of his elbows, his knees and his scalp.
Investigations:

Hb 12.0 g/dl

WCC 10.1 × 10 9/l

PLT 189 × 10 9/l

Na + 137 mmol/l

K+ 4.3 mmol/l

Creatinine 90 µmol/l

ESR 65 mm/1 st hour

Glucose 4.9 mmol/l

Which of the following is the most appropriate next intervention?

A Ibuprofen

B Infliximab

C Methotrexate

D Naproxen

E Oral corticosteroids

Explanation 

C Methotrexate

In this situation, with active skin disease and arthritis, a traditional DMARD that will act
positively on both aspects is the most appropriate next step. Weekly methotrexate is
therefore the correct answer. Leflunomide and sulphasalazine are alternative traditional
DMARDs to methotrexate.
A Ibuprofen

Ibuprofen is incorrect. Ibuprofen is a common non-steroidal anti-inflammatory drug (NSAID)


available over the counter. For this patient, it is an option for symptomatic relief, but since he
has evidence of psoriatic arthritis with joint deformity he would need treatment to affect the
underlying disease process.

B Infliximab

Infliximab is incorrect. Infliximab is an anti-tumour necrosis factor (TNF) therapy which is a


refractory option in the treatment of inflammatory bowel disease, rheumatoid arthritis,
psoriatic arthritis, psoriasis and ankylosing spondylitis. However, its use is generally limited by
cost and therefore this should be considered an option only in the case of failure on disease-
modifying anti-rheumatic drug (DMARD) treatment. Psoriasis responds well to anti-TNF
therapies; in the event that the response to methotrexate is inadequate, an anti-TNF agent is
an appropriate next step.

D Naproxen

Naproxen is incorrect. Naproxen is a potent NSAID. For this patient, it is an option for
symptomatic relief, but since he has evidence of psoriatic arthritis with joint deformity he
would need treatment to affect the underlying disease process.

E Oral corticosteroids

Oral corticosteroids is incorrect. Prednisolone is not recommended for the treatment of


psoriatic arthritis. In addition, in psoriasis there can be rebound disease following withdrawal
of oral steroids, making it a less than ideal treatment option.
40138

Rate this question:

Next Question

Previous Question Tag Question

Feedback End Review

Difficulty: Average

Peer Responses %
Q. Answered Flagged

Q1

Q2

Q3

Q4

Q5

Q6

Q7

Q8

Q9

0:00:36/03:00:00

A 40-year-old man presents to the Emergency Department with difficulty breathing and
swallowing. Examination is normal. Chest X-ray shows an enlarged upper mediastinum, so
you arrange a computed tomography scan, which is performed the next day. This shows an
enlarged thymus gland.
What would you do next?

A Arrange a biopsy of the thymus

B Arrange a fine-needle aspirate of the thymus

C Await the results of antibody testing prior to making a decision

D Refer to the oncologists for radiotherapy

E Refer to the surgeons for thymus excision

Explanation 

E Refer to the surgeons for thymus excision

This man has a thymoma, which is a tumour of epithelial origin arising in the thymus. Between
30% and 40% of patients with a thymoma have myasthenia gravis and will have positive anti-
acetylcholine-receptor antibodies; 20% of patients with myasthenia gravis have a thymoma.
The myasthenia often does not improve after the thymus is removed. Management of
thymoma is surgical resection, particularly for a patient experiencing symptoms of mass
effect such as dysphagia and dyspnoea.

A Arrange a biopsy of the thymus

Arrange a biopsy of the thymus is incorrect. Thymomas contained within the thymic capsule
tend to be benign, but those that have extended beyond it are generally malignant. Biopsy
can breech the capsule and so increase the risk of thymoma tumour seeding and should be
avoided.

B Arrange a fine-needle aspirate of the thymus

Arrange a fine-needle aspirate of the thymus is incorrect. Thymomas contained within the
thymic capsule tend to be benign, but those that have extended beyond it are generally
malignant. Fine-needle aspiration can breech the capsule and so increase the risk of
thymoma tumour seeding and should be avoided.
C Await the results of antibody testing prior to making a decision

Await the results of antibody testing prior to making a decision is incorrect. Awaiting the
results of antibody testing does not alter the management, which is thymectomy.

D Refer to the oncologists for radiotherapy

Refer to the oncologists for radiotherapy is incorrect. Initial management is surgical


resection. Post-operative radiotherapy is indicated for malignant or incompletely excised
thymomas.
45573

Rate this question:

Next Question

Previous Question Tag Question

Feedback End Review

Difficulty: Average

Peer Responses %

Q. Answered Flagged

Q1

Q2

Q3

Q4

Q5

Q6

Q7

Q8
Q. Answered Flagged 
0:00:36/03:00:00

A 64-year-old woman comes to the Emergency Department complaining of worsening


shortness of breath and dull right-sided chest pain. She smokes 20 cigarettes per day and
had a right breast lumpectomy some four years ago. Other history of note includes an inferior
myocardial infarction and hypertension for which she takes atorvastatin, furosemide and
ramipril. On examination her BP is 135/80 mmHg, pulse is 80 bpm and regular. Heart sounds
are normal. The right side of her chest is dull to percussion, and she has decreased breath
sounds at the right base.
Investigations:

Hb 119 g/l

WCC 9.5 x 10 9/l

PLT 189 x 10 9/l

Na + 133 mmol/l

K+ 4.5 mmol/l

Creatinine 95 µmol/l

CXR Large right sided pleural effusion

Which of the following would fit best with a malignant pleural effusion when pleural fluid is
analysed?

A BNP 80 pg/ml (<100)

B Elevated eosinophils

C Mesothelial cells

D pH 7.28

E Pleural fluid amylase

Explanation 

D pH 7.28

The pH of normal pleural fluid is approximately 7.6 because of the gradient between pleural
fluid and blood. pH below 7.3 represents a substantial accumulation of hydrogen ions, either
because of increased acid production by pleural fluid cells and bacteria, or decreased
hydrogen efflux due to pleuritic, tumour or fibrosis.
A BNP 80 pg/ml (<100)

BNP 80 pg/ml is incorrect. This BNP is in the normal range. Although elevated BNP in pleural
fluid is supportive of a diagnosis of heart failure, there is no value in measuring it in pleural
fluid versus plasma measurement.

B Elevated eosinophils

Elevated eosinophils is incorrect. Eosinophils are a non-specific finding in pleural fluid, but are
not generally found in tuberculous pleural effusions.

C Mesothelial cells

Mesothelial cells is incorrect. Mesothelial cells are seen in small numbers in normal pleural
fluid.

E Pleural fluid amylase

Pleural fluid amylase is incorrect. An amylase ratio of 0.95 is considered normal. Elevated
pleural fluid amylase can be seen in pancreatitis, oesophageal rupture, and where there is
underlying malignancy.
46249

Rate this question:

Next Question

Previous Question Tag Question

Feedback End Review

Difficulty: Average

Peer Responses %

Q. Answered Flagged

Q1
Q. Answered Flagged

Q2

Q3

Q4

Q5

Q6

Q7

Q8 
0:00:36/03:00:00

A 32-year-old woman presents to the Emergency Department with right hip pain a few days
after running in a half marathon. She says the pain has occurred previously associated with
training runs, but has got much worse in the period after the half marathon. She has no past
medical history of note and takes no regular medication. When you ask her to walk she has
an obvious limp and complains of pain on the outside of her hip, and you can elicit
tenderness on palpation over the greater trochanter. Active movement in particular is limited
by pain.
Which of the following is the most appropriate intervention?

A Diclofenac gel

B Iliotibial band lengthening surgery

C Local corticosteroid injection

D Oral naproxen

E Oral prednisolone

Explanation 

D Oral naproxen

This patient has greater trochanteric pain syndrome, precipitated by her distance running. It
is a repetitive overload tendinopathy of the gluteus medius and minimus muscles. These
muscles play a primary role in hip abduction and pelvic stabilization whilst walking, running,
and standing on one leg. In the first instance a reduction in weight bearing exercise such as
running, coupled with oral NSAID use, are the interventions of choice.

A Diclofenac gel

Diclofenac gel is incorrect. Diclofenac gel is less effective versus systemic naproxen, and is
therefore not likely to adequately resolve this patient’s pain.

B Iliotibial band lengthening surgery

Iliotibial band lengthening surgery is incorrect. Surgery to lengthen the band, bursectomy
and gluteal muscle repair are all potential interventions, but they are usually reserved for
patients failing NSAIDs and local corticosteroid injection.

C Local corticosteroid injection


Local corticosteroid injection is incorrect. Local corticosteroids are reserved for patients who
fail to gain control of symptoms on NSAIDs.

E Oral prednisolone

Oral prednisolone is incorrect. Oral prednisolone offers no added benefit compared to local
delivery of corticosteroids.
46264

Rate this question:

Next Question

Previous Question Tag Question

Feedback End Review

Difficulty: Average

Peer Responses %

Q. Answered Flagged

Q1

Q2

Q3

Q4

Q5

Q6

Q7

Q8

Q9

0:00:36/03:00:00

A 68-year-old handyman is admitted to the Emergency Department having fallen off a ladder
whilst cleaning windows. After complaining of pain over his thoracic spine, he undergoes a
chest X-ray which reveals evidence of pleural plaques. He is a non-smoker and complains of
no respiratory symptoms. Walking up to five miles per day.
Which of the following is the most appropriate next step?

A Bronchoscopy and biopsy

B CT thorax

C No intervention required

D Pulmonary function testing

E Trial of corticosteroids

Explanation 

C No intervention required

Although pleural plaques indicate previous exposure to asbestos, guidance from the British
Thoracic Society (BTS), indicates that in the absence of other chest disease, further
investigation is not indicated and may indeed be harmful. As such this patient should be
discharged without further intervention.

A Bronchoscopy and biopsy

Bronchoscopy and biopsy is incorrect. This would only be indicated for potential malignancy
or to aid diagnosis of fibrotic lung disease.

B CT thorax

CT thorax is incorrect. CT thorax is an option for evaluation of focal abnormalities on the x-


ray or changes consistent with pulmonary fibrosis.

D Pulmonary function testing

Pulmonary function testing is incorrect. There are no symptoms indicating a need for
pulmonary function testing.
E Trial of corticosteroids

Trial of corticosteroids is incorrect. A trial of corticosteroids may be considered for


pulmonary fibrosis, not a consideration here.
46261

Rate this question:

Next Question

Previous Question Tag Question

Feedback End Review

Difficulty: Average

Peer Responses %

Q. Answered Flagged

Q1

Q2

Q3

Q4

Q5

Q6

Q7

Q8

Q9

0:00:36/03:00:00

A 41-year-old nurse is admitted to the Emergency Department following an anaphylactoid


reaction on the third occasion over the last 6 months. She has acute onset stridor, facial
flushing, hypoxia and hypotension. On examination her BP is 95/60 mmHg, pulse is 95 bpm
and regular. O 2 saturation is reduced at 91% on air.
Which of the following blood tests is most useful in biochemical confirmation of
anaphylaxis?

A Serial C3 measures

B Serial CRP measures

C Serial eosinophils

D Serial mast cell tryptase measures

E Serial total IgE measures

Explanation 

D Serial mast cell tryptase measures

Anaphylaxis is characterised by systemic mast cell degranulation, which is associated with


elevated mast cell tryptase. Single measurements may however be difficult to interpret, at
least one sample at peak symptoms, followed by one or two in the recovery period are
therefore recommended.

A Serial C3 measures

Serial C3 measures is incorrect. Complement levels are more variable and therefore less
useful in confirming an episode of anaphylaxis versus mast cell tryptase. They may also be
reduced as a result of other causes of immune activation.

B Serial CRP measures

Serial CRP measures is incorrect. CRP is an acute phase protein is likely to rise more slowly
versus mast cell tryptase, and is non-specific.

C Serial eosinophils

Serial eosinophils is incorrect. Eosinophils are more likely to be elevated in chronic asthma or
atopic dermatitis, and are less useful in evaluation of acute anaphylaxis.
E Serial total IgE measures

Serial total IgE measures is incorrect. Total IgE is not useful in evaluation of allergic reactions,
although antigen specific IgE testing may be useful in identifying a potential allergen.
46270

Rate this question:

Next Question

Previous Question Tag Question

Feedback End Review

Difficulty: Average

Peer Responses %

Q. Answered Flagged

Q1

Q2

Q3

Q4

Q5

Q6

Q7

Q8

Q9

0:00:36/03:00:00

You are asked to see a 73-year-old man who is being treated with dual IV antibiotic therapy
for bacterial endocarditis. He underwent prosthetic aortic valve replacement some nine
months earlier. The nurses are concerned because despite three days of antibiotics he
continues to spike fevers at least twice per day. Examination reveals a BP of 115/70 mmHg,
pulse is 55 bpm and regular. He has a systolic murmur loudest in the aortic area.
Which of the following would be the strongest indicator for surgical referral?

A Elevated creatinine

B Fevers after three days’ antibiotics

C Mild cardiac failure

D Prolonged PR interval

E Staphylococcus aureus on blood culture

Explanation 

D Prolonged PR interval

Prolonged PR interval raises the possibility of an aortic root abscess, which will not resolve
with antibiotics alone, is associated with risk of perivalvular rupture of the abscess, and
developing heart block.

A Elevated creatinine

Elevated creatinine is incorrect. Although elevated creatinine is associated with poorer


surgical outcomes, it is not an indicator for urgent progression to surgery.

B Fevers after three days’ antibiotics

Fevers after three days’ antibiotics is incorrect. Persistent fevers for 5 days or more after
antibiotic initiation are considered an indication for surgery.

C Mild cardiac failure

Mild cardiac failure is incorrect. This may of course pre-date the valve replacement and be
related to pre-existing ischaemic cardiovascular disease.
E Staphylococcus aureus on blood culture

Staphylococcus aureus on blood culture is incorrect. Although S aureus is associated with


poor outcomes in prosthetic valve endocarditis, it is not an immediate driver for surgical
intervention.
46259

Rate this question:

Next Question

Previous Question Tag Question

Feedback End Review

Difficulty: Average

Peer Responses %

Q. Answered Flagged

Q1

Q2

Q3

Q4

Q5

Q6

Q7

Q8

Q9

0:00:36/03:00:00

A 49-year-old man with alcoholic liver disease comes to the clinic for review. He has given up
alcohol six months previously and followed a low salt diet, yet he has trouble with gradually
increasing fluid retention. He takes BD propanolol and thiamine, but is on no other
medication. His BP is 115/82 mmHg, pulse is 64 bpm and regular. There is moderate ascites,
and pitting oedema to the mid shin.
Investigations:

Hb 109 g/l

WCC 6.2 × 10 9/l

PLT 105 × 10 9/l

Na + 133 mmol/l

K+ 3.7 mmol/l

Creatinine 85 µmol/l

Albumin 25 g/l

ALT 92 U/l

ALP 132 U/l

Bilirubin 18 µmol/l

Which of the following is the most appropriate way to manage his ascites?

A Furosemide 40mg

B Furosemide 40mg and Spironolactone 50mg

C Spironolactone 100mg

D Stop propanolol

E Therapeutic ascitic tap

Explanation 

C Spironolactone 100mg

Patients with advanced cirrhosis suffer from mineralocorticoid excess; as such


spironolactone, an aldosterone antagonist is the initial intervention of choice. Rather than
increasing the dose beyond 100mg in the event that ascites is not controlled, addition of
furosemide is recommended.

A Furosemide 40mg

Furosemide 40mg is incorrect. Loop diuretics are preferred as add-on therapy in the
management of ascites, rather than initial therapy.

B Furosemide 40mg and Spironolactone 50mg

Furosemide 40mg and spironolactone 50mg is incorrect. Combination loop and aldosterone
antagonist therapy is recommended as the next step if ascites is not controlled on
spironolactone alone.

D Stop propanolol

Stop propranolol is incorrect. Stopping the propranolol risks worsening portal hypertension
and should therefore be avoided.

E Therapeutic ascitic tap

Therapeutic ascitic tab is incorrect. Therapeutic ascitic tap is reserved for ascites resistant to
diuretics, and is usually accompanied by human albumin replacement.
46265

Rate this question:

Next Question

Previous Question Tag Question

Feedback End Review

Difficulty: Average

Peer Responses %

Q. Answered Flagged
Q. Answered Flagged

Q1

Q2

Q3

Q4

Q5

Q6

Q7

Q8
0:00:36/03:00:00

A 37-year-old man presents to the Neurology Department a few months after a significant
rear shunt road traffic accident where a car travelling at approximately 30 miles per hour
collided with his vehicle. Over the past few months he has begun to develop loss of pain and
temperature sensation affecting both arms, pain in his neck and shoulders, and weakness
affecting his hands. He takes no regular medication, is a non-smoker who drinks occasional
alcohol, and is currently unable to work as a chef because of the problem with his hands. On
examination his BP is 122/70 mmHg, pulse is 75 bpm and regular. There is sensory loss over
the upper arms and shoulders, and you confirm weakness affecting the small muscles of the
hands.
Which of the following is the most likely diagnosis?

A Anterior spinal artery syndrome

B Cervical radiculopathy

C Spondylolisthesis

D Spondylosis

E Syringomyelia

Explanation 

E Syringomyelia

Traumatic syringomyelia, seen after an episode of trauma, is not uncommon, and the slow
onset of neurological symptoms, predominantly in the upper arms, a few months after the
original accident, fits well with the diagnosis. MRI of the spine is the initial investigation of
choice.

A Anterior spinal artery syndrome

Anterior spinal artery syndrome is incorrect. Anterior spinal artery syndrome does present
with pain and temperature sensation loss below the level of the lesion, although it occurs
acutely, and is associated with complete motor paralysis below the lesion rather than just
weakness of the hands as was seen here.

B Cervical radiculopathy
Cervical radiculopathy is incorrect. Cervical radiculopathy is associated with neck pain,
although weakness and sensory loss is associated with the particular nerve root which is
compressed, tending to be more marked on one side than the other.

C Spondylolisthesis

Spondylolisthesis is incorrect. Spondylolisthesis, or forward movement of a vertebra is seen


most often in the lumbar spine, patients tend to hold their posture in forward flexion,
complain of dull lower back pain, and tingling in both lower limbs.

D Spondylosis

Spondylosis is incorrect. Cervical spondylosis occurs due to degenerative / arthritic change


in the neck, presenting with chronic neck pain and evidence of radiculopathy, in contrast to
the symptoms here that cannot be mapped to a single nerve root.
46286

Rate this question:

Next Question

Previous Question Tag Question

Feedback End Review

Difficulty: Average

Peer Responses %

Q. Answered Flagged

Q1

Q2

Q3

Q4

Q5
Q. Answered Flagged

Q6

Q7

Q8
0:00:36/03:00:00

A 19-year-old student comes to the Emergency Department complaining of a generalised


skin rash. He suffered from a sore throat two weeks earlier, for which he was prescribed
some penicillin via the student health service. He is worried, as on further questioning he
admits to having had oral sex with a new partner just a few days before becoming unwell. On
examination he is apyrexial. His pharyngitis has resolved, but he has multiple papules and
scaly red plaques on his body, ranging from a few millimetres to around 10–15 mm in
diameter; he says these are mildly itchy.
Investigations:

Hb 12.4 g/dl

WCC 9.1 x 10 9/l

PLT 230 x 10 9/l

Na + 139 mmol/l

K+ 4.4 mmol/l

Cr 100 µmol/l

Which one of the following is the most likely diagnosis?

A Guttate psoriasis

B Penicillin allergy

C Reactive arthritis

D Rheumatic fever

E Secondary syphilis

Explanation 

A Guttate psoriasis

Guttate psoriasis follows upper respiratory tract infection, and streptococcal infection in
particular. The rash normally appears around 2 weeks after the antecedent infection, and it
appears very acutely with multiple papules/small plaques appearing within a very short time.
The palms and soles are usually spared, and the diagnosis is a clinical one. The condition
normally resolves over the course of a few weeks, and simple emollients are often the only
treatment required. For resistant cases topical steroids or ultraviolet (UV) light therapy may
be of value.
B Penicillin allergy

Penicillin allergy is incorrect. A drug allergy rash is very different, and more likely to be a
maculopapular rash without scales.

C Reactive arthritis

Reactive arthritis is incorrect. There is no history of arthritis.

D Rheumatic fever

Rheumatic fever is incorrect. He does not fit the criteria for rheumatic fever, which may
include a geographical-like rash (erythema marginatum), which is non-itchy.

E Secondary syphilis

Secondary syphilis is incorrect. Secondary syphilis occurs approximately 4–10 weeks after the
primary infection (his sexual activity was only a few days ago). There may be a symmetrical,
reddish-pink, non-itchy rash on the trunk and extremities, including the palms and soles.
48970

Rate this question:

Next Question

Previous Question Tag Question

Feedback End Review

Difficulty: Difficult

Peer Responses %

Q. Answered Flagged

Q1

Q2
Q. Answered Flagged

Q3

Q4

Q5

Q6

Q7

Q8 
0:00:36/03:00:00

A 72-year-old man with a 2-day history of severe neck pain and headache presents to the
Emergency Department with protrusion of his tongue and left arm and leg weakness. He has
a history of hypertension for which his GP has prescribed three anti-hypertensive agents,
although he admits that his compliance with therapy is poor. On examination his BP is
175/100 mmHg, pulse is 80 bpm and regular. There is weakness of the tongue with protrusion
towards the right hand side. There is left sided arm and leg weakness although the face is
spared, and you note right sided loss of vibration and proprioception sense.
Where is the most likely site of the lesion?

A Basilar artery

B Carotid artery

C Posterior communicating artery

D Posterior inferior cerebellar artery

E Vertebral artery

Explanation 

E Vertebral artery

This patient has features consistent with the medial medullary (also known as Dejerine)
syndrome. The cause is occlusion of the vertebral artery or its branch to the anterior spinal
artery with involvement of the pyramid, medial lemniscus and they hypoglossal nerve.

A Basilar artery

Basilar artery is incorrect. Basilar artery lesions more usually present with ischaemia affecting
the upper brainstem or Pons, with infarction of the upper ventral Pons leading to
presentation with “locked-in” syndrome.

B Carotid artery

Carotid artery is incorrect. Carotid territory lesions present with hemiparesis without facial
sparing and sensory loss on the same side, accompanied by other features such as dysphasia.

C Posterior communicating artery


Posterior communicating artery is incorrect. Posterior communicating artery lesions can
present with dorsal midbrain syndrome, leading to ipsilateral oculomotor palsy, ptosis, and
mydriasis and contralateral involuntary movements.

D Posterior inferior cerebellar artery

Posterior inferior cerebellar artery is incorrect. PICA lesions present with the classical features
of lateral medullary syndrome, rather than those of medial medullary syndrome seen here.
46240

Rate this question:

Next Question

Previous Question Tag Question

Feedback End Review

Difficulty: Average

Peer Responses %

Q. Answered Flagged

Q1

Q2

Q3

Q4

Q5

Q6

Q7

Q8

Q9

0:00:36/03:00:00

A 54-year-old woman with a history of urgency and urinary incontinence comes to the clinic
for review. She is taking a number of medications for epilepsy and is concerned that these
may lead to loss of control over bladder emptying.
Bladder emptying occurs as a result of what?

A Decreased parasympathetic nervous activity

B Decreased sympathetic nervous activity

C Increased dopaminergic nervous activity

D Increased parasympathetic nervous activity

E Increased sympathetic nervous activity

Explanation 

D Increased parasympathetic nervous activity

Parasympathetic nerves originating from S2-S4 innervate visceral motor neurones in


parasympathetic ganglia in or near the bladder wall. They promote bladder contraction
leading to emptying. Muscarinic antagonists are the mainstay of treatment of bladder
hyperactivity.

A Decreased parasympathetic nervous activity

Decreased parasympathetic nervous activity is incorrect. This is responsible for hypoactive


bladder and may lead to incomplete emptying.

B Decreased sympathetic nervous activity

Decreased sympathetic nervous activity is incorrect. Sympathetic nervous system activity


leads to closure of the internal urethral sphincter. Decreased activity therefore promotes
sphincter relaxation but not bladder emptying where muscle contraction is required.

C Increased dopaminergic nervous activity

Increased dopaminergic nervous activity is incorrect. Although dopaminergic nervous system


dysfunction may result in incontinence, it is parasympathetic nerves that have the primary
role in bladder emptying.
E Increased sympathetic nervous activity

Increased sympathetic nervous activity is incorrect, this promotes closure of the internal
urethral sphincter.
46217

Rate this question:

Next Question

Previous Question Tag Question

Feedback End Review

Difficulty: Average

Peer Responses %

Q. Answered Flagged

Q1

Q2

Q3

Q4

Q5

Q6

Q7

Q8

Q9

0:00:36/03:00:00

A 51-year-old man who is known to suffer from gout comes to the Emergency Department
with a hot, swollen right knee. He takes allopurinol 200mg per day but has suffered two
attacks of acute gout during the past year. On examination his temperature is 37.8 °C, BP is
135/80 mmHg, pulse is 85 bpm and regular. There is a significant right knee effusion with
erythema and limitation of flexion to only 30 degrees.
Investigations:

Hb 129 g/l

WCC 10.9 x 10 9/l

PLT 210 x 10 9/l

CRP 88 mg/l

Na + 138 mmol/l

K+ 4.5 mmol/l

Creatinine 105 µmol/l

Urate 0.45 mmol/l

Joint aspirate 55,000 white cells per microliter, (no organisms identified)

Which of the following is the most appropriate intervention?

A Intra-articular corticosteroids

B Increased allopurinol

C IV antibiotics

D Oral colchicine

E Oral prednisolone

Explanation 

C IV antibiotics

Although the CRP is only moderately elevated and this patient has a past history of gout with
elevated uric acid, the joint aspirate white count is suspicious of septic arthritis (>50,000
white cells per microliter). As such it is safest to manage them as septic arthritis whilst
cultures are awaited.
A Intra-articular corticosteroids

Intra-articular corticosteroids is incorrect. Given the elevated synovial fluid white cell count,
intra-articular steroids should be avoided at least until the sample is confirmed as culture
negative.

B Increased allopurinol

Increased allopurinol is incorrect. Even if this is an acute attack of gout, increasing allopurinol
in the acute situation should be avoided.

D Oral colchicine

Oral colchicine is incorrect. Whilst colchicine is an acceptable treatment for acute gout, given
the synovial fluid white count, the presumed diagnosis should be septic arthritis until proven
otherwise.

E Oral prednisolone

Oral prednisolone is incorrect. Oral prednisolone is usually considered a second or third line
option for acute gout in patients who cannot tolerate a NSAID or colchicine.
46269

Rate this question:

Next Question

Previous Question Tag Question

Feedback End Review

Difficulty: Average

Peer Responses %

Q. Answered Flagged

Q1
Q. Answered Flagged

Q2

Q3

Q4

Q5

Q6

Q7

Q8 
0:00:36/03:00:00

A 65-year-old man, who has smoked 20 cigarettes a day for 50 years and who has worked in
the ship-building industry, presents to the clinic with chest pain and increasing shortness of
breath. Clinical examination and chest radiography show he has a right pleural effusion.
What investigation would you perform next?

A Abrams’ pleural biopsy

B Bronchoscopy

C CT scan

D Pleural aspiration under ultrasound guidance

E Thoracoscopy

Explanation 

D Pleural aspiration under ultrasound guidance

Pleural aspiration is a useful preliminary test and can be performed in the clinic, under
ultrasound guidance. Pleural fluid should be sent for cytology, protein, lactate dehydrogenase
(LDH), pH and culture. The likely causes for the pleural effusion in this man are metastatic
lung cancer or mesothelioma. Pleural aspiration can be diagnostic in the case of lung cancer,
but not in mesothelioma.

A Abrams’ pleural biopsy

Abrams’ pleural biopsy is incorrect. Abrams’ pleural biopsies are now rarely used in the
investigation of suspected malignant pleural disease. CT pleural biopsy has been shown to
have a diagnostic advantage over a blind Abrams’ biopsy in cytologically negative malignant
pleural effusions with pleural thickening, with a sensitivity of 87% vs 47%. (see Maskell NA, et
al., Lancet 2003. 361: 1326–1330).

B Bronchoscopy

Bronchoscopy is incorrect. Bronchoscopy may not necessarily be required in this case to


obtain a diagnosis. Ultrasound-guided diagnostic pleural aspiration can be performed as a
first-line investigation and may reveal malignant cells on cytological examination. CT scanning
should then be performed to stage any suspected intrathoracic malignancy. Bronchoscopy
would then be indicated if pleural fluid cytology was negative and CT scanning revealed a
relatively central primary lung malignancy.
C CT scan

CT scan is incorrect. CT scanning should be arranged following diagnostic pleural aspiration.


In lung cancer this might show a chest primary and, depending on its location, a
bronchoscopy or CT-guided biopsy can then be performed. In cases of mesothelioma, CT
might show pleural thickening and nodularity, which would be amenable to either CT-guided
biopsy or thoracoscopy.

E Thoracoscopy

Thoracoscopy is incorrect. Thoracosopy is a very helpful test used in the investigation of


pleural effusions as histology can be be taken, pleural fluid can be drained and pleurodesis, if
appropriate, can be performed. However, it would not be performed as the first investigation
in the situation described in this case.
45227

Rate this question:

Next Question

Previous Question Tag Question

Feedback End Review

Difficulty: Average

Peer Responses %

Q. Answered Flagged

Q1

Q2

Q3

Q4

Q5

Q6
Q. Answered Flagged

Q7

Q8

0:00:36/03:00:00

A 23-year-old woman presents to the Sexually Transmitted Diseases Clinic complaining of a


grey frothy vaginal discharge which has an ammonia or fishy like odour. She admits to
unprotected sexual intercourse with two different partners over the past three months, and
that she has washed up to 2-3 times per day which seems to have no impact on the
discharge. General physical examination is unremarkable. You confirm a grey, frothy vaginal
discharge which has a fishy odour, and clue cells are seen on microscopy.
Which of the following is the most appropriate treatment?

A Amoxicillin

B Cephalexin

C Ciprofloxacin

D Doxycycline

E Metronidazole

Explanation 

E Metronidazole

The symptoms and clue cells seen on microscopy are typical of a diagnosis of bacterial
vaginosis, it is associated with an increase in vaginal pH from approximately 4.5 to 5.5, which
may actually be further increased in women who use over the counter non pH balanced
shower gels. Metronidazole, in the form of local gel and/ or systemic therapy, is the
intervention of choice. Whilst Gardnerella is not considered a sexually transmitted disease,
given she has had unprotected intercourse on two occasions in the past 3 months, it is
appropriate for her to be offered screening for other STDs.

A Amoxicillin

Amoxicillin is incorrect. Gardnerella is not sensitive to penicillins.

B Cephalexin

Cephalexin is incorrect. Cephalosporins may be used in the treatment of gonorrhoea.

C Ciprofloxacin
Ciprofloxacin is incorrect. Ciprofloxacin was formerly used as a treatment for gonorrhoea,
although resistance patterns mean it is no longer considered a first line option.

D Doxycycline

Doxycycline is incorrect. Doxycycline is an intervention for the treatment of gonorrhoea.


46223

Rate this question:

Next Question

Previous Question Tag Question

Feedback End Review

Difficulty: Average

Peer Responses %

Q. Answered Flagged

Q1

Q2

Q3

Q4

Q5

Q6

Q7

Q8

Q9

0:00:36/03:00:00

A 38-year-old city trader comes to the Emergency Department with severe agitation
following a cocaine overdose in a local night club. He has not previously presented to the
hospital before, and has no past medical history of note. He tells you that he snorts the drug
on up to two occasions per day. His BP as measured in the ambulance is 155/90 mmHg, pulse
is 100 bpm and regular.
Which of the following other symptoms is he most likely to complain of?

A Abdominal pain

B Bloody diarrhoea

C Chest pain

D Constipation

E Wheeze

Explanation 

C Chest pain

Cocaine abuse leads to dose-dependent coronary vasoconstriction, with proven cardiac


ischaemia associated with ECG changes +/- a rise in troponin being seen in approximately 5%
of attendees at the Emergency Department. In this situation shortness of breath, pleuritic,
and central angina type chest pains are common. Tobacco smoking is thought to accelerate
the development of cocaine abuse related atherosclerosis.

A Abdominal pain

Abdominal pain is incorrect. Mesenteric ischaemia is associated with cocaine abuse, and is a
cause of abdominal pain, although it less likely to be a presenting feature of cocaine abuse
versus chest pain.

B Bloody diarrhoea

Bloody diarrhoea is incorrect. This is associated with ischaemic colitis, a rarer complication of
cocaine abuse.

D Constipation
Constipation is incorrect. Cocaine is more likely to lead to reduced bowel transit time and
diarrhoea, rather than constipation

E Wheeze

Wheeze is incorrect. While crack cocaine is associated with pulmonary damage, so called
“crack lung”, intranasal use of cocaine is not, although it may be associated with
exacerbations of obstructive lung disease in a patient with pre-existing asthma.
46231

Rate this question:

Next Question

Previous Question Tag Question

Feedback End Review

Difficulty: Average

Peer Responses %

Q. Answered Flagged

Q1

Q2

Q3

Q4

Q5

Q6

Q7

Q8

Q9

0:00:36/03:00:00

A 36-year-old paediatric nurse who received a renal transplant for IgA nephropathy some six
years earlier comes to the Renal Clinic for review. She has suffered from increasing bone pain,
polyuria and polydipsia over the past seven months, and has been started on anti-
depressants by her GP for mood changes. Her GP has recently reduced her dose of vitamin D
due to hypercalcaemia. On examination her BP is 155/88 mmHg, pulse is 80 bpm and regular.
She has a BMI of 21 and you can feel a mass on palpation of the abdomen consistent with the
transplanted kidney.
Investigations:

Hb 109 g/l

WCC 6.7 x 10 9 /l

PLT 168 x 10 9 /l

Na + 137 mmol/l

K+ 5.1 mmol/l

Creatinine 232 µmol/l

Ca 2.81 mmol/l

PO4 1.7 mmol/l

PTH 18 pmol/l (1.2-5.8)

Which of the following is the most appropriate intervention?

A Cinacalcet

B Dipyridamole

C Increase vitamin D

D Parathyroidectomy

E Sevelamer

Explanation 

D Parathyroidectomy
Debate exists between proceeding straight to surgery for tertiary hyperparathyroidism
following renal transplant, versus a trial of cinacalcet. In patients taking tacrolimus, cinacalcet
is associated with at least a moderate reduction in exposure, and should therefore be
avoided. One RCT suggests that surgical outcomes are better in this population versus
medical therapy.

A Cinacalcet

Cinacalcet is incorrect. Cinacalcet is appropriate for patients with tertiary


hyperparathyroidism where surgery is considered inappropriate, usually due to pre-morbid
condition. It is not associated with any cardiovascular benefit when used over the long term,
despite impacting on symptoms of hyperparathyroidism.

B Dipyridamole

Dipyridamole is incorrect. Dipyridamole increases tubular reabsorption of phosphate and is


therefore considered an intervention for hypophosphataemia.

C Increase vitamin D

Increase vitamin D is incorrect. This will result in a further rise in serum calcium.

E Sevelamer

Sevelamer is incorrect. Sevelamer is used to reduce phosphate, in this situation PTH has
already escaped feedback control, reducing phosphate is therefore likely to have little impact
on hyperparathyroidism.
46222

Rate this question:

Next Question

Previous Question Tag Question

Feedback End Review

Difficulty: Average

Peer Responses %
Q. Answered Flagged

Q1

Q2

Q3

Q4

Q5

Q6

Q7

Q8

Q9

0:00:36/03:00:00

A 44-year-old man with a moderate learning disability lives with his elderly mother who has
recently suffered a stroke. His mother reports that, since her stroke, her son has stopped
going to his local day centre, has lost weight and has taken to his bed, refusing to talk to her.
What is the most likely diagnosis?

A Dementia

B Depression

C Anxiety

D Psychosis

E Mania

Explanation 

B Depression

People with learning disabilities are less likely than those without learning disabilities to
complain of mood changes or to express depressive ideas. Depression may manifest with the
presence of other symptoms of depression, such as loss of interest in normal activities,
behavioural changes and changes in appetite and sleep. Depression in older people also
presents less with depressed mood and more with the physical symptoms, such as weight
loss and disturbed sleep. The management of depression in these patients remains the same
as other members of the general population.
Depression diagnosis as explained in the NICE guidance requires at least one of the following
key symptoms, present most days and most of the time for at least two weeks:

Persistent sadness or low mood; and/or


Marked loss of interests or pleasure

If any of these are present, the following associated symptoms should also be explored:

Disturbed sleep (increased or decreased from usual)


Decreased or increased appetite
Fatigue or loss of energy
Agitation or slowing of movements
Poor concentration or indecisiveness
Feelings of worthlessness or excessive or inappropriate guilt
Suicidal thoughts or acts.

A Dementia

Dementia is incorrect. There is no mention of cognitive impairment, and dementia has a slow
progression and longer duration of illness than described in this case.

C Anxiety

Anxiety is incorrect. The patient does not have any cognitive symptoms of anxiety such as
apprehension, fear or a state of hyper-vigilance or somatic symptoms such as palpitations,
shortness of breath, nausea, urinary frequency, tingling around the mouth or in the fingers.

D Psychosis

Psychosis is incorrect. There is no evidence of hallucinations, delusions or thought disorders.

E Mania

Mania is incorrect. The patient is not experiencing elevated mood, over activity, inflated self-
esteem or grandiosity or over activity that would be associated with mania.

Useful external weblinks:

Depression in adults (http://www.nice.org.uk/guidance/cg90/chapter/Appendix-C-


Assessing-depression-and-its-severity)
1874

Rate this question:

Next Question

Previous Question Tag Question

Feedback End Review

Difficulty: Average

Peer Responses %
Q. Answered Flagged

Q1

Q2

Q3

Q4

Q5

Q6

Q7

Q8

Q9

0:00:36/03:00:00

A 63-year-old man who has worked in the shipyard making warships for the past 40 years is
admitted to the Emergency Department complaining of gradually progressive shortness of
breath, right sided dull chest ache, and severe lethargy. He now finds it very difficult to even
get out of the house to go to the shops. He smokes ten cigarettes per day and takes a
salbutamol inhaler for COPD. On examination in the department, he is short of breath at rest.
His BP is 132/70 mmHg, pulse is 85 bpm (AF) and he has reduced right-sided chest
expansion. His chest is dull to percussion at the right base, and there are decreased breath
sounds on the right-hand side. O 2 saturation is 92% on air.
Investigations:

Hb 109 g/l

WCC 8.1 × 10 9/l

PLT 167 × 10 9/l

Na + 128 mmol/l

K+ 4.5 mmol/l

Creatinine 122 µmol/l

CRP 15 mg/l

ESR 65 mm/1st hour

ALT 89 U/l

ALP 102 U/l

Bilburin 13 micromol/l

CXR Right sided pleural effusion, evidence of bilateral pleural plaques

Which of the following is the most likely diagnosis?

A Adenocarcinoma of the bronchus

B Mesothelioma

C Small cell carcinoma of the bronchus

D Squamous cell carcinoma of the bronchus

E Tuberculous effusion

Explanation 
B Mesothelioma

The clue here is the career as a shipyard worker, coupled with the presence of pleural
plaques and a pleural effusion. Taken together these factors are suggestive of a malignant
effusion due to underlying mesothelioma. Hyponatraemia is not uncommon in mesothelioma,
and is indicative of a poor outcome. Diagnostic pleural tap / biopsy is the next most
appropriate step in confirming the diagnosis.

A Adenocarcinoma of the bronchus

Adenocarcinoma of the bronchus is incorrect. Pleural effusions are associated with bronchial
carcinoma, although the scenario associated with possible asbestos exposure means
mesothelioma should be ruled out first.

C Small cell carcinoma of the bronchus

Small cell carcinoma of the bronchus is incorrect. Small cell bronchial carcinoma is less likely
to present with pleural effusion, it is however known to present with syndrome of
inappropriate ADH secretion (SIADH), and can be associated with systemic symptoms
related to early metastasis.

D Squamous cell carcinoma of the bronchus

Squamous cell carcinoma of the bronchus is incorrect. These are most commonly central
tumours associated with hilar changes and local invasion.

E Tuberculous effusion

Tuberculous effusion is incorrect. There is no history of TB exposure, and the fact that CRP is
only slightly raised counts against significant infection.
46262

Rate this question:

Next Question

Previous Question Tag Question

Feedback End Review

Difficulty: Average

Peer Responses %
Q. Answered Flagged

Q1

Q2

Q3

Q4

Q5

Q6

Q7

Q8

Q9

0:00:36/03:00:00

A 23-year-old woman presents to the Emergency Department some 48hrs after consuming
30x500mg paracetamol tablets. She has begun to vomit and her friend told her she has
yellow eyes. She takes the oral contraceptive pill and drinks five pints of strong lager per day.
On examination her BP is 95/70 mmHg, pulse is 80 bpm and regular. She has jaundiced
sclerae. Abdomen is soft, with tenderness in the right upper quadrant.
Which of the following findings would be the most reliable indicator of poor prognosis in
this case?

A ALT >4x upper limit of normal (ULN)

B Creatinine 122 µmol/l

C Elevated bilirubin

D Glucose 9.1 mmol/l

E INR 2.2

Explanation 

E INR 2.2

INR >2.0 at or before 48 hours or >3.5 at or before 72 hours is indicative of poor outlook in
paracetamol overdose, supporting referral to a specialist unit. Peak elevation occurs at the
72-96hrs stage. Where INR is >6.5, referral for liver transplantation should be considered.

A ALT >4x upper limit of normal (ULN)

ALT>4x ULN is incorrect. Liver function tests are a poor predictor of outcome in paracetamol
overdose.

B Creatinine 122 µmol/l

Creatinine 122 µmol/l is incorrect. Serum creatinine of 200 or higher is considered an


indication for specialist referral. For creatinine >400, dialysis is usually considered.

C Elevated bilirubin

Elevated bilirubin is incorrect. Although elevated bilirubin is an indicator of hepatic


dysfunction, it is less predictive of poor outcome than abnormal clotting.
D Glucose 9.1 mmol/l

Glucose 9.1 mmol/l is incorrect. Hypoglycaemia, not hyperglycaemia is an indicator of poor


prognosis in patients with paracetamol overdose.
46250

Rate this question:

Next Question

Previous Question Tag Question

Feedback End Review

Difficulty: Average

Peer Responses %

Q. Answered Flagged

Q1

Q2

Q3

Q4

Q5

Q6

Q7

Q8

Q9

0:00:36/03:00:00

A 17-year-old man presents to the Emergency Department for review. He has suffered from
headaches and facial oedema over the past two weeks, and feels his symptoms are similar to
two episodes he had in the past that required treatment with steroid tablets. On examination
his BP is 132/82 mmHg, you confirm mild facial oedema. Chest is clear, abdomen is soft and
non-tender, his BMI is 23.
Investigations:

Hb 129 g/l

WCC 9.8 x 10 9/l

PLT 232 x 10 9/l

Na + 133 mmol/l

K+ 4.5 mmol/l

Creatinine 95 µmol/l

Albumin 24 g/l

ALT 20 U/l

ALP 90 U/l

Urine Protein +++

Which of the following is the most appropriate initial intervention?

A Cyclophosphamide IV

B Furosemide IV

C Azathioprine

D Oral methotrexate

E Oral prednisolone

Explanation 

E Oral prednisolone

This patient has an episode of nephrotic range proteinuria related to minimal change disease,
as exhibited by the previous episodes with similar symptoms that responded well to steroid
tablets. An initial steroid dose of 1mg/kg prednisolone is typical, and response is usually seen
within a 6-week period. In patients who fail to respond initially, a steroid sparing agent such
as azathioprine can be given in combination therapy.

A Cyclophosphamide IV

Cyclophosphamide IV is incorrect. Cyclophosphamide is usually considered an induction


therapy for serious vasculitis such as Wegener’s.

B Furosemide IV

Furosemide IV is incorrect. Furosemide is usually considered when patients have troublesome


oedema which does not respond, or is exacerbated initially by the commencement of steroid
treatment.

C Azathioprine

Azathioprine is incorrect. Azathioprine is a steroid-sparing agent used as oral therapy in


patients who fail to gain a complete response with prednisolone alone.

D Oral methotrexate

Oral methotrexate is incorrect. Methotrexate is not used as a steroid-sparing agent in


patients with minimal change disease.
46290

Rate this question:

Next Question

Previous Question Tag Question

Feedback End Review

Difficulty: Average

Peer Responses %

Q. Answered Flagged
Q. Answered Flagged

Q1

Q2

Q3

Q4

Q5

Q6

Q7

Q8 
0:00:36/03:00:00

A 78-year-old man is admitted to hospital with a left hemiparesis and altered consciousness.
He is on aspirin 75 mg, bendroflumethiazide 2.5 mg, atorvastatin 10 mg and glibenclamide 15
mg daily. His wife says that he has been unwell for a couple of days and has been off his food.
She has still been giving him all his medication.
Which of the following tests is going to be most helpful in finding an immediately reversible
cause for his symptoms?

A Blood glucose level

B Computed tomographic brain scan

C Electrocardiogram

D Serum creatinine level

E Troponin level

Explanation 

A Blood glucose level

Hypoglycaemia in the elderly is a not uncommon problem, even if they are only taking oral
hypoglycaemic agents. Glibenclamide has a long half-life and should therefore be avoided in
the elderly. The reduced food intake and the ongoing intake of medication in this patient are
likely to have caused hypoglycaemia, which can be associated with neurological symptoms in
the elderly. The neurological symptoms will resolve promptly with intravenous dextrose.

B Computed tomographic brain scan

Computed tomographic brain scan is incorrect. This would be the correct investigation if the
most likely diagnosis was stroke. Whilst it is likely that a patient presenting with hemiparesis
will need a CT scan of the brain to exclude acute stroke or bleed, looking for hypoglycaemia
and reversing it is very quick and should be done first. If no hypoglycaemia was found or the
patient did not improve with correction of the blood glucose, then a CT brain should be done
urgently.

C Electrocardiogram

Electrocardiogram is incorrect. Whilst an ECG is a good idea in this presentation, it is not the
most important investigation to find a reversible cause. If the patient had had a stroke then
an ECG would be very important to look for AF as an underlying cause.
D Serum creatinine level

Serum creatinine level is incorrect. This is a very important investigation that would give
indications of renal function, which may be due to dehydration in this patient. This would be
important as it may show how his oral hypoglycaemic had increased concentration and
caused hypoglycaemia, but it is not a cause of his symptoms directly.

E Troponin level

Troponin level is incorrect. This is a patient without chest pain and no suggestive ECG
changes of an acute cardiac event. A troponin should not be requested unless further
indications are found.
635

Rate this question:

Next Question

Previous Question Tag Question

Feedback End Review

Difficulty: Average

Peer Responses %

Q. Answered Flagged

Q1

Q2

Q3

Q4

Q5

Q6

Q7
Q. Answered Flagged

Q8 
0:00:36/03:00:00

A 29-year-old woman who has a history of Crohn’s disease for which she takes azathioprine
comes to the Dermatology Clinic for review. She has an ileostomy and is concerned because
an ulcerated lesion has begun to develop next to the stoma and has rapidly enlarged over the
past few weeks. On examination there is an ulcerated lesion adjacent to the stoma with a
raised border and a boggy / necrotic base. The base is purplish red in colour. It appears to
extend to the muscle layer. Bloods reveal a moderate elevation in inflammatory markers with
an ESR of 38 mg/l.
Which of the following is the most likely diagnosis?

A Behçet’s disease

B Contact dermatitis

C Cutaneous TB

D Necrobiosis lipoidica

E Pyoderma gangrenosum

Explanation 

E Pyoderma gangrenosum

The peristomal location, and ulcer appearance are consistent with a diagnosis of pyoderma
gangrenosum, which is seen with increased frequency in patients with a history of
inflammatory bowel disease. Diagnosis is usually made on clinical appearance, biopsy
findings are non specific, (revealing features of inflammation), and it is thought that biopsy
may lead to ulcer extension, meaning that it is therefore avoided if possible.

A Behçet’s disease

Behçet’s disease is incorrect. Behçet’s is more commonly associated with oral, genital
ulceration and uveitis.

B Contact dermatitis

Contact dermatitis is incorrect. Contact dermatitis due to the application of stoma bags
would be associated with concentric erythema rather than the ulcerated lesion seen here.

C Cutaneous TB
Cutaneous TB is incorrect. Although patients who are immunosuppressed are at increased
risk of TB, this would be considered an unlikely diagnosis in the absence of signs of
pulmonary disease.

D Necrobiosis lipoidica

Necrobiosis lipoidica is incorrect. Necrobiosis lipoidica is classically found on the shins, and
occurs in patients with diabetes mellitus. It begins as isolated papules which expand to form
plaque like lesions with a waxy atrophic centre.
46237

Rate this question:

Next Question

Previous Question Tag Question

Feedback End Review

Difficulty: Average

Peer Responses %

Q. Answered Flagged

Q1

Q2

Q3

Q4

Q5

Q6

Q7

Q8

Q9

0:00:36/03:00:00

A 42-year-old IV drug abuser who has been lost to follow up over the past year presents to
the Emergency Department with a palpable skin rash, fevers and increasing lethargy over the
past 3 months. She also has loss of sensation in both feet accompanied by distal motor
weakness. On examination her BP is 135/82 mmHg, pulse is 82/min and regular. She has an
extensive skin rash with palpable purpura affecting widespread areas of her arms and legs.
Abdomen is soft and non tender and her BMI is 19.5.
Investigations:

Hb 101 g/l

WCC 9.9 × 10 9/l

PLT 110 × 10 9/l

Na + 139 mmol/l

K+ 4.9 mmol/l

Creatinine 201 µmol/l

Urine blood+ Protein ++

ALT 232 U/l

ALP 291 U/l

Albumin 32 g/l

Total protein 61 g/l

Bilburin 32 µmol/l

Which of the following is the most likely diagnosis?

A Hepatitis A

B Hepatitis B

C Hepatitis C

D Hepatitis D

E Hepatitis E

Explanation 
C Hepatitis C

The clinical picture here is consistent with mixed cryoglobulinaemia and consequent renal
impairment, the most likely underlying cause is hepatitis C related to her history of drug
abuse. Where hepatitis C infection is confirmed, as seems likely here, clearance of virus with
an appropriate anti-hepatitis C regime is recommended ahead of any immunosuppressive
intervention.

A Hepatitis A

Hepatitis A is incorrect. Hepatitis A is a self-limiting virus transmitted by the faeco-oral route.

B Hepatitis B

Hepatitis B is incorrect. Although hepatitis B may lead to mixed cryoglobulinaemia and renal
impairment, it is approximately 10 times less likely to do so compared to hepatitis C.

D Hepatitis D

Hepatitis D is incorrect. Hepatitis D is seen in the context of hepatitis B infection which is less
likely to cause mixed cryoglobulinaemia.

E Hepatitis E

Hepatitis E is incorrect. Hepatitis E, like hepatitis A is a self-limiting virus, although it is


associated with increased risk of fulminant hepatitis in pregnant women.
46226

Rate this question:

Next Question

Previous Question Tag Question

Feedback End Review

Difficulty: Average

Peer Responses %
Q. Answered Flagged

Q1

Q2

Q3

Q4

Q5

Q6

Q7

Q8

Q9

0:00:36/03:00:00

A 49-year-old man with a history of chronic alcohol abuse comes to the Gastroenterology
Clinic for review. He is opening his bowels some 6-8 times per day, often with voluminous
stool which is difficult to flush away. He has lost 6kg in weight over the past year, and has dull
epigastric pain almost every day. He continues to drink 20 units of alcohol per day and
smokes ten cigarettes per day. On examination his BP is 122/82 mmHg, pulse is 80 bpm and
regular. Auscultation of the chest reveals occasional wheeze but nil else. Abdomen is soft,
although he is tender in the epigastrium. BMI is reduced at 19.
Investigations:

Hb 99 g/l

WCC 7.2 x 10 9/l

PLT 98 x 10 9/l

Na + 138 mmol/l

K+ 3.6 mmol/l

Creatinine 85 µmol/l

Albumin 25 g/l

ALT 122 U/l

ALP 185 U/l

Bilirubin 15 µmol/l

Faecal elastase (80mcg/g) (normal>100)

Which of the following medications is likely to be most effective in reducing the diarrhoea?

A Cholestyramine

B Codeine

C Loperamide

D Octreotide

E Pancreatin

Explanation 

E Pancreatin
The symptoms of malabsorption described here, coupled with abdominal pain, a significant
history of alcohol consumption and reduced faecal elastase, are highly suggestive of a
diagnosis of pancreatic insufficiency as a result of chronic pancreatitis. Pancreatin, a
pancreatic enzyme supplement is therefore the most appropriate intervention.

A Cholestyramine

Cholestyramine is incorrect. Cholestyramine is useful for the management of diarrhoea


associated with bile acid malabsorption and ileitis, although it would fail in this case to treat
the underlying pancreatic insifficiency.

B Codeine

Codeine is incorrect. Codeine may manage symptoms by slowing GI transit, although it will
not treat underlying malabsorption.

C Loperamide

Loperamide is incorrect. Similarly to codeine, loperamide slows gut transit and reduces stool
water, although it would not treat underlying malabsorption.

D Octreotide

Octreotide is incorrect. Somatostatin analogues are used to reduce diarrhoea associated with
carcinoid syndrome.
46279

Rate this question:

Next Question

Previous Question Tag Question

Feedback End Review

Difficulty: Average

Peer Responses %
Q. Answered Flagged

Q1

Q2

Q3

Q4

Q5

Q6

Q7

Q8

Q9

 External Links

Creon 10000 Capsules


medicines.org.uk/emc/medicine/2068
(https://www.medicines.org.uk/emc/medicine/2068)
0:00:36/03:00:00

A 32-year-old woman attends with her mother. She has always lived at home and never
worked. Over the past few weeks she has become increasingly anxious and begs her mother
not to leave her on her own at home. Her mother reveals that her daughter has always
needed a lot of reassurance and has never liked being left alone.
What is the most likely diagnosis?

A Borderline personality disorder

B Conversion disorder

C Dependent personality disorder

D Depression

E Histrionic personality disorder

Explanation 

C Dependent personality disorder

This woman has several features of dependent personality disorder, these include:

Excessive need to be taken care of, with others assuming responsibility for major areas
in life
Difficulty making decisions without excessive advice and reassurance
Submissive and clingy behaviour
Fear of separation
Difficulty initiating projects or working independently due to lack of self-confidence

A Borderline personality disorder

Borderline personality disorder is incorrect. This woman’s symptoms do not fit with
borderline personality disorder: unpredictable behaviour, emotional outbursts, rapid
fluctuations in emotional state, unstable personal relationships and a strong tendency
towards suicidal thinking and self-harm.

B Conversion disorder
Conversion disorder is incorrect. A conversion disorder is when a person has symptoms and
signs affecting voluntary motor or sensory function that cannot be explained by a medical
condition (ie medically unexplained symptoms). The symptoms are thought to be associated
with psychological factors such as conflict or stress. Examples include: blindness, paralysis,
swallowing difficulties, non-epileptic seizures.

D Depression

Depression is incorrect. Although depression and anxiety often coexist, there are no cardinal
features of depression described here (eg persistent sadness or low mood, marked loss of
interest or pleasure, disturbed sleep, fatigue, etc).

E Histrionic personality disorder

Histrionic personality disorder is incorrect. Typical features of histrionic personality disorder


include excessive emotionality and attention-seeking behaviour, rapidly shifting, shallow
emotions and deceitfulness. Patients may seem vain and selfish and behave in an
inappropriately sexualised or provocative way.

Personality disorders are deeply ingrained, maladaptive patterns of behaviour that can
be thought of as extremes of normal behaviour but form the basis of an individual’s
daily interaction and approach to life, such that their social functioning is affected
adversely. Diagnosis of a personality disorder is not appropriate until after the age of
16.

49500

Rate this question:

Next Question

Previous Question Tag Question

Feedback End Review

Difficulty: Difficult

Peer Responses %
Q. Answered Flagged

Q1

Q2

Q3

Q4

Q5

Q6

Q7

Q8

Q9
0:00:36/03:00:00

A 75-year-old man is given a routine health check by his family doctor. He has no specific
complaints and is not on any regular medication. Biochemical tests reveal a serum alkaline
phosphatase activity of 550 U/l (upper limit of normal (ULN) 150 U/l); serum creatinine
concentration is 132 µmol/l, calcium 2.42 mmol/l, phosphate 1.21 mmol/l, albumin 41 g/l.
Which of the following is the most likely cause of the high alkaline phosphatase?

A Osteomalacia

B Osteoporosis

C Paget’s disease of bone

D Primary hyperparathyroidism

E Renal osteodystrophy

Explanation 

C Paget’s disease of bone

A high alkaline phosphatase level is characteristic of Paget’s disease, a common condition in


the elderly. As here, patients may present with monoarticular Paget’s, or even be
asymptomatic in some cases. The normal calcium, phosphate and renal function help point us
towards the diagnosis. Bisphosphonates are the intervention of choice.

A Osteomalacia

Osteomalacia is incorrect. Alkaline phosphatase is associated with osteomalacia, although it


is also associated with low calcium. Low vitamin D is the underlying cause.

B Osteoporosis

Osteoporosis is incorrect. Elevated serum alkaline phosphatase is not associated with


uncomplicated osteoporosis, although it may be seen in the presence of an osteoporotic
fracture.

D Primary hyperparathyroidism

Primary hyperparathyroidism is incorrect. Primary hyperparathyroidism is associated with


elevated alkaline phosphatase, although it is also associated with hypercalcaemia.
E Renal osteodystrophy

Renal osteodystrophy is incorrect. The mildly elevated creatinine of 132 counts against renal
osteodystrophy as the cause of this patient’s symptoms, as does the lack of elevation in
serum phosphate.
1052

Rate this question:

Next Question

Previous Question Tag Question

Feedback End Review

Difficulty: Average

Peer Responses %

Q. Answered Flagged

Q1

Q2

Q3

Q4

Q5

Q6

Q7

Q8

Q9

0:00:36/03:00:00

A 32-year-old man who is known to be HIV positive presents to the Emergency Department
with a cough productive of rusty coloured sputum. He has been feeling progressively more
short of breath over the past 48hrs with worsening fevers. He has previously attended the
HIV Clinic but refused to start anti-retroviral therapy because he said he felt too well. On
examination he is pyrexial 38.9 °C, his BP is 105/80 mmHg, pulse is 90 bpm and regular.
There are signs of consolidation at the right base on auscultation of the chest.
Investigations:

Hb 135 g/l

WCC 15.9 × 10 9/l

Neutrophils 11.1 × 10 9/l

PLT 203 × 10 9/l

CRP 245 mg/l

Na + 138 mmol/l

K+ 4.3 mmol/l

Creatinine 122 µmol/l

Glucose 5.9 mmol/l

CD4 count 240/mm 3

CXR Right lower lobe consolidation

Which of the following is the most likely diagnosis?

A CMV pneumonitis

B Haemophilus influenzae pneumonia

C Mycoplasma pneumoniae

D Pneumocystis jirovecii

E Streptococcus pneumoniae pneumonia

Explanation 

E Streptococcus pneumoniae pneumonia


The CD4 count is above 200/mm 3, and community acquired pneumonia is common, including
in patients who are HIV positive. The short history of rusty coloured sputum, a cough and
fever, coupled with lobar consolidation fits well with Streptococcus as the likely cause.

A CMV pneumonitis

CMV pneumonitis is incorrect. Pulmonary infiltrates are more commonly associated with CMV
pneumonitis than lobar consolidation.

B Haemophilus influenzae pneumonia

Haemophilus influenzae (HI) pneumonia is incorrect. HI pneumonia occurs with greater


frequency in patients with previous existing chest disease, and is often less acute in onset.

C Mycoplasma pneumoniae

Mycoplasma pneumoniae is incorrect. Mycoplasma has a gradual onset of days to weeks, and
is usually characterised by a persistent, slowly worsening, cough, rather than the more acute
presentation seen here.

D Pneumocystis jirovecii

Pneumocystis jirovecii is incorrect. PJP is more usually associated with pulmonary infiltrates,
and occurs most commonly in patients with a CD4 count less than 200/mm 3.
46291

Rate this question:

Next Question

Previous Question Tag Question

Feedback End Review

Difficulty: Average

Peer Responses %
Q. Answered Flagged

Q1

Q2

Q3

Q4

Q5

Q6

Q7

Q8

Q9
0:00:36/03:00:00

You are reviewing the results of a lymph node biopsy from a woman who presented to the
ENT Clinic complaining of a mass on the left hand side of her neck consistent with a
Virchow’s node. The biopsy has demonstrated a number of small round cells where the
chromosomes appear to have aligned along the equator of the cell.
Which of the following represents the observed phase of cell division?

A Anaphase

B Interphase

C Metaphase

D Prophase

E Telophase

Explanation 

C Metaphase

In biopsies of patients with cancer a number of cells appear to be in metaphase because they
are rapidly dividing. In metaphase chromosomes align along the equator of the cell prior to
cell division into two daughter cells.

A Anaphase

Anaphase is incorrect. Anaphase occurs when replicated chromosomes move to opposite


poles of the cell.

B Interphase

Interphase is incorrect. Interphase is the phase between episodes of cell division.

D Prophase

Prophase is incorrect. Prophase is the phase of cell division when initial chromosome
condensation occurs.

E Telophase
Telophase is incorrect. In the telophase the nuclear membrane reforms in each daughter cell,
nucleoli reappear, and chromosomes unwind into chromatin.
46211

Rate this question:

End Session

Previous Question Tag Question

Feedback End Review

Difficulty: Average

Peer Responses %

Q. Answered Flagged

Q1

Q2

Q3

Q4

Q5

Q6

Q7

Q8

Q9

You might also like